PDF To Word
PDF To Word
PDF To Word
, LPT
1000
MMR
Victor A. Tondo Jr., LPT
Victor A. Tondo Jr. was a consistent Math contest champion, a national finalist for MTAP and PMO, a
national awardee for the Intel Science Fair,
a number theorist, a coach for international competitions, a review specialist, a licensed teacher, a regional
board top notcher, an accomplished gamer, and a pyrographer.
To my mom, Analyn N. Torizuka, for your all-out support in my entire life, even after I spent more than four years playing games on the computer. I love you so much! Without you, I am nothing.
To my sister, Vera-Lee A. Tondo, for always sensing my emotions and knowing everything that I am going through. Your unwavering support kept me sane, especially after the lowest points in my life.
To my daughter, Aira Quineisha R. Tondo, for being the sweetest, most loving daughter I could ever ask for. I have been loving you since the day you were born, and I will always be very proud of you.
To my teachers and professors, for molding me into the person I am today. You have taken good care of me, even with my special needs. You did your best to understand me, brought
out the best in me, and inspired me to be in the same profession as you. I will always be grateful for your gift.
To Ma’am Romana Valdecasa, for turning a poor nobody into someone who speaks and breathes Mathematics. You saw my potential and made me shine brighter than any star. You
really are my second mother – you figuratively gave birth to the successful Victor that I am now. I can never thank you enough.
To Ma’am Joaquina Birung, for always looking after me. You have been a very crucial factor to my success during high school and even during college. You took me under your wings.
You have been my teacher, my coach, my mentor, my cooperating teacher, and of course, one of my favorite mothers.
To Sir Pedro Suyu, to Sir Manuel Belango, and to the beautiful Ma’am Apolinaria Andres, for instilling in me the principles that a teacher must live by.
To my beloved Ma’am Sally Caldez, for never giving up on me. You are perhaps my second lola, and I will always love you. May you rest in peace.
To Ma’am Loida Calonia, for teaching me your Christian ways. While others have given up when they knew I am an atheist, you stood by my side patiently. When I was close to
committing suicide, you were there to guide me. You extended my life past the age of 30.
To my “daughters” Marone Khyme and Marby Alzate, for treating me not only as your tutor but also as a friend, as your tatay, and as your mentor. I love you both, and I have always been proud of you.
To my students, tutees, reviewees, subscribers, and friends, for always believing in me.
At sa aking Inang Bayan, inaalay ko sa iyo itong unang lathalain ko. Ang tanging hangad ko lang ay ang ika-uunlad mo.
Victor A. Tondo Jr., LPT
This is a free reviewer. All rights reserved.
1000 MMR
1. Expand your Mathematical vocabulary. You will find it difficult to learn concepts when your Mathematical vocabulary is limited.
2. Do not memorize the answers. Instead, know how to use the solution, how it works, and when it should be used.
3. Remember that there are many ways to deal with problems. Nobody wants a long solution when there are several shortcuts out there.
4. Try answering the items first before reading the solutions or explanations. This will give you an idea on your current standing and at the same time, what kinds of questions you should expect in the
exam.
5. Familiarize yourself with your own scientific calculator. Know where the different buttons are, like nPr, nCr, and n!. You should also know what these buttons do, and how to make them function.
Strategies in Answering the Mathematics Test
During the Licensure Exam
1. Keep track of the time. Before answering the questions, take note of the number of items and the time allotted for the exam. Estimate the average time you should allot for each item.
2. Scan the items. Go over the test booklet quickly to get an idea of the types of questions or problems in the test (such as word problems, analysis of graphs and tables, simple equations). This will help you determine the
appropriate pace to take in answering the test.
3. Pace yourself. Work quickly on topics you have mastered without being rash and careless. When you encounter a difficult question, skip it and move on to less difficult ones. When you have finished answering all the easy
questions, go back to the skipped items.
4. Keep your pen going. On your second reading of the item, create a picture of the problem and draw that picture on your scratch paper. Take down the relevant facts
(numbers, units, dimensions), specially what is asked for in the problem. Many items that seem difficult on first reading become easier to handle after the facts have been
taken down.
5. Visualize, draw diagrams. These help in answering word problems. Familiarize yourself with the types of diagrams in every topic as these may prove useful in solving problems.
6. Eliminate choices. The right answer can be determined if all but one of the choices are eliminated.
Don’t easily give up on difficult items.
7. Work backwards. This method is useful if you find it difficult to start a solution to a problem. To work backwards is similar to a trial-and-error on the choices. This can also be used to check if your answer is correct.
1000
Questions
C. 450 C. P24,800
2. Calculate the mean absolute deviation of the following numbers: 60, 80, 100, 75
and 95
A. 12.4
C. 16.1
A. 490
C. 1470
13. Ana and Beth do a job together in three hours. Working alone, Ana does
the job in 5 hours. How long will it take Beth to do the job alone?
9. At what rate per annum should P2400 be invested so that it will earn an A. 3 and 1/3 hours
C. 3 hours
B. 2 and 1/3 hours
D. 7 and 1/2 hours
interest of P800 in
8 years?
A.6½% B.5½%
C.4.17% D.6%
14. How much greater is the sum of the first 100 counting numbers than the sum of the first 50
counting numbers?
10. The area of a rectangle is (x2 + 2x - 8). If its length is x + 4, what is its A. 110
width?
A. x + 2 B. x - 2
C. x + 1 D. x + 6
17. The edges of a rectangular solid have these measures: 1.5 feet by 1½ feet by 3 inches. What is its 24. The legs of one right triangle are 9 and 12, while those of another right triangle are 12
volume in cubic inches? and
A. 324 B. 225 C. 972 D. 27
16. How much longer is the perimeter of the larger triangle than the perimeter of the smaller triangle?
18. In a certain school, the ratio of boys to girls is
5 is to 7. If there are 180 boys and girls in the school, how many boys are A.84 B.7 C.12 D.14
there?
A. 105
C. 45 25. An online shop sells a certain calculator for P950 and charges P150 for
shipping within Manila, regardless of the number of calculators ordered.
19. Ruben’s grades in 6 subjects are 88, 90, 97, Which of the following equations shows the total cost (y) of an order as a
90, 91 and 86. What is the grade that he should aim for in the 7th subject if he has to have an average of function of the number of calculators ordered (x)?
91?
A. 97 B. 95 C. 92 D. 89 A. y = (950
C. x = 950y
20. On a certain day, three computer technicians took turns in manning a 24-hour internet
shop. The number of hours Cesar, Bert, and Danny were on duty was in the ratio 3:4:5, respectively.
The shop owner pays them P50 per hour. How much would Danny receive for that day?
26. Which of these has the longest perimeter? A. A square 21 cm on a side
B. A rectangle 19 cm long and 24 cm wide
A. P 230
C. An equilateral triangle whose side is 28 cm D. A right triangle whose two legs are 24 and 32 cm
C. P160
21. A retailer buys candies for P90.25. The pack has 35 pieces of candies. If she sells each candy for 27. How many square inches are in 2 square yards?
P3.25, how much profit does she make?
A. 900
A. P11.50
C. 1296
C. P37.50
A. (2, 3/2)
C. (3, 3/2)
29. Aira takes ¾ hour to dress and get ready for school. It takes 4/5 hour to reach the school.
If her class starts promptly at 8:00 am; what is the latest time she can jump out of bed in order
not to be late for school?
A. 6:42 am B. 6:27 am
C. 6:57 am D. 7:02 am
1.
30. Which common fraction is equivalent to
215? 34. The hypotenuse of a right triangle is 25 feet. If one leg is 24 feet, what is the length of the other
leg?
A. 43/200 A. 6 ft. B. 5 ft. C. 20 ft. D. 7 ft.
C. 21/50
35. If two variables X and Y are directly related, which of these is NOT true?
A. When X is low, Y is also low.
B. As X increases, Y also increases. C. When X increases, Y
31. What are the next three terms in the progression 1, 4, 16 …? decreases.
D. A high Y is associated with a high X.
32. A man is 3 times as old as his son now. Four years ago, the sum of their ages was 36. Find the man’s
age now.
A. 33
C. 29
Author: Victor A. Tondo Jr., LPT C. 1/27
A. x
40. Factorize (x4 – 81) completely. A. (x-3)4
B. (x – 3)2 (x + 3)2 C. (x+3) (x-3) (x2+9) D.
(x+3)3 (x-3)
C. x = -1
37. A car travels D km in H hours. Which of the following expressions shows the distance travelled
by the car after M minutes?
A. MD/H B. 60MD/H
C. MD/60H D. 60HD/M 41. √8
A. 4√2
38. Find the surface area of a rectangular box whose dimensions are 30 cm x 40 cm x 50
cm.
A. 4700 cm2
39. If x – y = 3, then (y-x)-3 = ___. 43. The midpoint of P and (-7, 4) is (-3, 1). What are the coordinates of P?
A. (-5, 5/2)
A. 9
C. (1, -2)
A. 33%
45. What is the minimum value of f(x) = 3x2 + 6x +
7? C. 66%
A. 1
53. Given sin θ = 0.28, which of the following could possibly be cos θ?
46. If xy = 23 and x2 + y2 = 75, find x + y. A. 0.72
C. 0.96
A. 10.7845
C. 11.2155
A. 65o
48. A and B form a vertical pair. If m A = 3x and m B = 5x – 44, what is the value
of x? C. 50o
A. 50.5 B. 28 C. 22 D. 16.75
49. The angle of elevation from an observer to the top of a building is 30 o. If 55. If today is a Saturday, what day is 125 days from now?
A. Thursday B. Friday
the building is 50 meters high, how far is the observer from the building? C. Sunday D. Monday
A. 25
56. Car A is traveling towards the east at a speed of 35 kph, while car B is traveling
towards the west at 45 kph. If they left the same point at 1:00 PM, how far apart are they at
3:45 PM?
C. 200 km
parallelogram. If m 1 = 40o, what is m 57. Mr. Santos left the house at 1:00 PM and
traveled east at an average speed of 40 kph. His wife Mrs. Santos left the at 2:00 PM
A. 40o and traveled west at an average speed of 30 kph. How far apart are they at 4:00
PM?
51. Two parallel lines are cut by a transversal,
forming H and K. If the two angles are exterior angles on the same side of the transversal, what is A. 180 km
the measure of H if the
measure of C. 100 km
C. 100o
C. f(12)
C. (2x+2) ln (x2+2x)
62. Which of the following could be the value of x if x 3(mod 11)?
Author: Victor A. Tondo Jr., LPT
A. 33
68. If A and B are the roots of x2 + 7x + 15, what is AB?
A. 7√3
63. If C. 3√2
A. 12x + 8
C. 4097
C. (-4, 3)
65. Which of the following is a parabola that opens to the right? 70.24+12+6+3+1.5+…=____
A. 6y = (x+9)2 - 8
A. 48
C. -5x + 3 = (y-2)2 71. How many terms are there in the sequence
5, 13, 21, 29, …, 357?
A. 40
A. (6x + 5) (2x – 2)
72. How many ways can a group of 5 be selected from 5 boys and 5 girls if the group must
C. (3x + 2) (4x – 5)
contain
3 boys and 2 girls?
A. 151,200
C. 252
67. For which value of k does 4x2 + kx + 49 have only one root?
A. -28 B. -14 C. 7/2 D. -7/4
C. 1/6
̅̅
74. C is the midpoint of AB where A is at (-3,4) and B is at (7,-10). Find the coordinates of C.
A. (5,-7) B. (-5,7) C. (2,-3) D. (-2,3)
A. 42
77. Two parallel lines are cut by a transversal to form X, Y, and Z. Given that X and Y
are
alternate interior angles while Y and Z are interior angles on the same side of
the 81. Solve for x: 2log2 3 – log2 18 = x
transversal, find m
A. ½
A. 40o
C. 130o
82. Twinkle Bucks has four serving sizes for their milk tea: Small, Medium, Large,
and Extra Large. What level of data are they using for their serving sizes?
A. nominal B. ordinal
C. interval D. ratio
78. The measure of each interior angle of a regular polygon is 144o. How many vertices does it
have?
A. 36 83. After receiving a 20% markup, a bag was sold for P960. How much was it
originally?
A. P1152
C. P800
C. x < -9 x > 3
Author: Victor A. Tondo Jr., LPT
84. Given ̅̅BT bisects ABC and m ABT = 40o, find m ABC.
A. 126
C. 3024
C. 405 π cm3
A. -1
C. √7
87. A 10 ft ladder leans against a wall, forming a 30o angle with it. How high on the wall does it reach?
91. If two numbers have a product of 71 and the sum of their squares is 147, what is their
sum?
A. 5 ft
A. -17
C. 12√3
C. 10 √3
93. How many 3-digit numbers can be formed using the digits 0, 1, 2, 3, 4 and 5 if repetition is not
allowed?
A. 60 B. 80 C. 100 D. 120 100. A pipe can fill a pool in 6 hours while another pipe can drain empty the
pool in 15 hours. How long will it take to fill the pool if both pipes are open?
94. How many ml of 20% acid must be added to
400 ml of 50% acid to make a 30% acid solution?
A. 9 hours
A. 1000 ml
C. 9.45 hours
C. 800 ml
96. It takes 28 men a total of 24 days to build a house. How long would it take 32 men to build a
similar house?
A. log2
C. log2
A. 28 days
C. 21 days
103. The surface areas of two spheres are 12 π cm2 and 108 π cm2. What is the ratio of their
volumes?
C. 1:27
A. undefined
C. 8
104. The volume of a regular hexahedron is 64 in3. How long is each side?
A. 2 in B. 4 in C. 6 in D. 8 in
98. A box contains 7 red, 8 blue, and 9 white balls. When taking two balls in succession, what is the
probability that both balls are white? 105. Which of the following statements is ALWAYS true?
A. The square of a prime number is odd.
A. 9/64
B. The sum of two consecutive even numbers is divisible by 4.
C. Any even number is composite.
D. The product of two consecutive even numbers is divisible by 8.
C. 7/64
C. 108 cm3
A. 300 π cm3
C. 200 π cm3
C. 9√2
109. Which quadrilateral has two congruent diagonals that bisect each A. 66
other?
A. kite
C. rectangle
B. isosceles trapezoid
D. rhombus
116. Find the remainder when
x4 – 5x3 + 6x2 + 2x + 1 is divided by (x – 2).
117.
B.13 C.9
̅̅ B. ̅̅TC ̅̅ ̅̅
60.
A. MC C. MT D. CT
Twelve years ago, Fe was twice as old as Sita. How old is Sita now?
A. 18 B. 24 C. 30 D. 36
A. 120 cm B.
C. 120√2 cm D. 24√5 cm
A. y-1 = √ 25+5
B. y-1 = √ 25 –5
C. y-1 = √ + 25 +5
D. y-1 = √ +25–5
y = 4x – 11.
A. (-4/3, 0) B. (4/3, 0)
A. 60
1000 MMR
A.X=
√
A.
C.X=
C.
123. Two buses leave the same station at 8:00 pm. One bus travels north at the
rate of 30 kph and the other travels east at 40 kph. How many kilometers apart are the
buses at 10 pm?
A. 140 km B. 100 km
C. 70 km D. 50 km
127. RNHS has 130 quizzers. 67 of them are Math, 60 are Science, and 20
are quizzers for both Math and Science. How many quizzers are neither Math
nor Science?
A.0 B.13 C.17 D.23
124. A bus drove for 6 hours at 75 kph and 4 hours at 80 kph. What was its average
speed?
A. 76 kph B. 77 kph
C. 77.5 kph D. 78 kph
125. 18 students failed a quiz. They 128. Mr. Tondo has P100,000 to invest, from which he wants to earn P5600 per
represent year. Bank A offers 5% per annum while Bank B offers 6%. How much should he invest
30% of the class. How many students passed the quiz? at Bank B?
A. P45,000
C. P55,000
129. Evaluate
A. -38
131. Carla and Diana are on a seesaw. Carla weighs 50 kg and sits 168 cm to the left of
the fulcrum. If Diana weighs 60 kg, how far to the right of the fulcrum must she sit to balance
the seesaw?
A. 140 cm
C. 201.6 cm
132. Twenty guests shake hands with each other. If each guest is to shake hands with all the other
guests, how many handshakes will be made?
A. 400 B. 380 C. 200 D. 190
134. The longest chord of a circle is 80 cm. How long is its radius?
A. 20 cm
C. 20√2
A. 5
136. Find the largest area of a rectangle whose perimeter is 100 cm.
A. 2500 cm2
C. 625 cm2
A. 12:30 AM
C. 1:50 AM
B. 12:30 PM
D. 1:50 PM
2 hours later and traveled 80 kph on the same route. What time will Bus B catch up
with Bus A?
A.6PM B.9PM
C.11PM D.1AM
138. Find the product of two numbers whose GCF is 24 and LCM is 120.
A. 2880 B. 1440 C. 720 D. 360
139. The salary of 4 men for 5 days is P9,000. How much is the salary of 5 men for 6
days?
A. P12,000
144. The average of x+5, 2x-4, and x+7 is 20. Find x.
A.18 B.13 C.9 D.8
C. P13,500
145. Mia is 16 years younger than Kia. 13 years ago, Kia was thrice as old as Mia. What is Kia’s
present age?
A. 43 B. 40 C. 37 D. 34
140. The average grade of eleven students is 83. If the average of six of these students is 88, what is
the average of the other 5 students?
A. 77
C. 130%
Author: Victor A. Tondo Jr., LPT A. 1.142857
146. Insert one term between 18 and 32 to make a geometric sequence.
A. 20 B. 24 C. 25 D. 27
C. 1.5
147. There are 100 pigs and chickens in a farm, all of which are healthy. If there are 340 legs in total,
how many pigs are there?
A. 70
151. What is the highest possible product of two numbers if their sum is 45?
C. 60
A. 506
C. 506.5
148. Adam can do a job alone in 8 hours, while Bam can do the same job in 12 hours. One day,
they worked together for 1 hour before Bam left Adam to finish the job. How long will it take Adam
to finish the remaining job?
152. Which statistical test is used for comparing observed frequencies to expected
A. 6 hrs 50 mins frequencies?
A. ANOVA B. t-test
C. Pearson R D. Chi Square
C. 6 hrs 30 mins
153. The product of two consecutive odd counting numbers is 1443. What is their sum?
A. 76 B. 78 C. 80 D. 82
1000 MMR
A. 2200o
C. 1520o
154. Given
163. Find the equation of the line passing through (2,7) and (-3,-
3).
A. y = 4x -1 B. y = 3x + 1
C. y = 3x + 6 D. y = 2x + 3
find lim
A. 4
C. 0 164. In which quadrant can we find θ if tan θ < 0 and sin θ > 0?
A. First Quadrant B. Second
Quadrant C. Third Quadrant D.
Fourth Quadrant
165. Find the equation of the line passing through the point of origin and
(3,4).
156. If the sum of the supplement and the complement of an angle is 124, what is the
angle? A. y =
A. 71 C. y =
C. y
A.
C. 60o
172. Mrs. Pasay saved P250 after buying a phone with a 10% discount. How much did she pay for the
phone?
179. Dexter is twice as heavy as Pablo. Ming is 4kg heavier than Pablo. The sum of their masses is
164kg. How heavy is Dexter?
A. 40 kg B. 44 kg
C. 80 kg D. 88 kg
A. P2500
C. P2000
180. A circle is drawn inside a triangle such that it is tangent to the sides of the triangle. Its center
will be the triangle’s ___________________.
A. Incenter B. Circumcenter
C. Centroid D. Orthocenter
173. A book was sold for P270 after a 10% discount was given. How much was the book
originally? 181. Rayon can do a job in 3 hours, while Carlyn can do the same job in 7 hours. How long will it take
them to finish the job by working together?
A. 2.1 hours B. 2.5 hours
C. 5 hours D. 10 hours
A. P330
C. P297
182. This line is perpendicular to one side of the triangle passing through the opposite
vertex.
A. Longitude B. Median
C. Altitude D. Bisector
A. 36√3
A. 22
C. 44
̅̅
176. ∆ABC is similar to ∆DEF. AB is 9 cm long while ̅̅DE is 12 cm long. If the area of ∆ABC is
27 cm2, what is the area of ∆DEF?
A. 36 cm2
C. 60 cm2
186. How many ways can the letters of the word BANANA be rearranged?
A. 720 B. 240 C. 120 D. 60
194. If three-fourths of a number is 33 more than its one-fifth, what is that
number?
A. 240
Celsius?
A. Nominal B. Ordinal
C. Interval D. Ratio
C. [(3 + 3)2]2
189. What is formed when a plane intersects a cone parallel to its circular
base?
A. ellipse
C. circle
B. hyperbola
D. parabola 196. Which of the following has an undefined slope?
A. a vertical line B. a
horizontal line
C. a line parallel to the x-axis D. a diagonal line
190. In which non-Euclidean model for geometry can we have any given line ℓ
and a point A which is not on ℓ, wherein all lines through A will intersect ℓ?
A. hyperbolic B. elliptic
C. Saccheri D. Pythagorean
197. In solid geometry, what do you call a solid bound by polygons?
A. multigon B. tessellation
C. porygon D. polyhedron
192. Which numerical system makes use of dots and horizontal lines, and shell shapes for
zero?
A. Egyptian B. Roman
C. Greek D. Mayan
199. Which statistical test must be used in testing the significance of group differences
between 2 or more groups?
A. Chi Square B. t-test
C. ANOVA D. Pearson R
C. P233,050
A. 5
C. Quadrilateral HEAD which has one pair of congruent perpendicular bisecting diagonals. D.
Quadrilateral FROG which has 4 right angles.
204.
A. SAS
Which of the following is not a triangle congruence postulate?
B. ASA C. SAA D. AAA
210. What percent of 80 is 55?
A. 145.45%
205. If A is at (-8,5) and B is at (4,-11), find C if C is three-fourths the way from A to C. 68.75%
B.
211. The hypotenuse of a right triangle
A. (1, -7)
40
measures
cm. Find its area if one angle measures 30o.
C. (1, 1)
A. 100√3
212. Nine cans of soda and four hamburgers cost a total of P257. Five cans of
soda and seven hamburgers cost a total of P224. How much is a can of soda?
A. P17 B. P19 C. P21 D. P23
213. The product of two consecutive even numbers is 728. What is the smaller
number?
A. 22 B. 24 C. 26 D. 28
A. (
C. x
216. After getting a 20% discount, Mr. Lopez paid P4,000 for a gadget. How much was its
original price?
C. x = 1
217. When a number is increased by 3, its square increases by 111. By what does its square increase
when the number is increased by 6?
A. 222 B. 240 C. 444 D. 480
222. The diagonal of a rectangular prism is 13 cm long. If it is 3 cm thick and 12 cm long, how
wide is it?
C.4√3
C. y
Author: Victor A. Tondo Jr., LPT A. 10
223. Which of the following is not a function? A. y = x 2 + 2017x – 2017
B. y = |2017x| - 2017
C. y = √2017 + 2017
D. y2 = x + 2017
224.12+17+22+27+…+117=_____
A. 1409 228. Find the y-intercept of 2x + 3y = 4.
C. 1419
A. 1704
C. (0, 10)
226. When a number is increased by 4, its square also increases by 168. What is this
number?
A. 15
trinomial:
A. √3
240. Find the average rate of change of y = x3 – 2x + 3 from x = 0 to x
= 3.
A.5 B.6 C.7 D.8
C.
242. Gian has 8 more P5 coins than P1 coins. If he has a total of P106, how many P5 coins does he
have?
A. 13 B. 15 C. 17 D. 19
233. What is the degree of the polynomial 9x4 + 5x3 – 2x2 + 3x –
17?
A. 4
243. After using half of her budget on bills, one-third on groceries, and P270 on a shirt, Mrs. D still
had P130 left. How much was her budget?
A. P2400
C. P3000
234. log2 32√2
A. 2.5
A. 3
235. If y = √3
79
C. x = √ A. 79
C. 80
236. Which of the following is a pair of parallel lines? 246. The average grade of 23 students in Section A is 86, while the average grade of 27 students in
A. y = 2 and x = 2 Section B is 91. What is the average grade of all
B. 12x + 13y = 14 and 13x + 14y = 15 C. y = 3x + 8 and 3y = x 50 students in both sections?
D. 4x + 5y = 6 and 8x + 10y = 21
238. Find the altitude to the hypotenuse of a right triangle whose sides measure 5 cm, 12 cm, and
13 cm.
A. B. C. D. 26
This is a free reviewer. All rights reserved.
1000 MMR
247. Find the axis of symmetry of y = 3x2 – 5x. A. x =
Author: Victor A. Tondo Jr., LPT
C. x = 254. Find the equation of the circle with center at (2, 3), passing through (5,
-1).
A. x2 + y2 + 4x + 6y = 0 B. x2 + y2 + 4x +
6y = 12 C. x2 + y2 – 4x – 6y = 0 D. x2 + y2 –
4x – 6y = 12
A. 12
256. Find the equation of the horizontal line passing through (-3, 4).
A. x = -3 B. x = 4
C. y = -3 D. y = 4
249. Mr. C travels for 2 hours at a speed of 38 kph and then north for 3 hours at a speed of 53 kph.
What is his average speed?
A. 44 kph B. 45.5 kph
C. 47 kph D. 48.5 kph
257. Which of the following lines passes through the point (3, -2)?
A. y = x + 5 B. y = 2x – 8
C. y = 5 – x D. y = 5 – 2x
250. Victor, Chris, and Aira volunteered to teach at a nearby daycare. Chris worked for 2
hours less than Aira. Victor worked twice as many hours as Chris. Altogether, they worked for 58
hours. How many hours did Victor work?
A. 14 B. 16 C. 28 D. 32
258. Given that I(2, -3) is the midpoint of V(-4, 5) and C, find the coordinates of
C.
A. (-1, 1)
251. What conic figure does the equation x 2 + y2 + 4x = -4
form?
A. Real circle B. Degenerate circle C. (8, -11)
C. Imaginary circle D. Ellipse
252. What conic figure does the equation x2 + y2 + 8x – 6y = -100 259. The endpoints of the diameter of a circle are A(9, -5) and B(-3, 11). What is the equation of the
form? circle?
A. Real circle B. Degenerate circle
C. Imaginary circle D. Ellipse
253. Find the center of x2 + y2 + 6x – 10y = 2. A. (x – 3)2 + (y – 3)2 = 100 B. (x – 3)2 + (y – 3)2 =
400 C. (x + 3)2 + (y + 3)2 = 100 D. (x + 3)2 + (y +
A. (6, -10)
3)2 = 400
C. (-3, 5)
260. Find the distance between the line
3x + 4y – 5 = 0 and the point (8, -1).
A. 2
A. (2, -3)
C.
C. (2.5, -3.5)
D.
261. Find the area of the triangle whose vertices are X(-9, -3), Y(-2, 8), and
Z(5, 1).
A. 61 B. 62 C. 63 D. 64
262. Which of the following is outside the circle defined by the equation (x – 3) 2 + y2 =
40?
A. (5, 6)
C. (0, -5)
263. Which of the following is parallel to the line defined by the equation y =
3x – 4?
A. y + 3x = 5 B. x +
3y = 6
C. y = x+7
D. y = 3x + 8
264. Which of the following is perpendicular to the line defined by the equation y = 3x –
4?
A. y + 3x = 9
C. y =
265. Which of the following is coincidental to the line defined by the equation y = 2x
+ 13?
A. y + 2x = 13
C. y =
Author: Victor A. Tondo Jr., LPT
267. Which of the following equations pertain to a parabola that opens to the
right? 271. Find the equation of the directrix of the parabola defined by (y – 2) 2 = -4(x +
A. 4(y + 3) = (x – 2) 2 B. -3(y – 4) = 3).
(x + 5)2 C. (y – 6)2 = 5(x + 1) D. (y + A. x = -2
C. y = -4
B. x = 2
D. y = -2
8)2 = -2(x – 3)
272. Find the coordinates of the focus of the parabola defined by -12(y – 4) = (x +
268. Which of the following equations pertain to a parabola that opens 5)2.
downward?
A. 4(y + 3) = (x – 2) 2 B. -3(y – 4) = A. (-5, 7)
(x + 5)2 C. (y – 6)2 = 5(x + 1) D. (y +
8)2 = -2(x – 3)
C. (-8, 4)
269. How long is the latus rectum of the parabola defined by 12(y – 4) = (x
+ 3)2? 273. In which quadrant would G(3,-4) fall? A. First quadrant B. Second quadrant C. Third
A.12 B.6 C.4 D.3 quadrant D. Fourth quadrant
270. How far is the vertex from the directrix of the parabola defined by 16y 274. Find the distance between the parallel lines y = 3x √+ 9 and y = 3x – 12.
= x2? A.7
√
B.21 C. D.
A.16 B.8 C.4 D.2
C. (4, 10)
283. The hypotenuse of a 30-60-90 triangle is 48 cm long. How long is its shortest
side?
276. Which of the following pertains to a circle that is concentric with (x – 3) 2 + y2 =
24?
A. x2 + y2 – 6x + 3 = 0 B. x 2 + y2 + 6y –15 = 0
C. x2 + y2 + 6x – 25 = 0 A. 24 cm
D. x2 + y2 + 6x + 6y – 24= 0
C. 24 √3
285. A hundred cards are numbered 1 to 100. What is the probability of drawing a card whose
number is divisible by seven?
A. B. C. D.
A. 1.309o
A.
C. 150o
C. 323o
C. (3x – 2) (x + 1)
A. tan θ =
C. cos θ =
1000 MMR
289. A certain University has a dormitory. If 10 students stay in a room, 24 290. Which of the following is not between and ?
students will not have a room. If 12 students stay in a room, there will A. B. C. D.
be 6 vacant beds. How many rooms are there in the dormitory? How
many students are staying in the dormitory? 291. Which value describes the position of C?
A. -0.75 B. -0.6 C. -1.25 D.
A. 116 B. 115 C. 114 D. 113
-1.4
Author: Victor A. Tondo Jr., LPT
295. Determine the relation that matches the table of values.
292. Choose the correct value of (x + y)(x – y) when x = 3.5 and y = –8.7
A. -63.44
A. y = 21 – x
C. -148.84
model.
A. 2x2 + x + 4
C. 3x2 – x + 4
C. -2x2 + 3x – 2
A. A and B
C. B and D
B. A and C
D. B and C
A. 45o, 55o
294. Determine which equation is equivalent to 4x – 1 = 11. C. 48o, 48o
A. 15 sin 72o
C. 72 sin 72o
A. 42n + 3
306. The greatest number of Fridays that can occur in a 75 day period
is:
A. 10 B. 11 C. 12 D. 13
A. 2
C. 200
303. The graph below represents the motion of a car. The graph shows us that the car
is:
A.
B.
accelerating
standing still
C. travelling north-east
D. travelling at a constant speed
A. -8132
312. A weather station recorded the amount of rain that fell during an 8-
hour time frame using a rain gauge. The findings are recorded in the
graph below.
309. What is 5 × 10–4 written in standard
notation?
A. 0.00005
C. 5,000
A. -25
Between which hours was the rate at which the rain fell greater than the rate at which the rain fell
between hours 0 and 1?
A. B. A. between hours 3 and 4
B. between hours 4 and 5
C. between hours 5 and 6
D. between hours 7 and 8
This is a free reviewer. All rights reserved.
1000 MMR Author: Victor A. Tondo Jr., LPT
312. Each day of the month, Carl earns an
allowance, in cents, equal to the square of that date of the month. Which is a number of cents Carl 316. Praetor jogged on a path that was 2 miles long, took a break, and then jogged back
could earn in a single day? along the same path to where he started. He jogged at different speeds for different distances
A. 21 B. 31 C. 64 D. 111 along the path as shown in the graph.
314. Rayon has a piece of rectangular paper that is 12 inches wide by 16 inches
long. He drew a straight line along the diagonal of the paper. What is the length of the
line Rayon drew?
Between which times did Praetor jog the
A. √28
fastest?
A. 0 minutes and 10 minutes
B. 10 minutes and 25 minutes
C. 25 minutes and 30 minutes
D. 30 minutes and 60 minutes
C. 20 inches
317. Which expression has a value of -2?
A. |2| + |-4| B. |-2| – |4|
C. |4| – |-2| D. |-4| + |2|
A. x
C. x
320. The table below shows the resting heart rates in beats per minutes of six students. The
rate, 40 beats per minute, seems to be an outlier. Which measure of central tendency changes the
least by dropping 40 from the data?
Heart
Rate
A. mean
C. mode
321. The sum of a number, n, and 5 is subtracted from 8. Which expression represents this statement?
A. 8 – (n + 5) 324. In the spinner, what is the probability of the arrow NOT landing on the space with the
∆?
C. (n + 5) – 8
A. 1,000
326. The lengths of two sides of a triangle are 8 inches and 13 inches. Which of the C. 100,000
following represents x, the possible length in inches of the remaining side of the triangle?
A. 5 < x < 21
C. x < 5 or x > 21
329. Which of the following expressions has a value of 0?
A.(2–3)–(2–3)
C.(2–3)+(-3+2)
C. (5x + 3) (2x – 7)
A. 7√7
ellipse defined by
A.
333. Let A be a set such that A = {v, w, x, y, z}. How many subsets does set A
have?
A. 5
A. x = 3
C. x = -1
C. 3780 x3y4
336. The shell shape , as used in the Mayan numeral system, is the symbol for which
number?
338. Aira is six years older than Zayne. Six years ago, she was twice as old as he. How old is Aira
A. 100 now?
A. 21 B. 18 C. 15 D. 12
337. Which of the following is irrational? 339. The two parallel sides of a trapezoidal lot measure 100m and 70m. If these sides are 80m
apart, what is the area of the lot?
A. 13600 m2
A. 0.125
C. 3400 m2
Author: Victor A. Tondo Jr., LPT
343. A ride in a Feak Taxi costs P25.00 for the first km and P10.00 for each additional
km. Which of the following could be used to calculate the total cost, y, of a ride that was x
km?
340. If y = x and y = 2x A. y = 25x + 10 B. y = 10x
+ 25
A. x = -2 C. y = 25(x
C. x = 0 D. y = 10(x
341. If the difference between the squares of two consecutive counting numbers is 49, what is the 344. Which of the following points is in the fourth quadrant?
larger number?
A.99 B.49 C.25 D.7
A. (3, 4)
C. (3, -4)
342. Rayon needed to find the perimeter of an equilateral triangle whose sides measure x + 4 cm
each. Jake realized that he could multiply
3 (x + 4) = 3x + 12 to find the total perimeter in terms of x. Which property did he use to
multiply?
345. The distance from the sun to the earth is approximately 9.3 × 10 7 miles. What is this
distance expressed in standard notation?
A. Associative Property of Addition
B. Distributive Property of Multiplication over Addition A. 9,300,000,000
C. Commutative Property of Multiplication D. Inverse Property of
Addition C. 93,000,000
347. The sum of the square of a number and 12 times the number is 27. What is the smaller possible
value of this number?
A.-9 B.-3 C.3 D.9
C. (3p + 1) (p – 5)
A. 2
C. 1,024
352. Jay bought twenty-five P4.57 stamps. How much did he spend?
A. P 104.25
C. P 119.75
355. Mulan and Lilo are competing to see who can sell the most candy bars for a
fundraiser. Mulan sold 4 candy bars on the first day and 2 each day after that. Lilo sold 7 on the
first day and 1 each day after that. On what day will they have the same number of candy bars
sold?
A. 7th B. 6th C. 4th D. 3rd
A. -3x2 +
C.
Author: Victor A. Tondo Jr., LPT 360. =
358. The cost of renting a bike at the local bike shop can be represented by the A. B. C. D.
equation
y = 2x + 2, where y is the total cost and x is the number of hours the bike is rented. Which of the
following ordered pairs would be possible number of hours rented and the corresponding total
cost?
361. The sum of two angles is 180°. The measure of one angle is 34°
A. (0, 2) greater than the measure of the other angle. What is the measure
of the smaller angle?
A. 74° B. 73° C. 72° D. 71°
C. (6, 2)
359. The distance from the earth to the moon is approximately 240,000 miles. A. ( 2,
3x 2y = 12
3) B. (2, 3)
What is this distance expressed in scientific notation? C. (3, 2) D. ( 3, 2)
C. 2.4 × 105
C. 60o
371. How many 3-digit numbers can be made using the digits 5, 6, 7, 8, 9, and 0 if repetition is not
allowed?
A. 80 B. 100 C. 120 D.
140
365. Samantha owns a rectangular field that has an area of 3,280 square meters.
The length of the field is 2 more than twice the width. What is the width of the field?
A. 40 m
372. A researcher is curious about the IQ of students at the Utrecht University. The entire
C. 82 m group of students is an example of a:
A. parameter B. statistic
C. population D. sample
A. (2, 4)
C. x < -5
C. (2, 1)
375. How many solutions are there for the following system of linear
equations?
369. Given f(x) = 7x3 – 3x2 + 2x – 9, f(2) = A. only one solution
-3x + 5y = 6
6x 10y = 0
B. two solutions
C. infinitely many solutions
D. no solution
A. 56
381. What is the radius of the circle defined by (x + 2)2 + (y – 3)2 = 16?
377. What is the probability choosing only one vowel when three letters are randomly selected from A.256 B.16 C.8 D.4
the word NUMBERS?
A. B. C. D.
A.
C.A<B+2
379. Statistical techniques that summarize and organize the data are classified as
what?
A. Population statistics B. Sample statistics C.
Descriptive statistics
D. Inferential statistics
386. Find the slope of the line passing the points A(2, 3) and B(-7, -15).
A.1 B.-1 C.½ D.2
384. Find the equation of the line passing (1, 4) with slope equal to 5.
A. y = 5x + 3 B. y = 5x + 1
C. y = 5x – 1 D. y = 5x – 3
388. In now many ways can the letters AAABBCDEEE be arranged in a straight
line?
A. 50,400
385. The seminar rooms in the library are identified by the letters A to H. A
researcher records the number of classes held in each room during the first C. 12,600
C. 54.15o
A.
396. There are ten true - false questions in an exam. How many responses are
possible?
A. 1024 B. 256 C. 20 D. 10
C.
392. A teacher gave a statistics test to a class of Geography students and computed the 398. In how many ways can 4 girls and 5 boys be arranged in a row so that all the four girls are
measures of central tendency for the test scores. Which of the following statements cannot be an
accurate description of the scores? together?
A. The majority of students had scores above the mean.
B. The majority of students had scores above the median.
C. The majority of students had scores above the mode.
A. 4,320
D. All of the above options (A, B and C) are false
statements. C. 17,280
A. 784 π cm2
C. 28 π cm2 399. A box contains 8 batteries, 5 of which are good and the other 3 are defective. Two
batteries are selected at random and inserted into a toy. If the toy only functions
with two good batteries, what is the probability that the toy will function?
A. B. C. D.
394. Find the length of each side of an equilateral triangle whose perimeter is
90 cm.
A. 45 cm B. 30 cm
C. 22.5 cm D. 10 cm
400. IQ tests are standardized so that the mean score is 100 for the entire group of
people who take the test. However, if you select a group of 50 who took the test, you
probably would not get
100. What statistical concept explains the difference between the two
means?
A. Statistical error B. Inferential error
C. Residual error D. Sampling error
403. Surface area and volume, center of gravity, and hydrostatics are some of the studies of which
Mathematician?
A. Apollonius B. Euclid
C. Archimedes D. Hipparchus
404. The book Philosophiæ Naturalis Principia Mathematica, more fondly known simply as
Principia, is the work of which Mathematician?
A. Euclid B. Newton
C. Einstein D. Archimedes
405. A researcher is interested in the travel time of Rayon’s University students to college. A
group of 50 students is interviewed. Their mean travel time is 16.7 minutes. For this study, the
mean of 16.7 minutes is an example of a
A. parameter B. statistic
C. population D. sample
A. 2, 5, 8, 11
412. Which of the following terms does NOT describe the number 9?
A. rational number B. integer
C. real number D. prime number
422. The sum of five consecutive integers is 215. What is the largest of these
integers?
415. When five is added to three more than a certain number, the result is 29. What is the A. 43 B. 44 C. 45 D. 46
number?
A.24 B.21 C.8 D.4
423. You go to the cafeteria for lunch and have a choice of 4 entrees, 5 sides, 5 drinks, and 4
desserts. Assuming you have one of each category, how many different lunches could be made?
416. The math club is electing new officers. There are 3 candidates for A. 18 B. 81 C. 40 D. 400
president, 4 candidates for vice-president, 4 candidates for secretary, and 2
candidates for treasurer. How many different combinations of officers are
possible?
A. 13
424. What can be said about the following statements?
i. Any quadrilateral with four congruent sides is a square.
C. 480
ii. Any square has four congruent sides. A. Only the first statement is
true.
B. Only the second statement is true. C. Both statements are
true.
D. Both statements are fall.
417. Twelve points lie on a circle. How many cyclic quadrilaterals can be drawn by
using these points? [Note: Cyclic quadrilaterals are quadrilaterals whose vertices are on a
circle.]
425. Out of 6 boys and 4 girls, a committee of 5 has to be formed. In how many ways can this be
A. 48 done if we take 2 girls and 3 boys?
A. 120 B. 186 C. 240 D. 256
C. 11,880
A. x = B. x =
419. When 18 is subtracted from six times a certain number, the result is 42. What is the
number?
A. 10 B. 4 C. -4 D. -10
A. y = 2x + 1
C. y = x + 2
C. 34.97 cm
A. 800 mL
C. 900 mL
441. Which of the following is equidistant from the vertices of the triangle?
A. circumcenter B. orthocenter
C. incenter D. centroid
448. In terms of a conditional statement, what is the statement formed by exchanging and negating
the antecedent and the consequent?
A. inverse B. converse
C. adverse D. contrapositive
442. Which of the following is equidistant from the sides of the triangle?
A. circumcenter B. centroid
C. orthocenter D. incenter
449. What is formed when the hypothesis and the conclusion of the conditional statement are
interchanged?
A. converse B. inverse
443. Although rarely used in proving, what is the extra line or line segment drawn in a figure to help C. adverse D. contrapositive
in a proof?
A. base line B. auxiliary line
C. converse line D. Euler’s line
450. What is formed when both the hypothesis and the conclusion of the conditional statement are
negated?
A. converse B. inverse
444. What is the measure of V in the following figure? C. adverse D. contrapositive
A. 60o B. 65o C. 70o D. A. If two angles are not congruent, then they do not have the same measure.
B. If two angles have the same measure, then they are congruent.
C. If two angles do not have the same measure, then they are not congruent.
75o D. If two angles are not congruent, then they have the same measure.”
453. What do we call the ratio of two numbers (larger number: smaller number) whose ratio
to each other is equal to the ratio of their sum to the larger number? [Note: This is applied in A. y-1 =
Fibonacci sequences]
A. pi B. golden ratio
C. 1.618 D. Euler’s ratio
C. y-1 =
A. x = 8
C. x = 10
B. x = 9
D. x = 11
456. What do we call an angle formed by two chords of the circle with a common endpoint
(the vertex of the angle)?
A. inscribed angle
C. circumscribed angle
Author: Victor A. Tondo Jr., LPT
458. Which lines are not in the same plane and do not intersect but are not
parallel?
A. asymptotes B. tangent lines
C. skew lines D. directrices
459. Two adjacent angles whose distinct sides lie on the same line are called
what?
A. linear pair B. vertical pair
C. alternate D. corresponding
A. 50 tan 37o
C. 50 sin 37o
C. 10√2
472. How many mL of 40% acid must be added to 1000 mL of 10% acid solution to make a 20% acid
solution?
A. 250 B. 500 C. 600 D. 750
A. 216 cm
468. A coin is tossed 60 times. Head appeared 27 times. Find the experimental probability of getting A. 123 cm
heads.
A. B. C. D.
469. A parabola is defined by the equation
5x = -3y2 – 4y + 2. Which of the following is true C. 123√3
about the parabola?
A. It opens to the left.
B. It opens to the right.
C. It opens upward.
D. It opens downward.
475. How many odd 4digit numbers can be formed using the digits 7, 6, 5, 4, 3, 2, and 1 if
repetition is not allowed?
A. 720 B. 480 C. 240 D. 120
476. There are 70 dogs and geese in a farm. If there are a total of 200 legs, how many dogs are
there?
A. 25 B. 30 C. 35 D. 40
This is a free reviewer. All rights reserved.
1000 MMR
477. Find the maximum area of a rectangle if the perimeter is set at 350 cm.
480. Find the measure of each interior angle of a regular 20-sided polygon.
A. 8656.25 cm2
A. 162o
C. 6656.25 cm2
C. 144o
479. Find the length of the diagonal of a cube given each side measures 482. How many ways can 14 people be seated in a Ferris wheel given that each cart can only contain
17 cm. one person?
A. 15! B. 14! C. 13! D. 12!
C. 17√3
A. B. C. D.
Author: Victor A. Tondo Jr., LPT
484. In a gathering of gamers and admins, there are 24 gamers of which 6 are females, and 3 485. What is the remainder when 3x6+ 4x5 – 5x4 + 6x3 + 7x2 – 8x + 3 is divided by
admins of which one is female. If a female is randomly called, what is the probability that she is an (x – 1)?
admin? A.8 B.9 C.10 D.11
486. Seven people have an average weight of 49 kg. A child was added to the group and the average
became 45 kg. How heavy is the child?
A. 15 kg B. 16 kg
C. 17 kg D. 18 kg
487. Six numbers have an average of 71. If 85 is added to the group, what is the new
average?
A. 72 B. 73 C. 74 D. 75
488. In an arithmetic sequence, the 7th term is 25 and the 10th term is 67. What is the common
difference?
A.42 B.21 C.14 D.7
489. Triangle ABC has sides measuring 20 cm, 20 cm, and 29 cm. What kind of triangle is
ABC?
A. acute B. right
C. obtuse D. reflex
490. Find the surface area of a sphere given that the sphere sits perfectly inside a cube whose sides
measure 20 cm each.
A. 400
C. 1200
A. B. C. D.
This is a free reviewer. All rights reserved.
1000 MMR
492. Simplify: eln 2019 x
Author: Victor A. Tondo Jr., LPT
499. Mocha can finish a job in 24 hours, while her sister Tiramisu can do the same job in
only
A. 2019x
20 hours. How long will it take them to finish the job by working
together?
C.
A.
C. 11 hrs
493. If the roots of a quadratic equation are and , which of the following could be
the
quadratic equation?
A. 63x2 + 22x – 21 = 0
B. 63x2 – 22x – 21 = 0
C. 63x2 + 22x + 21 = 0
D. 63x2 – 22x + 21 = 0
500. What conic figure does the equation x2 + y2 + 10x – 16y = -100
form?
A. Real circle B. Degenerate circle
C. Imaginary circle D. Ellipse
494. If three more than twice a number is seventeen less than seven times the number, what
is the number?
A.2 B.3 C.4 D.5
496. Find the measure of the smaller angle formed by the hands of the clock at
11:20.
A. 130o
C. 140o
497. How many even 3-digit even numbers can be formed using the digits 7, 6, 5, 4, 3, 2, 1, and 0 if
repetition is not allowed?
A. 150 B. 160 C. 170 D. 180
498. There are 50 students in a class. Twenty of them have a laptop. Thirty-two of them
have a smartphone. Seven of them have both a laptop and a smartphone. How many of them
have neither a laptop nor a smartphone?
A.4 B.5 C.6 D.7
This is a free reviewer. All rights reserved.
1000 MMR
501. Victor, Praetor, and Rowena volunteered to
Victor work?
A. 14 B. 16
A. (x + 2019)2 (x – 2019)2
B. (x – 2019)4
D. (x – 4)(x + 4)
A. 706
C. 704
0.25(20x – 2020) = x?
A. 125.25
C. 126.25
A. P1,260,000
C. P630,000
job alone?
A. 12 hours
C. 10 hours
This is a free reviewer. All rights reserved.
A. 18
A. -30√3
C. 14
200?
A. 15x + 2
C. x3 – 4x – 2
B. x2 + 2x + 5
D. x2 + x + 2
524.
500
How many ml of 10% acid must be added to
ml of 40% acid to make a 30% acid solution?
A. 1000 ml
C. 500 ml
516. For which value of k does 9x2 – kx + 25 have only one root?
A. 3.75 B. 7.5 C. 15 D. 30
525. 3log2 3 + 2 log2 5 – log2 7 = ______.
A. log2
517. If A and B are the roots of x2 + 19x + 20, what is AB?
C. log2
A. 20
C. 3√2
518.0.25+0.5+1+2+4+…+1024=____
526. Find the intersection of y = 3x + 4 and y = 5x – 8.
A. 2046.75
A. (8/5, 0)
C. 2048.75
A. 11
A. 40
C. 50
A. 1009.5
522. If 3x = 5 and 2y = 11, what is 12(x – y)? 530. x varies directly as y and inversely as z. If x = 40 when y = 8 and z = 2, what is x when y = 24 and
z = 4?
A. 120 B. 90 C. 60 D. 30
A. -46
532. When a number is increased by 5, its square also increases by 255. What is this number?
A. 22 B. 23 C. 24 D. 25 538. Find the remainder when the polynomial x4 – 5x3 + 7x2 – 11x + 19 is divided by (x – 2).
A.5 B.3 C.1 D.-1
C. x = 1
535. When a number is increased by 3, its square increases by 135. By what does its square increase when the
number is increased by 5?
A. 225 B. 235 C. 445 D. 485
536. Ten cans of soda and six hamburgers cost a total of P440. Five cans of soda and
seven hamburgers cost a total of P360. How much is a hamburger?
A. P31 B. P33 C. P35 D. P37
Author: Victor A. Tondo Jr., LPT
540. Rowena can do a job in 15 hours, while
Victor can do the same job in 25 hours. How long will it take them to finish the job by working together?
A. 9.375 hours B. 9.25 hours 543. The product of two consecutive odd counting numbers is 1023. What is their sum?
C. 9.125 hours D. 8.875 hours A. 60 B. 64 C. 68 D. 72
541. Armel is twice as heavy as Kuku. Gabbi is 17kg heavier than Kuku. The sum of their masses is 217kg. 544. There are 250 pigs and chickens in a farm, all of which are healthy. If there are 720 legs in total, how
How heavy is Kuku in kg? many pigs are there?
A. 110 B. 100 C. 90 D. 80
A.
547. Factorize (x4 – 16) completely. A. (x – 2)4
B. (x – 2)2 (x + 2)2 C. (x + 2)2 (x – 2)2
D. (x + 2) (x – 2) (x2 + 4)
557. 2+9+16+23+…+135=____
1000 MMR A. 1353
C. 1370
548. √125
C. 2√5
559. The area of a rectangle is (x2 – x – 30). If its length is x + 5, what is its width?
A. x + 4 B. x – 3
C. x + 2 D. x – 6
549. The product of two consecutive even counting numbers is 3968. Find the smaller number.
A. 42 B. 46 C. 54 D. 62
560. Factorize 81x2 – 225y2 completely. A. (9x + 15y)(9x – 15y)
B. 9(9x2 – 25y2) C. 9(3x – 5y)2
D. 9(3x + 5y)(3x – 5y)
550. A pipe can fill a pool in 5 hours while another pipe can drain empty the pool in 10
hours. How long will it take to fill the pool if both pipes are open?
A. 8 hours B. 9.25 hours
C. 9.5 hours D. 10 hours
561. What are the missing terms in the series 3, 6, 12, 24, ___, 96, ____?
A. 48; 192
C. 46, 196
551. Today, Rayon is 13 years old while his father is thrice his age. How many years from now will his
father be twice as old as he?
A. 15 B. 13 C. 11 D. 10
A. 18
C. 128 cm
553. Insert one term between 81 and 169 to make a geometric sequence. 563. Which fraction is equivalent to 0.425?
A. 113 B. 117 C. 121 D. 125
A. 21/50
C. 27/40
554. Find the common difference of an arithmetic sequence whose 21 st term is 1987 and 29th term is 2019.
A. 2.5 B. 3 C. 3.5 D. 4
555.
7, 16, 25, 34, 43, 52, 61, …
Find the general term An of the sequence
A. An = 9n – 2 B. An = 8n – 1
C. An = 9n + 2 D. An = 8n + 1
567. Mr. Yu is four times as old as his daughter. Ten years from now, he will only be thrice as old as her. How
old is his daughter now?
A. 10 B. 15 C. 20 D. 25
A.
C. 568. The speed of the current of a river is 9 kph. Rayon rows his boat upstream for 8 hours, and then
travels back to his original position by rowing downstream for 4 hours. What is Rayon’s speed on calm
water?
A. 9 kph B. 18 kph
C. 27 kph D. 36 kph
565. Seven more than four times a number is 79. What is five less than three times the number?
A. 59 B. 49 C. 39 D. 29
569. Thrice Rayon’s age 16 years ago is equal to his age 16 years from now. How old is Rayon now?
A. 16 B. 24 C. 32 D. 40
566. Mr. Park is five times as old as his son. Eight years from now, he will only be thrice as old as his son. How
old is Mr. Park now?
A. 30 B. 35 C. 40 D. 45
570. Divide 8x5 – 4x3 + 2x2 – 3x + 4 by x + 2. A. 8x4 – 16x3 + 28x2 – 54x + 105 r. -206
B. 8x4 + 16x3 + 28x2 – 54x + 105 r. -206 C. 8x4 – 16x3 + 28x2 – 54x – 105 r. 206 D. 8x4 + 16x3
+ 28x2 – 54x – 105 r. 206
Author: Victor A. Tondo Jr., LPT
571. Factorize 27x3 + 125. A. (3x + 5)(9x2 + 15x + 25)
B. (3x + 5)(9x2 – 15x + 25) C. (3x – 5)(9x2 + 15x + 25) D. (3x
– 5)(9x2 – 15x + 25)
572. What is the 5th term of (2x + 3)7? 575. What is the 4th term of (3x – 2)5?
A. 22680x3
A. 360x2
C. 21600x3
C. -240x2
A. 36
A. 10x15y20z10
580. Simplify: (2x3y4z2)5
B. 25x15y20z10
C. 32x15y20z10 D. 100x15y20z10
589. A farmer has 15 goats, 23 pigs, and a few chickens in his farm. If he counted a total
of 200 legs in his farm (excluding his, of course), how many chickens does he
have?
581. Find k such that 5x2 + 30x + k has two unequal real roots. A. 96
A. k > 6 B. k < 6
C. k > 45 D. k < 45
A. π r2 + 2 π r
590. Rationalize
C. √2
A. 0.193193193193…
A.
C. √1023
C.
A. 78
A. 343
587. Factorize 9x2 + 24xy + 16y2 – 81z2. A. (3x + 4y + 9z) (3x + 4y – 9z)
B. (3x – 4y + 9z) (3x + 4y – 9z) C. (3x + 4y + 9z) (3x – 4y – 9z) D. (3x – 4y +
9z) (3x – 4y – 9z)
594. Rayon invested P1,600,000.00 in a bank that offers 2.5% interest per annum. How much will his
account hold after 4 years?
A. P160,000
C. P1,460,000
598. Solve for x: 272x–8 = 9x+10 607. Find the surface area of a rectangular box whose dimensions are 25 cm x 35 cm x 45 cm.
A. 7000 cm2
A. 9
C. 7150 cm2
A. 17.5
C.
parallelogram. If m
A. 40o
601. Which of the following is ALWAYS true? A. Vertical pairs of angles are supplementary. B.
Vertical pairs of angles are congruent.
C. Linear pairs of angles are congruent.
D. Linear pairs of angles are complementary.
C. 412
A. 40o
603. The vertex angle of an isosceles triangle is 70°. What is the measure of one of the base angles?
A. 45° B. 50° C. 55° D. 60° 611. Two parallel lines are cut by a transversal, forming R and V. If the two angles are corresponding angles, what is
the measure of R
if the measure of
604. A shoebox measures 20 inches by 15 inches by 9 inches. What is its volume in cubic inches? A. 35o
A. 900 C. 160o
C. 1650
612. If the sum of the supplement and the complement of an angle is 100 degrees, what is the angle?
A. 65o B. 70o C. 75o D. 85o
A. 160o
C. asymptote
A. 50o B. 130o
it have?
A. 36 B. 24
dodecahedron have?
A. 45 B. 54
is 480 cm.
area?
A. 17 π cm2 B. 34 π cm2
A. Median B. Altitude
C. Bisector D. Longitude
A. Centroid B. Circumcenter
C. Incenter D. Orthocenter
629. The radius of a cylinder measures 9 cm and its height is 15 cm. Find its volume.
A. 1215 π cm3
C. 540 π cm3
of a regular decagon?
A. 108o B. 72o
A. 250 cm2
C. 300 cm2
to an acute triangle?
A. 5 cm, 7 cm, 9 cm
C. 6 cm, 9 cm, 12 cm
A. 24 cm2
C. 56 cm2
1000 MMR
641. Find the equation of the line perpendicular to 9x –7y = -6, passing through (4, -3).
A. 7x + 9y = 1 B. 7x + 9y = -1
C. 7x – 9y = 1 D. 7x – 9y = -1 642. A rectangle is drawn with dimensions 28 cm by 42 cm. A larger rectangle is drawn by
adding
5 cm margins from each side of the original rectangle. What is the area of the larger rectangle?
Author: Victor A. Tondo Jr., LPT
646. In the figure below, m CAB = 2x + 4, while m COB = 5x – 7. Find x.
A. 1456 cm2
C. 1556 cm2
643. Find the length of the diagonals of a rectangular prism 6 cm thick, 15 cm long, and 10 cm wide.
A. 17√3
C. 19 cm
A.
644. Find the volume of the following pool.
648. Find the sum of all interior angles of a regular 15-sided polygon.
A. 5400o
645. What is the slope of the line defined by the equation 3x + 5y = 11?
A. B. C.3 D.5 C. 2340o
649. If each square in the figure has an area of 36 square cm, find the perimeter of the
figure.
A. 864 cm
C. 144 cm
653. Find the center of a circle given one of its longest chords has endpoints at M(-11,8) and N(23,-
24).
A. (12, -16)
C. (-17,16)
A. 9720 π cm2 654. Find the radius of a circle if its diameter has endpoints at M(3,5) and N(-12,13).
A. 8 B. 8.5 C. 9 D. 9.5
C. 540 π cm2
655. Find the equation of the line passing through (7, 5) and (5, 1).
A. y = 2x + 9 B. y = 2x – 9
̂ ̂ C. y = ½ x + 9 D. y = ½ x – 9
651. In the figure, = 120o and m E = 28o. Find .
C. 51,000 π
Author: Victor A. Tondo Jr., LPT
656. Find the equation of the line passing through (2, -7) with a slope of -3.
A. y = -3x – 1 B. y = -3x + 1
C. y = -3x – 2 C. y = -3x + 2
C. D.
C. D.
663. X(7,9) is the midpoint of A(1,2) and B. Find the coordinates of B. A. 7,500 π
A (-13, -16)
C. 30,000 π
C. (-13, 16)
672. A right triangle is inscribed in a circle. If the triangle’s legs are 16 cm and 30 cm, what is the area of the
circle?
664. Point O is two-thirds the way from A(-4,5) to B(8,-10). Find the coordinates of point O.
A. 289 π cm2
A. (0, 0)
C. 480 π cm2
C. (3, -6)
A. 13
674. What is the area of the red trapezium in the given figure?
No item #666, as requested by many subscribers.
667.
A. 23 sq. units
C. 25 sq. units
B. 24 sq. units
D. 26 sq. units
4x + 20 and m G = 6x – 10, find x.
A. 13
668. Point C is two-fifths the way from M(-3,4) to N(7,-11). Find the coordinates of point
C.
A. (0, 0)
C. (1, -2)
25
676. A rectangle is 24 cm long. If its diagonal is
cm, what is its perimeter in cm?
Author: Victor A. Tondo Jr., LPT 682. Which of the following is false?
A. Radii of the same circle are always congruent.
B. Any two right angles are congruent.
A. 49 B. 62 C. 64 D. 98
C. An inscribed right angle always intercepts a semicircle.
D. There are infinitely many lines that may be drawn parallel to a given line passing through a given point.
683. The volume of a cube is 343 cm3. What is its surface area?
A. 24 cm2 B. 24√3 cm2
677. Square FACE is drawn such that one of its sides AC is the diagonal of a rectangle, ABCD. If AB = 20
and BC = 43, find the area of FACE.
A. 2209 cm2 C. 48 √13
C. 2249 cm2
684. Find the volume of a rectangular plank of wood that is half-inch thick, six inches wide, and
6 feet long.
A. 18 cubic inches B. 36 cubic inches C. 108 cubic inches D. 216 cubic inches
678. If the vertex angle of an isosceles triangle is 50 o, what is the measure of each base angle?
A. 130o B. 80o C. 65o D. 25o
685. In the following figure, O is the center of the circle. If m CAB = 36o, what is m DBA?
679. Which of the following is a square? A. VCTR whose sides measure 20 cm each B. LUVS whose angles
are 90o each
obtuse triangle?
A. 20, 20, 29
C. 20, 22, 28
A. 4 π cm
C. 8 π cm
B. “If two angles not vertical angles, then they are not congruent.”
C. “If they are not congruent, then two angles are not vertical angles.”
D. “If two angles are linear angles, then they are supplementary.”
694. Each side of a regular hexagon is 256 cm long. Find the distance around it.
A. 1536 cm
689. Give the inverse of the conditional statement “If two angles are vertical angles, then they are congruent.”
C. 1024 cm
A. “If they are are congruent, then two angles are vertical angles.”
B. “If two angles not vertical angles, then they are not congruent.”
C. “If they are not congruent, then two angles are not vertical angles.”
D. “If two angles are linear angles, then they are supplementary.”
695. What can be said about these statements? i. Any rectangle has two congruent
diagonals. ii. Any rhombus has two perpendicular diagonals.
A. Only the first statement is true. B. Only the second statement is true. C. Both
statements are true.
D. Both statements are false.
690. In which geometry do two distinct lines intersect in two points? 696. In the figure, A is the midpoint of and
A. Euclidean Geometry B. Hyperbolic Geometry . By which triangle congruence postulate can we prove that △CAB △DAE?
C. Elliptic Geometry D. Plane Geometry
691. In which geometry can the sum of the measures of the angles of a triangle be less than
180 degrees?
A. Euclidean Geometry B. Hyperbolic Geometry
C. Elliptic Geometry D. Plane Geometry 697. Find the circumference of a coin whose radius is 13 cm.
A. 13 π cm B. 26 π cm
C. 169 π cm D. 676 π cm
692. In which geometry are the summit angles of a Saccheri quadrilateral obtuse angles?
A. Euclidean Geometry B. Hyperbolic Geometry
C. Elliptic Geometry D. Plane Geometry
This is a free reviewer. All rights reserved.
C. 19600cm2 C. 240 cm
50
708. cm long, how long is a leg?
If the hypotenuse of a 45-45-90 triangle is
699. Find the altitude to the hypotenuse of a right triangle whose legs are 60 cm and 80 cm.
A. 40 cm B. 48 cm A. 25 cm
C. 96 cm D. 100 cm
C. 25√3
700. This is formed by two rays with a common end point. 709. Which of the ff. is coterminal with 143o?
A. polygon A. 2403o
C. line C. 2103o
710. What is the measure of the smaller angle formed by the hands of the clock at 5:55?
701. Convert A. 180o
C. 163.625o
A. 210o
C. 630o
711. What is the measure of the smaller angle formed by the hands of the clock at 3:50?
A. 185o
A. 32o
712. Which of the following is false?
A. tan θ = B. csc θ
= C. sec θ = D. sin
θ=
703. A negative sine and a positive cosine are properties of angles in which quadrant?
A. QI B. QII C. QIII D. QIV 714. If sin x = 0.936, which of the following could be cos x?
704. Which of the following equal to sec 50o?
A. sin 40o B. cos 40o
C. csc 40o D. cot 40o A. 0.064
A. 219o
C. 199o
A. 25 m
C. 50 m
C. 40 m
A.
716. If csc x =
721. What is the reference angle of 156o?
A. B. C. D.
A. 14o
A. 20 meters
C. 20 √3
725. A meter stick leans to a wall and reaches a point 50 cm away from the wall.
What is the measure of the angle formed by the meter stick and the wall?
A. 30o B. 45o C. 60o D. 75o
726. A tight rope connects the top of a 500 cm pole to the ground. If the angle formed
by the ground and the tight rope is 45o, how long is the tight rope?
A. 500 cm
C. 500√3
C. 2
C. 0
A. x | x
C. x 2
A. 3
C. x > 0
A. LDNE
A. +
C. y = 2020 A. y | y
C. y
741. Give the range of f(x) = 745. Give the range of f(x) = x2019.
A. y | y B. y 0
C. y 2019 D. y = { }
A. y | y
C. y = 2019
C. y 2
C. y = 20 D. y = { }
743. Give the range of f(x) = x125. 747. Give the range of f(x) =
A. y | y
C. y
A. y | y
C. y
A. x | x
C. x > -81
This is a free reviewer. All rights reserved.
C. y 2019 759. In Mathematics, what does the delta ∆ symbol mean or refer to?
751. Find the first derivative: f(x) = (2x + 3)5.
A. the change in B. the summation of
C. therefore D. belonging to
760. In Mathematics, what does the sigma symbol mean or refer to?
A. the change in B. the summation of
C. therefore D. belonging to
752. Find f ‘(x) given f(x) = (3x2 – 2x + 1 )7. A. f ’(x)= (42x – 14) (3x2 – 2x + 1)6
B. f ’(x)= (42x – 28) (3x2 – 2x + 1)6
C. f ’(x)= (42x + 14) (3x2 – 2x + 1)6 D. f ’(x)= (42x + 28) (3x2 – 2x + 1)6
761. In Mathematics, what does the epsilon symbol mean or refer to?
A. such that B. element of
C. therefore D. ergo
753. Find the maximum area that can be enclosed by a rectangle given its perimeter should only
be 202 meters.
A. 2550 m2
762. In set theory, what does the symbol mean or refer to?
A. not O B. not zero
C. does not D. empty
C. 2550.5 m2
755. Find the instantaneous rate of change for f(x) = 5x3 – 2x2 + 7x – 9 at x = 2.
A. 37 B. 48 C. 59 D. 70
C. y = x + 2
756. Find the instantaneous rate of change for f(x) = 4x3 + 5x2 – 7x – 11 at x = 1.
A. -15 B. 0 C. 15 D. 30 765. In set theory, what does the symbol mean or refer to?
A. no elements B. common elements
C. all elements D. half of the elements
757. Find the slope of the line tangent to y = 3x 5 – 4x3 + 5x2 – 10x + 9 at x = 2.
A. 50.5 B. 101 C. 151.5 D. 202
mean?
A. since
C. therefore
A. B.
find X Y.
A. {1, 4}
C. {2, 3}
X–Y.
A. {1, 4, 8}
C. {3, 8}
Y–X.
A. {1, 4, 8}
C. {3, 8}
X Y.
A. {3, 4}
C. {1, 2}
following
represents the
shaded region in
A. A B
C.A–B
1000 MMR
781. Using the digits 0, 1, 2, 3, 4, 5, 6, and 7 without repetition, how many 4-digit numbers can be made?
A. 1680 B. 1470 C. 840 D. 735
A. 24
783. Using the digits 0, 1, 2, 3, 4, 5, and 6 without repetition, how many odd 3-digit numbers can be made?
A. 180 B. 150 C. 90 D. 75
790. How many ways can the word ABABA be rearranged?
A. 120 B. 60 C. 20 D. 10
784. Using the digits 0, 1, 2, 3, 4, 5, 6, and 7 without repetition, how many odd 4-digit numbers
can be made?
A. 1470 B. 1440 C. 735 D. 720
791. There are 10 different beads and a locking mechanism to be used in making a bracelet. How many
different bracelet patterns can be made?
A. 3,628,800
785. Using the digits 0, 1, 2, 3, 4, 5, 6, and 7 without repetition, how many odd 3-digit numbers
can be made? C. 1,814,400
A. 336 B. 288 C. 168 D. 144
786. Using the digits 0, 1, 2, 3, 4, 5, 6, and 7 without repetition, how many even 3-digit numbers 792. There are six different Math books, four different Science books, and two
can be made? different English books to be arranged on a shelf. How many ways can this be done?
A. 162 B. 156 C. 150 D. 144
A. 12C3 B. 12! C. 48 D. 3!
787. Using the digits 0, 1, 2, 3, 4, 5, 6, and 7 without repetition, how many even 4-digit numbers
can be made? 793. There are six different Math books, four different Science books, and two different English
A. 765 B. 750 C. 735 D. 720 books to be arranged on a shelf. If books of the same subject must be together, how many ways can this be
done?
A. 479,001,600
C. 34560
788. Using the digits 0, 1, 2, 3, 4, 5, and 6 without repetition, how many even 3-digit numbers can be made?
A. 120 B. 105 C. 90 D. 75
794. There are six people to be seated on a bench for a picture. A certain couple,
Vic and Rowena, are to be seated next to each other. How many ways can this
be done?
A. 120 B. 240 C. 360 D. 720
795. There are six people to be seated on a bench for a picture. A certain trio, Vic,
Rayon, and Aira, do not want to be separated. How many ways can this be
done?
A. 60 B. 72 C. 144 D. 180
799. If three-fourths of a number is 33 more than one-fifth of itself, then what is the number?
A. 80 B. 60 C. 40 D. 20
801. In a certain fastfood chain, soft drinks are served in Small, Medium, and Large cups. What level of
data is this?
A. Nominal B. Ordinal
C. Interval D. Ratio
802. The prime factorization of a number is given as 23 x 52 x 133. How many factors does it have?
A. 96 B. 48 C. 36 D. 18
804.
Author: Victor A. Tondo Jr., LPT
There are ten red, nine blue, and six black pens in a bag. If a pen is randomly drawn from the bag, what C. √3
A. (3x + 4)6+ c
805. There are ten red, nine blue, and six black pens in a bag. If a pen is randomly drawn from the bag, what
is the probability that it is black?
A. 0.06 B. 0.6 C. 0.24 D. 0.7
C. 5(3x + 4)4+ c
A. √2
C. 2x3 + 3x2 – 6x + c
C. √3
811.
A. +c
808. Simplify: √
C.
A. √6
812. ∫ dx
A. +c
C. +c
A. bar graph
C. pie graph
A. bar graph
C. pie graph
A. cluster
C. stratified
14 15
25 26
31 33
A. 31 B. 31.4
834.
Author: Victor A. Tondo Jr., LPT
Fifteen guests shake hands with each other. If each guest is to shake hands with all the other guests, how
many handshakes will be made?
A. 105 B. 150 C. 210 D. 225
1000 MMR
835. The salary of 5 men for 6 days is P9,000. How much is the salary of 7 men for 8 days?
A. P15,000
826. Given BF bis ects
C. P16,800
find m
A. 17o
836. The average grade of eleven students is 84. If the average of seven of these students is 82, what is the
average of the other four students?
A. 88 B. 87.5 C. 86.5 D. 86
A. 20√2
828. How many 3-digit numbers can be formed using the digits 0, 1, 2, 3, 4, 5, and 6 if repetition is not
allowed?
A. 160 B. 180 C. 200 D. 210
̅̅ ̅̅ ̅̅
A. VC B. AC C. VA D. CA
830. Which quadrilateral has two congruent diagonals that are perpendicular to each other? 838. Find the largest area of a rectangular piece of land that can be enclosed with 800 meters of fencing
A. kite
C. rectangle
B. isosceles trapezoid
D. rhombus
material.
831. If the length of a rectangle is increased by A. 40,000 m2
30% while the width is decreased by 30%, what will happen to its area?
A. It stays the same.
B. It is increased by 9%. C. It is decreased by 9%. D. It is
decreased by 6%. C. 62,500 m2
832. Rayon had an average of 93 on his first five Math tests. After taking the next test, his average increased to 839. Which statistical test is used for testing for relationship between two variables?
94. Find his most recent grade.
A. 96 B. 97 C. 98 D. 99
A. ANOVA
C. Pearson R
833. A bus drove for 7 hours at 73 kph and 3 hours at 88 kph. What was its average speed?
A. 75.5 kph B. 76.5 kph
C. 77.5 kph D. 78.5 kph
840. Given
find lim
A. 5
C. 16
841. Find the equation of the line passing through (3, 7) and (-3, -5).
A. y = 2x + 1 B. y = 3x + 1
C. y = 2x – 1 D. y = 3x – 1
C. 57
843. If A is at (-9, 11) and B is at (6,-9), find C if C is three-fifths the way from A to B.
A. (1, -1)
850. An item is sold for P7,280 after being marked down by 30%. What was its original price?
C. (1, 1)
C. P10,400
851. Which of the following lines passes through the point (-4, 5)?
A. y = x + 1 B. y = 2x – 3
C. y = 1 – x D. y = 3 – 2x
C.(, ) D.(, )
845. Mr. G sold 70% of his chickens and
still had 852. Which of the following is outside the circle defined by the equation (x + 2) 2 + y2 = 50?
129 chickens left. How many chickens did he have originally? A. (-9, 1)
A. 344
C. (-5, 7)
C. 598
C.
848. What do you call an equation in which only integer solutions are allowed?
A. Diophantine equations B. Euclidean equations
C. Fibonacci equations D. Newtonian equations 854. Which of the following equations pertain to a parabola that opens to the left?
A. -2(y + 7) = (x – 3)2 B. 5(y – 9) = (x – 5)2
C. (y – 4)2 = 2(x + 7) D. (y + 3)2 = -3(x + 4)
855. The lengths of two sides of a triangle are 7 inches and 15 inches. Which of the
849. The sum of two numbers is 73 and their difference is 41. What is the larger number? following represents x, the possible length in inches of the remaining side of the triangle?
A. 8 < x < 22
C. x < 8 or x > 22
1000
856.
MMR
Rayon has 46 coins in P1 and P5 denominations for a total worth of P186. How many P5 coins
Author: Victor A. Tondo Jr., LPT
865. Which of the following Mathematicians had a method “ Sieve” for identifying prime
does he have?
numbers?
A. Pythagoras B. Euclid
C. Eratosthenes D. Archimedes
A. 25
866. Which Mathematician developed a formula for finding the area of a triangle using its side lengths?
857. If x = 2 and y = 3, what is x2 – y2 + 3xy? A. Hipparchus
C. Heron
B. Pythagoras
D. Euclid
A. 31
A. √
C. √
869. Prime numbers that are 1 less than a power of 2 are called ______.
A. Pythagorean Primes B. Mersenne Primes
C. Fermat Primes D. Pascal Primes
861. ln e4 = _____
870. Which Mathematician developed the triangle of binomial coefficients?
A. Gottfried Leibniz B. Isaac Newton
C. Blaise Pascal D. Johann Bernoulli
A. 24.1295
C. 6.032375
871. Two Mathematicians are known for the theory of hyperbolic geometry although
they worked independently from each other. Who are these Mathematicians?
A. Euclid and Pythagoras B. Euclid and Descartes C.
Lobachevsky and Bolyai D. Babbage and Bolyai
862. eln 5 = _____
A. 5
863. What is ln e?
A. 0
C. Chinese
879. Convert
873. Which Mathematician is known for his last theorem?
A. Andrew Wiles B. Pierre de Fermat C. John
Wallis A. 70o
D. Rene Descartes
C. 210o
874. Who have the first fully-developed base-10 number system in use?
A. Romans B. Egyptians
C. Indians D. Sumerians
880. Find the volume of a cylinder whose radius is 50 cm and height is 24 cm.
A. 20,000 π cm3
875. Notched tally bones proved their use of Mathematics around 35000 BCE. C. 45,000 π cm3
A. Africans B. Asians
C. Europeans D. Americans
876. Which sampling method is best used when the population has subgroups? 881. Find the altitude to the hypotenuse of a right triangle whose legs measure 16 cm and 30 cm.
A. systematic sampling B. stratified sampling C. quota
sampling
D. cluster sampling
C. x 3
25
882.non-collinear points?
How many line segments can be made from
A. 900 B. 600 C. 450 D. 300
C. 1152 π cm2
888. Two triangles have a pair of congruent angles. Which of the following is true about these triangles? A. (-2019)2
B. –(2019 x 2019)
893. The product of two numbers is 1425. If each of the two numbers is doubled, the product of these larger
numbers is _____.
889. Rayon deposited an amount of P500,000 in a bank that offers 4% interest
compounded per annum. How much will he have in his account after 3 years? A. 2,850
A. P720,000 C. 8,550
C. P560,000
A. 7
A. 34
Author: Victor A. Tondo Jr., LPT
894. The amount of money you have falls under what level of data?
A. nominal B. ordinal
C. interval D. ratio
898. How long is the latus rectum of the parabola defined by (y + 3) 2 = 20x –
196?
A.3 B.12 C.20 D.19
895. To study tooth decay a researcher takes a sample at random but with the stipulation that all
age groups are represented proportionally. What sampling method did the researcher use?
A. systematic B. cluster
C. stratified D. convenience
899. In the set of real numbers, what do we call the set that includes only the counting numbers and zero?
A. Integral equations
B. Differential equations C. Diophantine equations
D. Bolyai-Lobachevsky equations
909. Find the instantaneous rate of change for f(x) = 4x3 + 7x2 – 27x – 49 at x = 1.
902. The top ten students of a graduating class got the following scores in their final examination in
Calculus:
A. -2
903. The top ten students of a graduating class got the following scores in their final examination in
Calculus:
911. After using one-sixth of her budget on bills, two-fifths on groceries, and P1700
on books and magazines, Mrs. Lazada still had P7400 left. How much was her budget?
A. P20,000
C. P22,500
What is the modal score?
A. 83
912. An assistant’s response times were recorded on the table below. What is her average response time in
minutes?
904. The sum of five consecutive integers is 980. What is the value of the greatest integer?
A. 192 B. 194 C. 196 D. 198
A. 10
A. 3.625
C. (5x + 4)(3x – 5)
907. Which of the following is the axis of symmetry of the parabola defined by the 913. The average grade of 22 students in Section Abaca is 95, while the average grade of 28
equation y = x2 – 6x + 5? students in Section Acacia is 89. What is the average grade of all 50 students in both sections?
A. x = 5 B. x = -5 A. 92.36 B. 91.64 C. 90.72 D. 89.8
C. x = 3 D. x = -3
C. P700
917. If the sum of the supplement and the complement of an angle is 146, what is the angle?
C. (5, -13) A. 61 B. 62 C. 63 D. 64
916. A book was sold for P630 after a 10% discount was given. How much was the book originally? 918. A bag contains some marbles. When the marbles are grouped by 2, 3, 4, 5, or 6, there is
always one marble left. Which of the following could be the number of marbles in the bag?
A. 31 B. 41 C. 51 D. 61
Author: Victor A. Tondo Jr., LPT
922. If x = 10, which of the following is equal to
219?
A. 23x – 1 B. 2x2 + 5x + 5
919. Amitaf, Bernard, and Chloe share a total of P2,460 in the ratio 3:5:4 respectively. How much is Amitaf’s C. 2x2 + 2x – 1 D. x2 + x + 2
share?
A. P600
C. P630
923. In the equation 3x – 4y = 25, what is x when y is 5?
A.18 B.15 C.12 D.9
920. Fifty-four kilometers per hour is equal to how many meters per second?
924. What is the measure of each interior angle of a regular 30-sided polygon?
A. 15
A. 158o
C. 165o
921. Simplify
925. Simplify:
A. 1
A. csc x – tan x
C. csc x – cot x
926. Simplify:
A.
A.[ 2
931. When dealing with matrices A, B, and C, which of the following is always
true?
A.AB=BA
B. If AB = 0, then A = 0 or B = 0 C. If AB = AC, then B =
C
D. None of the above.
932. If the numbers x-1, x+2 and 2x+4 are consecutive terms of a geometric sequence, what is x?
A.1 B.2 C.4 D.8
933. What are the zeros of 12x2 – x – 35?
A.
A.
D.
B.
C.
C. 1980o
C. 20(45x – 20)
B.
940. If
C. 13
C.
941. If
A. 3
C. 7
D.
942. Give the domain of f(x) = √
A. x | x
938. Given f(x) = (x + 2)(x – 1)(2x – 3)(x – 5), where can we find f(3)?
A. On the x-axis.
B. Above the x-axis. C. Below the x-axis. C. x > 24
D. On the intersection of the x- and y-axis.
C. 10A2B3
A. 1
947. Find the length of the intercepted arc of a central angle measuring 45 o given the radius is
A. 20 π cm
80 cm.
B. 30 π cm 955. Factorize: x3 + 5x2 – 9x – 45 A. (x + 1)(x + 9)(x – 5)
C. 40 π cm D. 50 π cm B. (x + 1)(x – 9)(x + 5) C. (x + 3)(x + 3)(x – 5) D. (x +
3)(x – 3)(x + 5)
956. Which of these is equal to A-1 + B-1?
A.( ) B.+
C. D.
948. If f(x) = 5x2 – 4x + 9, find the average rate of change from x = 1 to x = 4.
A. 20 B. 21 C. 22 D. 23
957. Find the equation of the line with slope 3, passing through (2, 5).
A. y = 3x – 2 B. y = 3x – 1
949. If f(x) = 4x2 + x – 5, find the instantaneous rate of change at x = 5. C. y = 3x + 1 D. y = 3x + 2
A. 39 B. 40 C. 41 D. 42
958. Find the distance of the point (7, 8) from the line 2x – 3y + 4 = 0
C. 67.5 x 104
A.
24
A. 24
959. Find the distance between the parallel lines y = 2x – 1 and y = 2x + 4.
A.5 B.2√5 C.√5 D.½
952. Rayon’s Social Security System (SSS) ID number is 02-2525255-2. What level of data is the SSS ID?
960. There are 80 cows and ducks in a farm, all of which are healthy. If there are 190 legs in total, how
A. nominal B. ordinal many cows are there?
C. interval D. ratio A.5 B.10 C.15 D.20
953. A tree is 4 meters tall, while the flagpole is 8 meters tall. What level of data is used?
A. nominal B. ordinal
C. interval D. ratio
A. sin x =
968. Find k such that (x + 2) is a factor of the polynomial 5x 3 + 11x2 – kx +
12.
A.11 B.7 C.-3 D.-8
A.A=
A. 105o
964. V and
and m R = 5x + 72, find m
A. 112o
A. LDNE
970.
Author: Victor A. Tondo Jr., LPT
The distance D of a projectile from the ground
t seconds
after launch is given 973. Find the slope of the line tangent to the graph of f(x) = x3 – 3x2 + 5x – 1 at x = 1.
A.-1 B.0 C.1 D.2
D = 8t – t2. How
974. Which of the following is a diagonal matrix?
many seconds
the ground?
A. 4
972. The distance D of a projectile (in meters) from the ground t seconds after launch is given as D = 16t –
2t2. What is its maximum height?
A. 16 m
C. 32 m
984. The numbers x, y, z, and w have an average of 30. If x, y and z have an average 35, what is w?
A.5 B.10 C.15 D.20
985. A is a constant. Find A such that the equation 2x + 1 = 2A + 3(x + A) has a solution at x =
2.
A. -0.4 B. -0.2 C. 0 D. 0.2
977. Find the quotient when the polynomial 7x5 – 3x3 + 2x2 – 9x + 3 is divided by x – 1. A. 7x4 +
7x3 + 4x2 + 6x – 3
B. 7x4 + 7x3 + 4x2 – 6x – 3 C. 7x 4 + 7x3 – 4x2 – 6x – 3 D. 7x 4 986. It takes 4 men 9 days to build 2 houses. How many days will it take 6 men to build 5
– 7x3 – 4x2 – 6x – 3
houses?
A. 12
978. Find k such that 9x2 + 36x + k = 0 has only one unique root.
A.4 B.16 C.18 D.36
988. In a certain university, a student’s grade is computed as the sum of 25% of his prelims grade,
980. A certain bacteria doubles its population after 3 minutes. If the bacteria in a 30% of his midterms grade, and 45% of his finals grade. He knows that his prelims grade is 72 and his
petri dish is 512,000 at 8:45 AM, at what time was its population count 125? midterms grade is 70. What is his grade for the finals if he got a grade of 75 in Calculus?
A. 6:30 AM B. 7:15 AM A. 75 B. 77 C. 79 D. 80
C. 8:09 AM D. 9:21 AM
989. Mr. Lazada bought 75 pieces of Elunium for 8,000 per piece and sold them for a total of 720,000. What
981. A square picture was framed and given 3 cm margins. If the total area of the margin is 324 cm 2, what is his mark-up rate?
is the area of the picture?
A. 20%
A. 441 cm2
C. 32.5%
C. 729 cm2
990. Triangular numbers are numbers that can be shown by triangular arrangements of dots.
982. Given f(x) = x2 + 10x + 25 and g(x) = 3x + 1, find f(g(x)). The triangular numbers are 1, 3, 6, 10, 15, 21, …
What is the 20th triangular number?
A. 200 B. 205 C. 210 D. 220
A. 3x2 + 30x + 26
C. 9x2 + 30x + 26
A. 88.145o
C. 97.593o
992. Find the equation of the line perpendicular to 5x + 3y = 12, passing through (3, -7).
A. 3x – 5y = -44 B. 3x + 5y = 44
C. 3x – 5y = 44 D. 3x + 5y = -44
995. The first term of an arithmetic sequence is 3 and the 20 th term is 98. What is the 25th term?
A. 108 B. 115 C. 118 D. 123
993. Point V is two-fifths the way from A(9, -7) to B(-6, 13). Find the coordinates of point V.
A. (2, 2)
C. (3, 1) 996. An educational psychologist classifies students as high, medium and low intelligence. What kind of
scale is being used?
A. nominal scale B. ordinal scale C. interval scale D. ratio scale
Author: Victor A. Tondo Jr., LPT
999. A certain Math challenge gives the competitors a score of 4 for each correct
answer, and a deduction of 1 point for each wrong answer. If a contestant
answered all 100 items and got a score of 200, how many items did the
997. The GCF of two numbers is 8 and their LCM is 80. What is their product? contestant answer correctly?
A. 80
A. 45
C. 217
End of 1000MMR.
Congratulations!!
Solutions
and
Explanations
non-collinear points?
A. 900
C. 450
Solution:
30C2 = 435; or
....... .....................
A. 12.4
C. 16.1
Solution:
................................................
A. (x + 4)(x + 2)
B. (x – 4)2
C. x(x + 2x + 2)
D. (x – 4)(x + 4)
Explanation:
................................................
0.4(5x – 1470) = x?
A. 490
C. 1470
Solution:
0.4(5x - 1470) = x
2x – 588 = x
2x – x = 588
x = 588
1000 MMR
Solution:
Author: Victor A. Tondo Jr., LPT
12. The vertex angle of an isosceles triangle is 20°. What is the measure of one of the base angles?
Since the common ratio is 4, then next terms should be 80(4) and 1280(4), or 320 and
A. 150° B. 60° C. 75° D. 80°
5120. ................................................ Solution:
(180-20)/2 = 160/2 = 80
................................................
9. At what rate per annum should P2400 be invested so that it will earn an interest of P800 in
8 years?
13. Ana and Beth do a job together in three hours. Working alone, Ana does the job in 5 hours. How long
will it take Beth to do the job
A.6½% alone?
Solution:
i=PRT C. 3 hours
800 = 2400 x R x 8
0.0416666 = R
800 = 19200 R Solution:
................................................ Time to finish a job by working together =
= 3where A = 5
=3
5B=15+3B
10. The area of a rectangle is (x2 + 2x - 8). If its length is x + 4, what is its width? ................................................
2B=15
B=7.5
A. x + 2
14. How much greater is the sum of the first 100 counting numbers than the sum of the first 50
C. x + 1 counting numbers?
Explanation: A. 110
Just factorize.
................................................
11. What is the value of 12⅙ - 3 ⅜ - 5 ⅔ + 20 ¾?
C. 3,155
Solution:
Solution:
LCD = 24
12
= 12
= 32
= 24
=23
................................................
Solution:
65 = 7,776
................................................ 91(7) – (88+90+97+90+91+86) = N 637 – 542 = 95
................................................
16. A water tank contains 18 liters when it is 20% full. How many liters does it contain when 50% full?
A. 60 B. 30 C. 58 D. 45 20. On a certain day, three computer technicians took turns in manning a 24-hour internet
Solution: shop. The number of hours Cesar, Bert, and Danny were on duty was in the ratio 3:4:5,
18:20 = n : 50 respectively. The shop owner pays them P50 per hour. How much would Danny receive for
18(50) = 20n
900 = 20n
that day?
n = 45
................................................
A. P 230
17. The edges of a rectangular solid have these measures: 1.5 feet by 1½ feet by 3 inches. What is its volume C. P160
in cubic inches?
A. 324 B. 225 C. 972 D. 27
Solution:
Solution:
Let their respective times be 3x, 4x, and 5x for a total of 24 hours.
Convert the side measures from feet to inches before proceeding with multiplication 1.5 ft = 1.5(12) or 18 in 3x + 4x + 5x = 24
................................................
.: Danny works for 10 hours at P50/hr, or P500 for that day.
................................................
18. In a certain school, the ratio of boys to girls is 5 is to 7. If there are 180 boys and girls in the school, how many boys
are there?
21. A retailer buys candies for P90.25. The pack has 35 pieces of candies. If she sells each candy for P3.25,
how much profit does she make?
A. 105
A. P11.50
C. 45
Solution: C. P37.50
Let 5x = boys
7x = girls Solution:
5x + 7x = 180
12x = 180
5x = 5(15) = 75
x = 15
Profit = 35(3.25) – 90.25 = 113.75 – 90.25 Profit = 23.50
................................................ ................................................
22. Determine the midpoint of the line segment joining the points (7, -3) and (-1, 6).
A. (2, 3/2)
C. (3, 3/2)
23. One side of a 45° - 45° - 90° triangle measures x cm. What is the length of its
hypotenuse?
A. x √3 cm
Solution:
C. (x √3)/2 cm
Explanation:
A.P=4S;
In a 45-45-90 triangle, the hypotenuse is √2 times of the leg.
................................................
B. P = 2(L+W)
24. The legs of one right triangle are 9 and 12, while those of another right triangle are 12 and C.P=3S
16. How much longer is the perimeter of the larger triangle than the perimeter of the smaller triangle?
A.84 B.7 C.12 D.14 D.P=L1+L2+H
Solution: ................................................
Solve for the hypotenuse of the two triangles. The first one will have 15, while the other will have 20.
27. How many square inches are in 2 square yards?
Get their respective perimeters. The first triangle has a perimeter of 9+12+15 or 36. The other triangle’s perimeter is A. 900
12+16+20 or 48.
48–36=12
................................................
C. 1296
Solution:
1 yard = 3 feet = 3(12) or 36 inches
25. An online shop sells a certain calculator for P950 and charges P150 for shipping within 1 square yard = 362 or 1296 square inches
.: 2 square yards = 2(1296) = 2592 sq in
Manila, regardless of the number of calculators ordered. Which of the following equations shows ................................................
the total cost (y) of an order as a function of the number of calculators ordered (x)?
A. y = (950 28. In a playground for Kindergarten kids, 18 children are riding tricycles or bicycles.
If there are 43 wheels in all, how many tricycles are there?
C. x = 950y
A. 8
Explanation:
The cost of each calculator is P950, so x calculators cost P950x. Add the constant shipping cost
which is P150 and that’s D.
Solution:
T+B=18
3T+2B=43
................................................
29. Aira takes ¾ hour to dress and get ready for school. It takes 4/5 hour to reach the school. If
her class starts promptly at 8:00 am; what is the latest time she can jump out of bed in
order not to be late for school?
A. 6:42 am
C. 6:57 am
4x – 8 = 36
A. 43/200
C. 21/50 3x = 3(11) = 33
................................................
Explanation:
0.215 is read as 215 thousandths. In fraction form, that’s . In simplest form, . 33. What is the least common multiple of 12, 24 and 72?
Alternative Method:
Just use your calculator. A. 12
................................................
6
32. A man is 3 times as old as his son now. Four years ago, the sum of their ages was 36. Find the man’s age
now.
2
A. 33
2
C. 29
6 x 2 x 2 x 1 x 1 x 3 = 72
................................................
Solution: 34. The hypotenuse of a right triangle is 25 feet. If one leg is 24 feet, what is the length of the other leg?
A. 6 ft. B. 5 ft. C. 20 ft. D. 7 ft.
Explanation:
Use the Pythagorean Theorem.
................................................
First, create a table.
35. If two variables X and Y are directly related, which of these is NOT true?
A. When X is low, Y is also low.
B. As X increases, Y also increases. C. When X increases, Y decreases.
D. A high Y is associated with a high X.
Explanation:
C refers to an inverse or indirect relation.
................................................
A. x
C. x = -1
37. A car travels D km in H hours. Which of the following expressions shows the distance travelled by the
car after M minutes?
A. MD/H
41. √8
C. MD/60H
Solution:
A. 4 √2
38. Find the surface area of a rectangular box whose dimensions are 30 cm x 40 cm x 50 cm.
................................................
C. 9400 cm2
Solution:
SA=2(LW+WH+LH) 42. By which property can we state the following:
SA = 2 (50x40 + 40x30 + 30x50) “If ax + b = c, then ax + b - b = c – b.”
SA = 2 (2000 + 1200 + 1500) = 9400
................................................
39. If x – y = 3, then (y-x)-3 = ___. A. transposition
A. 9 C. additive inverse
Explanation:
C. 1/27
Solution: We added –b to both sides of the equation, thus we used APE (addition property of equality).
................................................
C. (1, -2)
Solution:
Let P be at (x,y). By Midpoint formula:
(-7 + x)/2 = -3
-7 + x = -6
x = -6 + 7
x=1
................................................
44. What is the slope of the line 3x – y = 11?
A. -1/3 B. 1/3 C. -3 D. 3
Isolate y on one side of the equation to rewrite the equation in the form y = mx + b.
3x – y = 11
-y = -3x + 11
47. How much water must be evaporated from 90 ml of a 50% salt solution to increase its concentration to 75%?
y = 3x – 11
................................................
Author: Victor A. Tondo Jr., LPT
45. What is the minimum value of f(x) = 3x2 + 6x + 7? 48. A and
A.1 B.-1 C.4 D.-4
Solution:
A. 40 ml
Min Value = c – b2/4a B. 38 ml
and m B = 5x – 44, what is the value of x?
That’s 7 – 36/12 or 7-3=4
................................................
46. If xy = 23 and x2 + y2 = 75, find x + y. A. 50.5
C. 35 ml D. 30 ml Solution:
A. 10.7845
Solution:
C. 11.2155
Since the two angles form a vertical pair, then they are congruent.
Solution: 3x = 5x – 44
44
Since we are evaporating water, we will be adding a NEGATIVE volume of water (or simply put, we are subtracting water, diba?) ................................................
49. The angle of elevation from an observer to the top of a building is 30 o. If the
building is 50 meters high, how far is the observer from the building?
A. 25 B. 25√3 C. 50√3 D. 100
Solution:
90(50) + (-X)(0) = (90-X)(75)
Use a 30-60-90 triangle. The side opposite of the 30 o angle will represent the building.
................................................
4500 + 0 = 6750 – 75X
50. 1 and
75X = 2250; X = 30
parallelogram. If m
................................................
A. 40o
Explanation:
measure of
A. 25o
C. 100o
Explanation:
Exterior angles on the same side of the transversal are supplementary. *Mnemonic: SST
(same side of transversal) means supplementary. ALTERNATE (either interior or
exterior) means congruent. Also, CORRESPONDING angles are congruent.
This is a free reviewer. All rights reserved.
A. 33% 56. Car A is traveling towards the east at a speed of 35 kph, while car B is traveling
towards the west at 45 kph. If they left the same point at 1:00 PM, how far apart are
they at 3:45 PM?
C. 66%
Solution: A. 240 km
Solution:
Time spent driving: 1:00 to 3:45 = 2.75 hrs
53. Given sin θ = 0.28, which of the following could possibly be cos θ? (45 mins in decimals is 45/60 since there are 60 mins in 1 hr)
Car A distance from mid: 2.75 (35) = 96.25
Car B distance from mid: 2.75 (45) = 123.75
Total distance: 123.75 + 96.25 = 220 km
A. 0.72
Alternative Solution:
C. 0.96 Since the two cars are traveling in two opposite directions, add their speeds and multiply by elapsed time.
Solution: 2.75 (45+35) = 2.75 (80) = 220 km
................................................
sin2 θ + cos2 θ = 1
(0.28)2 + cos2 θ = 1 57. Mr. Santos left the house at 1:00 PM and
cos2 θ = 1 – 0.0784
cos2 θ = 0.9216 traveled east at an average speed of 40 kph. His wife Mrs. Santos left the at 2:00 PM and
cos θ = √0.9216 = ±0.96
................................................ traveled west at an average speed of 30 kph. How far apart are they at 4:00 PM?
A. 180 km
54. If the sum of the supplement and the complement of an angle is 130 degrees, what is the angle?
C. 100 km
Solution:
Mr. Santos’s data:
A. 65o Speed: 40 kph
Elapsed time: 1PM to 4PM = 3 hrs
Distance: 40kph (3hrs) = 120 km
Mrs. Santos’s data:
Speed: 30 kph
C. 50o Elapsed time: 2PM to 4PM = 2 hrs
Distance: 30 kph (2hrs) = 60 km
Solution: Total Distance: 60 + 120 = 180 km
(90-x) + (180-x) = 130
270 – 2x = 130
270 – 130 = 2x
140 = 2x
70 = x
................................................
C. Sunday
Solution:
Every 7 days, it would be a Saturday again.
This is a free reviewer. All rights reserved.
1000 MMR 58. Five consecutive even numbers have a sum of
Author: Victor A. Tondo Jr., LPT
120. What is the sum of the 2nd and 5th even numbers?
A. 46 B. 48 C. 50 D. 52
61. Given f(x) = ln
Solution:
Let x = lowest even number
x + (x+2) + (x+4) + (x+6) + (x+8) = 120
5x + 20 = 120
5x = 100
x = 20
Solution:
C. x3 – 4x – 2
60. If f(x) = x2 + 4x + 3, which of the following is equal to 99? Remember: ln eu = u, wherein u is the exponent to which e is being raised.
The derivative of x2 + 2x is, of course, 2x + 2.
................................................
A. f(11)
C. f(12)
62. Which of the following could be the value of x if x 3(mod 11)?
A.33 B.47 C.52 D.2
Solution: Solution:
x2 + 4x + 3 = 99
x2 + 4x + 3 + 1 = 99 + 1
x2 + 4x + 4 = 100 Just divide the numbers by 11 and see which one gives a remainder of 3.
√( +2) = 100 ................................................
x + 2 = ± 10
x = -2 ± 10
That’s -2+10 or 8, and -2-10 or -12
63. If
A. 12x + 8
Explanation:
Anti-derivatives. If you already forgot how to do that, simply check which choice has a derivative of 6x 2 + 8x – 7.
................................................
64. What is the center of x2 + y2 – 8x + 6y = 0?
A. (-8.6)
C. (-4, 3)
Solution:
The center, C(h,k) is given as h = -D/2 and
k = -E/2 wherein D and E are from the equation x2 + y2 + Dx + Ey + F = 0.
When x is the squared variable, the parabola opens upward when the coefficient of y is positive (example: C. (3x + 2) (4x – 5)
A). ................................................
67. For which value of k does 4x2 + kx + 49 have only one root?
A. -28 B. -14 C. 7/2 D. -7/4
Explanation:
You may use Completing Square Trinomials. The middle term is twice the product of the square roots of the first and Author: Victor A. Tondo Jr., LPT Explanation:
third terms. In the problem,
the middle term is twice the product of √4 and
√49. That’s 2(2x)(7) or 28x. Don’t forget that the middle term could be positive or negative. Since A and B are the roots, then AB pertains to the product of the roots which is given as c/a.
................................................
69.1+2+4+8+…+2048=____
Solution:
You may use the Geometric Series formula which
68. If A and B are the roots of x2 + 7x + 15, what is AB?
is ∑
In this problem, however, you cannot easily use the GS formula since you don’t know n, the number of terms.
I will personally use the shortcut for the sum of a geometric sequence wherein the ratio is 2 or ½.
The shortcut is SUM = 2(largest) – smallest. In this problem, that’s 2(2048)-1 = 4095.
You may also apply this in the next item, #70.
70.24+12+6+3+1.5+…=____
A. 48
Solution:
You may use the Infinite Geometric Series
A. 7√3
C. 3√2
1000 MMR 71. How many terms are there in the sequence
Author: Victor A. Tondo Jr., LPT
̅̅
5, 13, 21, 29, …, 357? 74. C is the midpoint of AB where A is at (-3,4) and B is at (7,-10). Find the coordinates of C.
A. (5,-7) B. (-5,7) C. (2,-3) D. (-2,3)
Solution:
A. 40
Midpoint Formula: ( , )
Midpoint: ( , )
C. 45 ................................................
Solution:
An = A1 + (n – 1)d
357 = 5 + (n – 1)(8)
357-5 = 8(n – 1)
352 =8(n – 1) 75. It is a line segment formed by connecting two non-consecutive vertices of a polygon.
44 = n – 1
45 = n
Alternative Solution: A. side
(this is the “y=mx+b” solution I taught my grade 3 student for Singapore. Yes, Grade 3.)
Before anything else, since this might be “new” to you, your d is our m, your A n is our y, your n is our x, and b is your C. altitude
A1 – d. Explanation:
357 = 8x +(5 – 8)
357 = 8x – 3
360 = 8x
45 = x
................................................ A side is formed by connecting two consecutive vertices of a polygon.
The apothem is only for regular polygons. It is the perpendicular bisector of one of its sides, passing through
the center.
A diagonal is a line segment formed by connecting two non-consecutive vertices of a polygon.
72. How many ways can a group of 5 be selected from 5 boys and 5 girls if the group must contain ................................................
3 boys and 2 girls?
76. Find the equation of the line perpendicular to
2x – 3y = 7, passing through (1,2).
A. 151,200
A. 2x + 3y = 8
C. 252
Solution: C. 2x – 3y = -4
Solution:
A group, committee, or team (any set with no hierarchy of members) calls for Combinations.
To pick 3 boys from a total of 5 boys, use 5C3 and that’s 10. To pick 2 girls from a total of 5 Simply interchange the numerical coefficients of x and y in the original equation, then
girls, use 5C2 and that’s 10. Lastly, 10x10 = 100.
................................................
change the operation between them. 2x – 3y becomes 3x + 2y.
For the constant, simply substitute the x and y values of the point ((1 ,2) in this problem) and
solve for the constant.
3(1) + 2(2)=7.
Thus, 3x+2y=7.
73. What is the probability of getting a sum of 9 when rolling 2 dice?
A. 1/9
C. 1/6
The only pairs with a sum of 9 are (3,6), (4,5), (5,4), and (6,3). There are only 4 pairs out of 36.
A. 40o 78. The measure of each interior angle of a regular polygon is 144o. How many vertices does it have?
C. 130o
Explanation: A. 36
Solution:
Personally, I always go for the exterior angle first to get the number of sides or vertices. Since the exterior Since the parabola opens upwards, then the parts less than 0 should be between the
zeros of the inequality. That means x should be between -9 and 3.
and interior are supplementary, then each exterior measures 180-144 or 36. The formula for number of ................................................
sides or vertices given the measure of each exterior is 360÷MEA, so that’s 360÷36 or 10 vertices.
By the way, you may derive this solution by
80. The product of two consecutive even counting numbers is 3248. Find the smaller number.
A. 42 B. 46 C. 52 D. 56
manipulating the formula for MIA:
Solution:
x(x+2) = 3248
x2 + 2x = 3248
x2 + 2x + 1 = 3249
√ +2 +1=√3249
x +1=57
x = 56
becomes 180 –
WAIS Solution:
Get your scientific calculator, extract √3248 and then scrape the decimals or round down. #2EZ4U
................................................
or
................................................
79. Solve: (x + 9) (x – 3) < 0
81. Solve for x: 2log2 3 – log2 18 = x
A. -9 < x < 3
A. ½
C. x < -9 x > 3
82. Twinkle Bucks has four serving sizes for their milk tea: Small, Medium, Large, and
Extra Large. What level of data are they using for their serving sizes?
86. If f(x) = x2 + 4x + 4 and g(x) = x-2, find f(g(x)).
A. nominal
A. x2
C. interval
................................................
C. x2 + 8x + 16
Solution:
f(g(x)) = f(x-2)
83. After receiving a 20% markup, a bag was sold for P960. How much was it originally? =
=
(x-2)2 + 4(x-2) + 4
(x2 – 4x + 4)+ (4x – 8) + 4 = x2
................................................
A. P1152
87. A 10 ft ladder leans against a wall, forming a 30o angle with it. How high on the wall does it reach?
C. P800
Solution:
Selling Price = Original Price (1 + Markup Rate)
960 = OP (1 + 0.20)
960/1.2 = OP
800 = OP
................................................
A. 5 ft
C. 10 √3
means
ABT.
................................................
85. A cone has a radius of 9 cm and a slant height of 15 cm. Find its volume.
A. 243 π cm3
C. 405 π cm3
Explanation:
Be careful with cones. Tendency kasi sa LET that they will give the slant height while looking for volume and the
height while looking for the
This is a free reviewer. All rights reserved.
1000 MMR
Explanation:
6(x-y) = 6x – 6y = 2(3x) – 3(2y)
2(3x) – 3(2y) = 2(7) – 3(5) = 14-15 = -1
................................................
C. 120
Solution: C. 12√3
(0.6)(0.8)(500) = 240
................................................ Solution:
90. If 3x = 7 and 2y = 5, what is 6(x-y)? Let A and B be our two numbers.
AB=71; A2+B2=147
.: A2 + B2 + 2AB = 147 + 2(71) = 289 (A+B)2=289; A+B=±17
................................................
A. 10 and 11
C. 10.5
Solution:
Rearrange the numbers from least to greatest:
C. √7
Solution:
7, 9, 9, 10, 11, 13, 14, 17
Author: Victor A. Tondo Jr., LPT
93. How many 3-digit numbers can be formed using the digits 0, 1, 2, 3, 4 and 5 if repetition is not allowed?
A. 60 B. 80 C. 100 D. 120
Solution:
Use FCP (Fundamental Counting Principle):
The median is the __ x __ x __
For the first digit, we cannot use 0. That means we only have 5 choices for the first digit.
means we have to get half the sum of our 4th and 5th numbers. (10+11)/2 = 10.5 For the second digit, we can now use 0. Since we have already used one digit for the first, that means we
have 5 choices for the second digit. For the last digit, since we have already used two digits, we only have 4
choices.
5 x 5 x 4 = 100
................................................
94. How many ml of 20% acid must be added to
400 ml of 50% acid to make a 30% acid solution?
A. 1000 ml
C. 800 ml
Solution:
C1 V1 + C2 V2 = CRVR
20 (V) + 50 (400) = 30 (V + 400)
20V + 20,000 = 30V + 12,000
20,000 – 12,000 = 30V – 20 V
8,000 = 10V; 800 = V
................................................
95. How many ml each of 10% and 50% solution should be mixed to make 500 ml of 18% solution?
A. 400 ml of 10% and 100 ml of 50% B. 350 ml of 10% and 150 ml of 50% C.
300 ml of 10% and 200 ml of 50% D. 200 ml of 10% and 300 ml of 50%
Solution:
Since our resultant volume is 500, then our two volumes will be x and (500-x). C1 V1 + C2 V2 = CRVR
10(x) + 50(500-x) = 18(500)
10x + 25,000 – 50x = 9,000
An isosceles trapezoid has congruent diagonals, however, they do not bisect each other, nor are they
perpendicular.
A rhombus has diagonals that are perpendicular and that bisect each other. However, they are not congruent.
A. 28 days
PS: A square has diagonals that are congruent, perpendicular, and that bisect each other.
................................................
C. 21 days 100. A pipe can fill a pool in 6 hours while another pipe can drain empty the pool in
15 hours. How long will it take to fill the pool if both pipes are open?
Solution: A. 9 hours
A. undefined
A. 3
Solution:
C. 8 log n – 1 = 2
log n = 2 + 1 = 3
Explanation: (note that the base of the log is 10) log n = 3 translates to 10 3 = n Therefore n =
1000
................................................
You may simplify the function first before substituting x with 4. 102. log2 3 + 2 log2 7 – log2 5 = ______.
................................................
98. A box contains 7 red, 8 blue, and 9 white balls. When taking two balls in succession, what is the A. log2
probability that both balls are white?
A. 9/64
C. 7/64
Solution: C. log2
First white ball: 9/24
Second white ball: 8/23
9/24 x 8/23 = 9/69
................................................
Explanation:
Just apply the laws of logarithms.
99. Which of the following has two diagonals that are perpendicular bisectors of each
other?
A. kite
C. rhombus
Explanation:
A kite’s diagonals are perpendicular but only one diagonal will bisect the other.
This is a free reviewer. All rights reserved.
1000 MMR A. Counterexample: The prime number, 2. The square of 2 is 4 which is even.
103. The surface areas of two spheres are 12 π cm2 and 108 π cm2. What is the ratio of their volumes?
B. Always false. One example is 2 and 4. Their sum, 6, is not divisible by 4.
C. Counterexample: The prime even number, 2. D. Proof by Algebra: Let the first even
number be 2x. The second even number will be 2x + 2. Their product will be 4x2 + 4x.
If x is an odd number, x = 2y + 1 where y is a counting number. 4x2 + 4x = 4(4y2 + 4y
A. 1:3√3
+ 1) + 4(2y + 1) = 16y2 + 16y + 4 + 8y + 4 = 16y2 + 24y + 8, which is divisible by 8.
If x is even, x = 2y where y is any counting number. 4(4y 2) + 4(2y) = 16y2 + 8y, which is also divisible by 8.
Either ways, the statement holds true.
C. 1:27
Solution:
Ratio of surface areas: 12:108 or 1:9
Ratio of radii: √1: √9 or 1:3
Ratio of volumes: 13:33 or 1:27
................................................
104. The volume of a regular hexahedron is 64 in3. How long is each side?
A. 2 in B. 4 in C. 6 in D. 8 in
Explanation:
A regular hexahedron is simply a cube.
................................................
N=
A. 300 π cm3
Solution:
Vol = π r2 h = 52 (12) π = 300 π cm3
................................................
107. A cube sits perfectly inside a sphere of 108. Find the distance in cm of an 80 cm chord from the center of a circle whose radius is 41 cm.
volume 108 √3 π cm3. Find the volume of the cube.
A. 27 cm3
A. 41 - 2√10
C. 108 cm3
Solution:
Volume of sphere = 108√3 π cm3 C. 9√2
π r3 = 108√3 π
r3 = (108)√3 Solution:
r3 = 81√3; r = 3√3; d = 6√3
Diagonal of cube = s√3 = 6√3
.: s = 6; volume = s3 = 63 = 216 The chord is perpendicularly bisected by a segment connected to the center of the circle, whose length is
Alternative Solution: the distance we are looking for. If the radius is drawn connected to one endpoint of the chord, we can form
a right triangle whose hypotenuse is the radius and one leg is half of the chord. Using the Pythagorean
Ratio of volume of cube to sphere (cube is inside sphere) = 2 : √3 π theorem, the
distance is √41 40 or simply, 9.
N : 108 √3 π = 2 : √3 π
This is a free reviewer. All rights reserved.
̅̅
A.
114. Find the area of the triangle whose vertices are (1,4), (2,3), and (3,0).
MC
A. 0
Explanation:
m T = 180-(40+60) = 80
The longest side is opposite the largest angle, T.
................................................
Solution:
111. Find the altitude to the hypotenuse of a right triangle whose legs measure 10 cm and 24 cm.
A. 120 cm
Note: If the result is negative, that means your points are simply arranged clockwise. Just get the absolute value
of the answer.
................................................
D. y-1 =
Solution:
y = x2 + 10x
y + 25 = x2 + 10x + 25
y + 25 = (x+5)2
√ + 25 = x + 5 √ + 25 – 5 = x
√ + 25 – 5 = y-1
................................................
C. z =
Fe
Solution:
Sita
x – 12 = (2)(48 – x)
x – 12 = 96 – 2x
Make two equations wherein y will have the same numerical coefficients.
x + 2x = 96 + 12
x + 2x = 108
x = 36 2x = 3y
60 – x = 60 – 36 = 24
................................................
118. If the length of a rectangle is increased by 4y = 5z
20% while the width is decreased by 10%, what will happen to its area?
A. decreased by 10% B. increased by 10% C.
increased by 8% D. decreased by 2%
Solution:
(L x 1.2) (W x 0.9) = (1.08 x LW)
................................................
119. The 19th term of an arithmetic sequence is
85 and the 12th term is 43. Find the common difference.
A. 5
Solution:
d=
A. z =
C.X=
Solution:
X= Y
Author: Victor A. Tondo Jr., LPT By transitive property of equality, X = ( Z) = Z or Z
8x = 15z, or x=z
................................................
................................................
121. Victor had an average of 94 on his first four Math tests. After taking the next test, his average dropped to 123. Two buses leave the same station at 8:00 pm. One bus travels north at the rate of 30
93. Find his most recent grade.
A. 88 B. 89 C. 90 D. 91
kph and the other travels east at 40 kph. How many kilometers apart are the buses at 10 pm?
Solution:
A. 140 km
Solution:
From 8 to 10 PM is 2 hours. After two hours, one bus will have travelled 60 km while the other, 80 km.
Since the two buses are traveling on perpendicular directions, we can use the Pythagorean Theorem to find
their distance.
122. X is D = √60 + 80 = 100 km
................................................
124. A bus drove for 6 hours at 75 kph and 4 hours at 80 kph. What was its average speed?
A. 76 kph
A.X=
C. 77.5 kph
129. Evaluate
126. Rationalize:
A. -38
√
A.
Solution:
+
C.
÷ = x = 19
................................................
Solution: 130. Today, Vic is 11 years old while his father is
37. How many years from now will his father be twice as old as he?
To rationalize this, multiply both numerator and denominator by the conjugate of the A. 15
Solution:
B. 13 C. 11 D. 10
denominator. By doing this, we are sure to have a rational denominator. Let x = number of years from now
2(11+x) = 37+x
22 + 2x = 37 + x
2x – x = 37 – 22; 15 = x
................................................
131. Carla and Diana are on a seesaw. Carla weighs 50 kg and sits 168 cm to the left of the
fulcrum. If Diana weighs 60 kg, how far to the right of the fulcrum must she sit to balance the seesaw?
................................................
A. 140 cm
127. RNHS has 130 quizzers. 67 of them are Math, 60 are Science, and 20 are C. 201.6 cm
quizzers for both Math and Science. How many quizzers are neither Math nor Science?
A. 0
Solution:
(A B)’=
(A B)’ = 130
(A B)’ = 130
132. Twenty guests shake hands with each other. If each guest is to shake hands with all the other guests, how A. 5
many handshakes will be made?
A. 400 B. 380 C. 200 D. 190 Solution:
Solution:
20C2 = 190
................................................ Remember that for a number to be divisible by 9, the sum of its digits must be equal to 9. 3+4+k+6+7 = 20+k
133. How many line segments can be made from 2+0+k = 2 + k = 9; k=7
30 non-collinear points? ................................................
A. 900 B. 870 C. 450 D. 435
Solution:
30C2 = 435
................................................
134. The longest chord of a circle is 80 cm. How long is its radius?
A. 20 cm
C. 20√2
Explanation:
Author: Victor A. Tondo Jr., LPT 138. Find the product of two numbers whose GCF is 24 and LCM is 120.
136. Find the largest area of a rectangle whose perimeter is 100 cm. A. 2880
A. 2500 cm2
Explanation:
The product of two numbers is equal to the product of their LCM and GCF.
C. 625 cm2 24 x 120 = 2880
................................................
Solution:
Instead of jumping to differential calculus (minima and maxima) to solve this, simply make 139. The salary of 4 men for 5 days is P9,000. How much is the salary of 5 men for 6 days?
it a square. That’s the shortcut for this kind of question.
................................................
( ) (
C. 130%
Solution:
(x + 5) + (2x – 4) + (x + 7) = 60
4x + 8 = 60
4x = 52
x = 13
................................................
145. Mia is 16 years younger than Kia. 13 years ago, Kia was thrice as old as Mia. What is Kia’s present age?
A. 43 B. 40 C. 37 D. 34
x = 0.8 y
=y
.
1.25x = y
................................................
A.6PM
C. 11 PM
Solution:
60x = 80(x-2)
160 = 20x;
8=x
................................................
Explanation:
The degree of a polynomial is the highest sum of
exponents in a term.
This is a free reviewer. All rights reserved.
1000 MMR
Solution:
Author: Victor A. Tondo Jr., LPT
Let P = number of pigs;
C = number of chickens
150. Solve for x:
P+C=100 A. 1.142857
4P+2C=340
C. 1.5
Solution:
2(P + C = 100) First, express both numbers as powers of the same base.
49x = 343 (72)x = 73
4P+2C=340
Next, apply the laws of exponents.
(72)x = 73
72x = 73
2x = 3;
x = 3/2 or 1.5
................................................
................................................
151. What is the highest possible product of two numbers if their sum is 45?
A. 506
148. Adam can do a job alone in 8 hours, while Bam can do the same job in 12 hours. One day, they
worked together for 1 hour before Bam left Adam to finish the job. How long will it take Adam to finish
the remaining job? C. 506.5
Solution:
A. 6 hrs 50 mins
Instead of jumping straight to minima and maxima under differential calculus, simply
C. 6 hrs 30 mins make your numbers equal to maximize their product. 45/2 = 22.5
Both numbers will be 22.5, so their product is 22.5 x 22.5 = 506.25
................................................
Solution:
152. Which statistical test is used for comparing observed frequencies to expected frequencies?
A. ANOVA
C. Pearson R
Explanation:
Observed vs Expected: Chi Square
Relationship: Pearson R (R for relationship)
Group differences: ANOVA (variance = differences)
Comparing sets of normal distributions: T-test ................................................
153. The product of two consecutive odd
counting numbers is 1443. What is their sum?
That’s 6 hrs. and 20 mins.
Mnemonic: A. 76
For questions like this (about working together and then someone leaves), use PuTS U. When someone leaves you,
“PuTS U” !!
PuTS U stands for P(u)roduct, Time, Sum, Umalis ................................................
A. 144
Solution:
Express both sides as a power of 3.
(33)48 = (32)x
3144 = 32x
144 = 2x
72 = x
................................................
Solution:
x2 + 2x
x2 + 2x
√ + 2x + 1 = √1444 + = + = =
x+1 = 38;
................................................
x = 37 x+2 = 39 ................................................
Explanation:
154. Given The divisibility rule for 4 tells us that our concern would only be the last 2 digits.
15 ÷ 4 = 3 r. 3
................................................
find lim
159. Dividing by 0.125 is the same as multiplying by which number?
A.5 B.8 C.10 D.16
Explanation:
A. 4 Just use 1 as your test number.
1÷ 0.125 = 8
................................................
C. 0
Solution:
Limit from the left: 2(4) + 1 = 9
Limit from the right: 42 – 7 = 9 160. Find the surface area of a sphere whose radius is 6 cm.
Since both limits are equal, then the limit is 9.
................................................
A. 72 π cm2
155. If today is a Saturday, what day is 125 days from now? C. 144 π cm2
Solution:
A. Friday Surface Area = 4 π r2 = 4 (62) π = 144 π
161. Which of the following is the reference angle of 216 o?
A. 84o B. 66o C. 54o D. 36o
C. Monday Explanation:
Solution:
This is an application of modulo. The reference angle for angles from the different quadrants are as follows: QI: the angle θ itself
125 6 (mod 7) or 125÷7 = 17 r. 6
6 days after Saturday is Friday
................................................
QII. 180 – θ
QIII. θ – 180
QIV. 360 – θ
156. If the sum of the supplement and the complement of an angle is 124, what is the angle?
A. 71 B. 72 C. 73 D. 74
Solution:
(180 – x) + (90 – x) = 124
270 – 2x = 124
270 – 124 = 2x
146 = 2x; 73 = x
................................................
This is a free reviewer. All rights reserved.
1000 MMR Author: Victor A. Tondo Jr., LPT
162. Which of the following angles in standard position is coterminal with 40 o? 165. Find the equation of the line passing through the point of origin and (3,4).
A. 2200o
A. y =
C. 1520o
Explanation:
Coterminal angles are congruent, modulo 360. That means they will leave the same remainder when divided by
360.
In textbooks, θ is coterminal with any angle expressed as 360N + θ wherein N is an integer. To easily tackle
C. y =
this question, simply subtract 40 from each of the choices, then see if any of those is divisible by 360 (or
leaves a remainder of 0 when divided by 360) using your calculator. 2200 – 40 = 2160
2160 ÷ 360 = 6
.: 2200o is coterminal with 40o
................................................
Solution:
163. Find the equation of the line passing through (2,7) and (-3,-3).
A. y = 4x -1
y – y1 =
C. y = 3x + 6
Solution:
Two-point form of linear equations:
y – y1 = (x x ) y–0=
y= x
................................................
166. Find the range of f(x) = -2x2 + 4x.
y–7=
y – 7 = 2(x – 2) A. y
y – 7 = 2x – 4
y = 2x + 3
................................................
C. y
Explanation:
164. In which quadrant can we find θ if tan θ < 0 and sin θ > 0?
A. First Quadrant B. Second Quadrant C. Since this is a quadratic function, you need to know two things to determine its range: its opening and k of its
vertex (h,k).
Third Quadrant D. Fourth Quadrant The parabola opens downward since a = -2.
Explanation: k = c – (b2/4a) = -16/(-8) = 2
Since the parabola opens downward, the graph starts from - , going to k which is 2. Thus, y 2.
................................................
Use the CAST mnemonic.
A. 4
Solution:
a3/2 – 1 = 7
a3/2 = 8
(a3/2 = 23)2/3
a = 22 = 4
................................................
C. 60o
Author: Victor A. Tondo Jr., LPT Solution:
P250 = discount
Solution: 10% = discount rate
Original Price (OP) = ???
MEA = 360/N = 360/5 = 72o Selling Price = ???
................................................ DC = OP x DC Rate
170. How many diagonals does a nonagon have? 250 = OP x (0.1)
250/0.1 = OP
2500 = OP
A. 27
Selling Price = OP – DC
Selling Price = 2500 – 250 = P2250
Solution: ................................................
A. A. P330
Solution:
SP = OP (1-DC Rate)
Let x = 0.123123123123… 1000x = 123.123123123123… 270/0.9 = OP
270 = OP (0.9)
172. Mrs. Pasay saved P250 after buying a phone with a 10% discount. How much did she pay for the phone?
C. 60√3
A. P2500
Solution:
C. P2000
AreaEqTri =
................................................
C. 60 cm2
180. A circle is drawn inside a triangle such that it is tangent to the sides of the triangle. Its center will be
Solution: the triangle’s ___________________.
Ratio of sides = 9:12 or 3:4 Ratio of areas = 32:42 or 9:16 9:16 = 27:N A. Incenter
16(27) = 9N; 48 = N
................................................
C. Centroid
Explanation:
177. Find the remainder when x4 – 3x3 + 2x2 + 3x – 9 is divided by (x-3).
If the circle is inside the triangle, its center is the INcenter. If the circle circumscribes the triangle, its center is the
CIRCUMcenter.
................................................
A. -18
181. Rayon can do a job in 3 hours, while Carlyn can do the same job in 7 hours. How long will it take them to
finish the job by working together?
Solution:
A. 2.1 hours
x-a = x-3
178. Which of the following has its incenter, circumcenter, centroid, and orthocenter in just one point?
182. This line is perpendicular to one side of the triangle passing through the opposite vertex.
A. Longitude
A. Right Triangles
C. Altitude
C. Isosceles Triangles
................................................
Explanation:
Altitude: perpendicular to one side, passing through the opposite vertex.
179. Dexter is twice as heavy as Pablo. Ming is 4kg heavier than Pablo. The sum of their masses is 164kg. How Median: line segment from midpoint of one side to opposite vertex
heavy is Dexter? Bisector: line segment that bisects an angle of a triangle
A. 40 kg
C. 80 kg
wooden server on top of the table)? This problem is about Circular Permutations or arrangements on a circle. The formula is (N-
A. 720 B. 120 C. 36 D. 30 1)!. ................................................
Author: Victor A. Tondo Jr., LPT
184. In parallelogram MATH, m M = 7x – 12 and m T = 5x 186. How many ways can the letters of the word BANANA be rearranged?
A. 720 B. 240 C. 120 D. 60
+ 32. Find m A.
Solution:
This is a permutation with repeated elements:
A. 22
C. 44
P=
Solution:
elements (or letters) and a!, b!, c!, … are the number of times the different elements (or letters) were
repeated.
BANANA has 6 letters: 1 B, 3 A, and 2 N
M and P= ! !! != ( )( )=60
................................................
therefore m M = m T.
7x – 12 = 5x + 32
7x – 5x = 32 + 12
2x = 44
x = 22
187. “The temperature in Baguio City is 20 o while the temperature in
Tuguegarao City is 40o”. What level of data is temperature in degrees Celsius?
.: m M = 7(22)-12 = 154-12 = 142 M and A are consecutive angles,
therefore m M + m A = 180.
142 + m A = 180
m A=180–142 m A = 38 A. Nominal
................................................
C. Interval
Explanation:
185. Find the equation of the line perpendicular to 2x + 5y = 7, passing through (1, 2).
Since you cannot infer that Tuguegarao City is TWICE AS HOT as Baguio City, then the data is not ratio.
A. 2x + 5y = 12
C. 5x + 2y = 9
Remember: Temperature in degrees Celsius or Fahrenheit is interval, but temperature in Kelvin is ratio.
Solution: ................................................
Just like what we did in item #76, simply interchange the numerical coefficients of x and y
in the original equation, then change the operation between them. For the constant, 188. What is formed by the intersection of two planes?
simply substitute the x and y values of the point and solve.
A. a point
C. a plane
................................................
189. What is formed when a plane intersects a cone parallel to its circular base?
A. ellipse
B. hyperbola C. circle
D. parabola
190. In which non-Euclidean model for geometry can we have any given line ℓ and a C.90
point A which is not on ℓ, wherein all lines through A will intersect ℓ? Solution:
A. hyperbolic
x = x + 33
C. Saccheri
Explanation:
20 ( x = x + 33)
In Euclidean geometry, only one line will pass through A. In elliptic geometry, all lines will pass through A and
intersect ℓ. 11x = 660
................................................ x = 60
................................................
191. Which numerical system is sexagesimal (base-60)? 195. Which of the following has the greatest value:
A. Mayan A.3+32+(3+3)2
Explanation: Solution:
Mayans: base-20 (vigesimal)
Babylonians: base-60 (sexagesimal)
Romans and Hindu-Arabic: base-10 (decimal)
................................................ A.3+32+(3+3)2=3+9+36=48 B.33=27
C. [(3 + 3)2]2 = 362 = 1296
D.(3+3+3)2=92=81
................................................
192. Which numerical system makes use of dots and horizontal lines, and shell shapes for zero?
196. Which of the following has an undefined slope?
A. Egyptian
A. a vertical line B. a horizontal line
C. a line parallel to the x-axis D. a diagonal line
C. Greek Horizontal line (parallel to x-axis): m = 0
Vertical line (parallel to y-axis): m is undefined
................................................ Slanted downwards to the right: m is negative
Slanted upwards to the right: m is positive
................................................
C. porygon
A. stratified
C. systematic
Explanation:
sub-classifications
area
..........
A. Chi Square
C. ANOVA
..........
Fibonacci sequence?
A. Ptolemy
C. Pierre de Fermat
D. Luca Pacioli
................................................
theorem?
A. Pythagoras
C. Daniel Bernoulli
................................................
sides.
A. P230,000
C. P233,050
Solution: C. 200
Since the interest is compounded annually, Acct = Principal x (1 + rate)time Acct = 200,000 x (1.053)
Acct = 200,000 x 1.157625 = 231,525 Solution:
................................................
Since the hypotenuse is 40 cm, then the leg opposite 30 o is 20 cm (half the hypotenuse), and
the leg opposite 60o is 20√3 cm (√3 times the short leg). The area of a right triangle is given by
208. Find the remainder when the polynomial x4 – 3x3 + 2x2 – 5x + 8 is divided by (x – 3). = .
A.5 B.8 C.11 D.14 √
= = 200√3 cm2
Solution: ................................................
Use the Remainder Theorem.
Our divisor is (x – 3), so a = 3.
f(3) = 34 – 3 (33) + 2 (32) – 5(3) + 8
f(3) = 81 – 81 + 18 – 15 + 8 = 11
................................................
212. Nine cans of soda and four hamburgers cost a total of P257. Five cans of soda and
seven hamburgers cost a total of P224. How much is a can of soda?
A. P17 B. P19 C. P21 D. P23
Solution:
209. What is 60% of 120?
A. 50
Solution:
................................................
A. 145.45% B. 135%
C. 68.75% D. 44%
Solution:
Identify the Part (in textbooks, they use the word
“Percentage”), the Base, and the Rate when you
face questions like this. Part comes with the
word “is”, Base comes with the word “of”, while
This is a free reviewer. All rights reserved.
1000 MMR
213. The product of two consecutive even numbers is 728. What is the smaller number?
A. 22 B. 24 C. 26 D. 28
Solution:
Let x = smaller number;
x + 2 = larger number
h=
x (x+2) = 728
x2 + 2x = 728;
x2 + 2x + 1 = 729
√x + 2x + 1 = √729
x + 1 = 27
x = 26
................................................
214. What time is 219 minutes past 6:40 AM?
A. 8:59 AM h=
................................................
C. 9:49 AM
Solution: 216. After getting a 20% discount, Mr. Lopez paid P4,000 for a gadget. How much was its original
219 minutes = 3 hrs 39 mins price?
6:40
+ 3:39
9:79 or 10:19
................................................ A. P4,800
215. Find the vertex of y = 3x2 – 2x + 11.
C. P8,000
A.
C. (
Solution:
h=
172 = 289; (17+6)2 = 232 = 529
529 – 289 = 240
................................................
5000 = OP
................................................
C. y
Solution:
217. When a number is increased by 3, its square increases by 111. By what does its square increase when
the number is increased by 6?
This is a quadratic function so the graph is a parabola opening upwards (since A = 2). Solve for k to find its
minimum value.
A. 222
k = c – b2/4a = 9 – 64/8
k=9–8=1
Solution:
Therefore, y
A. x
C. x
n=
222. The diagonal of a rectangular prism is 13 cm long. If it is 3 cm thick and 12 cm long, how wide is it?
A. 3 cm
n=
C.4√3
Sum =
Solution:
Solution:
When y is raised to an even number, it automatically becomes not a function. 213 = 20% x Original number of apples
213 ÷ 0.2 = Original number of apples
1065 = Original number of apples
................................................
226. When a number is increased by 4, its square also increases by 168. What is this number?
A. 15 B. 19 C. 23 D. 27
Solution:
(x+4)2 – x2 = 168
x2 + 8x + 16 – x2 = 168
8x + 16 = 168
8x = 152
x = 19
................................................
trinomial:
A. 10
Solution:
9x2 + kx + 25 = (3x)2 + 2(3x)(5) + 52
= (3x)2 + 30x + 52
................................................
Solution:
229. Which of the following points is on the line y = 2x + 5?
A. (1, 3)
C. (0, 10)
Explanation: (-) y = 3x + 16
Just substitute the x and y-values of each point and see which one makes a true equation.
A. (1, 3)
B. (2, 9)
C. (0, 10)
................................................
231. Find the slope of 3x + 5y = 7.
A. B.
C.
Author: Victor A. Tondo Jr., LPT Solution:
Convert it to its slope-intercept form.
3x + 5y = 7 5y = -3x + 7 y=x+
.: m =
................................................
232. Which of the following is a polynomial?
A. √3
C.
Explanation:
A polynomial accepts only WHOLE numbers as exponents of the variable/s. Only D has whole numbers as
exponents of x.
................................................
Degree refers to the highest exponent or sum of exponents of the variables in any term of a polynomial.
................................................
A. 2.5
Solution:
32=25
5+½=5.5
................................................
235. If y = √3
A. x =
C. x = √
Divide by 3
Finally,
................................................ C. x = 2y + 3 and 2x + 3y = 4 D. y = 5x + 6 and y = 0.2x – 8
Explanation:
236. Which of the following is a pair of parallel lines? Perpendicular lines have negative reciprocal slopes. For choice A, x=5 is horizontal and y=7 is vertical, therefore they
A. y = 2 and x = 2 are perpendicular.
B. 12x + 13y = 14 and 13x + 14y = 15 C. y = 3x + 8 and 3y = x + 9 ................................................
D. 4x + 5y = 6 and 8x + 10y = 21
Explanation:
Parallel lines have the same slope. Both lines have a slope of -4/5 in D.
................................................
Complete the squares on the left side of the equation to return it to its center-radius form.
x2 + y2 + 2x – 4y = 44
x2 + 2x + y2– 4y = 44
(x2 + 2x + 1)+ (y2– 4y +4)= 44 + 1 + 4 (x2 + 2x + 1)+ (y2– 4y +4)= 49
Solution: ................................................
242. Gian has 8 more P5 coins than P1 coins. If he has a total of P106, how many P5 coins does he have?
A. 13 B. 15 C. 17 D. 19
Average Rate of Change = Solution:
Let x = number of P1 coins
.: x+8 = number of P5 coins
238. Find the altitude to the hypotenuse of a right triangle whose sides measure 5 cm, 12 cm, and 13 cm. 1(x) + 5 (x+8) = 106
x + 5x + 40 = 106
6x + 40 = 106
6x = 66
x = 11
.: He has 11 P1 coins and (11+8) or 19 P5 coins.
A.
Solution:
Altitude to the Hyp = (L1 L2)/Hyp = 5(12)/13 Altitude to the Hyp = 60/13
This is a free reviewer. All rights reserved.
1000
243.
MMR
After using half of her budget on bills, one-third on groceries, and P270 on a shirt, Mrs. D still had 246.
Author: Victor A. Tondo Jr., LPT
The average grade of 23 students in Section A is 86, while the average grade of 27 students in Section B is
P130 left. How much was her budget? 91. What is the average grade of all
50 students in both sections?
A. P2400
A. 88.5
C. P3000
Solution:
Let x = budget Solution:
x + x + 270 + 130 = x
x + 400 = x
400 = x – x
400 = x
2400 = x
................................................
Average =
................................................
244. x varies directly as y and inversely as z. If x = 24 when y = 32 and z = 4, what is x when y = 21 and
z = 7? 247. Find the axis of symmetry of y = 3x2 – 5x.
A. 3
A. x =
Solution:
x = ky/z C. x =
Explanation:
The axis of symmetry is located at x = .
................................................
248. Find the range of the following scores:
x = 3y/z
x = 3 (21) / 7
x = 63/7
x=9
................................................ Explanation:
245. Find the mode of the following scores: Range = Highest Score – Lowest Score
................................................
78
249. Mr. C travels for 2 hours at a speed of 38 kph and then north for 3 hours at a speed of 53 kph. What is
his average speed?
79
A. 44 kph
A. 79
C. 47 kph
C. 80
Explanation: Solution:
Mode, by definition, is the score with the highest frequency. Since each score has a frequency of 4, then there is no
mode.
................................................
Average Speed =
Average Speed =
250. Victor, Chris, and Aira volunteered to teach at a nearby daycare. Chris worked for 2 hours less
than Aira. Victor worked twice as many hours as Chris. Altogether, they worked for 58 hours. How many
hours did Victor work?
A. 14 B. 16 C. 28 D. 32
n + (n-2) + 2(n-2) = 58 n +
C. Imaginary circle
n - 2 + 2n - 4 = 58 4n – 6 = 58 4n = Solution:
64 n = 16
.: Victor worked for 2(16-2) or 28 hours. Use CTS (completing trinomial squares) to convert the equation to its center-radius form. x 2 + y2 + 8x – 6y =
................................................ -100
(x2 + 8x + 16) + (y2 – 6y + 9) = -100 + 16 + 9 (x + 4)2 + (y – 3)2 = -75
251. What conic figure does the equation x 2 + y2 + 4x = -4 form? Since r2 = -75, then the radius is imaginary, making it an imaginary circle.
................................................
253. Find the center of x2 + y2 + 6x – 10y = 2.
A. Real circle
A. (6, -10)
C. Imaginary circle
Solution: C. (-3, 5)
Use CTS (completing trinomial squares) to convert the equation to its center-radius form.
x2 + y2 + 4x = -4
(x2 + 4x + 4) + y2 = -4 + 4
(x + 2)2 + y2 = 0
A. x2 + y2 + 4x + 6y = 0 B. x2 + y2 + 4x + 6y = 12
C. x2 + y2 – 4x – 6y = 0 D. x2 + y2 – 4x – 6y = 12
Solution: 256. Find the equation of the horizontal line passing through (-3, 4).
Simply substitute x and y from your point. If the equation holds true, then the given line passes through your point.
260. Find the distance between the line 3x + 4y – 5 = 0 and the point (8, -1).
A.2 B.3 C.4 D.5
Solution:
y=x+5 Use the formula for distance of point (x1, y1)
from line Ax + By + C = 0: D = √
y = 2x – 8
y=5–x
D=
y = 5 – 2x
................................................
258. Given that I(2, -3) is the midpoint of V(-4, 5) and C, find the coordinates of C.
D=
A. (-1, 1)
................................................
C. (8, -11)
261. B is one-fourth of the way from A(-13,9) to C(7,-7). Find the coordinates of B.
A. (2, -3)
Solution:
C. (2.5, -3.5)
Solution:
Use the midpoint formula: xm = In this question, it is important to note the “from” point and the “to” point.
Abscissa of point B = abscissa of “from” point +
¼ (abscissa of “to” point minus abscissa of “from” point)
Abscissa of point B = -13 + ¼ (7 – (-13))
= -13 + ¼ (20) = -8
Ordinate of point B = ordinate of “from” point +
¼ (ordinate of “to” point minus ordinate of “from” point)
Ordinate of point B = 9 + ¼ (-7 – 9)
=9+¼(-16)=5
2= ................................................
4 = (-4) + x2
8 = x2
................................................
259. The endpoints of the diameter of a circle are A(9, -5) and B(-3, 11). What is the equation of the circle?
Next, find the length of the radius. You can do this by either A) getting half of the length of the diameter AB, or B)
using the center-radius form
This is a free reviewer. All rights reserved.
1000 MMR
261. Find the area of the triangle whose vertices are X(-9, -3), Y(-2, 8), and Z(5, 1).
A. 61 B. 62 C. 63 D. 64
Solution:
The biggest mistake you could make in solving this problem is by getting the equation of one the Author: Victor A. Tondo Jr., LPT Solution:
line formed by one pair of vertices, getting the length of the said side, and getting the distance of
the third point from the line you first obtained, then multiplying the obtained distance by the
length of the line segment and then dividing by two. That’s a very long solution. Two lines are parallel when their slopes are equal. Check the slopes of the lines by converting
them to their slope-intercept form (y = mx + b). A) y + 3x = 5 y = -3x + 5 m = -3
B) x + 3y = 6 y= x+2 m=
A∆=½* D) y = 3x + 8
................................................
A. y + 3x = 9
C. y =
Solution:
Two lines are perpendicular when their slopes are negative reciprocals of each other, or when m 1 x m2 = -1. The
slope of the given line is 3, so
the slope of the other line is . Check the slopes
262. Which of the following is outside the circle defined by the equation (x – 3) 2 + y2 = 40?
of the lines by using their slope-intercept form (y = mx + b).
A. (5, 6)
A) y + 3x = 9
C. (0, -5)
Solution:
A point is outside the circle when its distance from the center is greater than the radius. B) -x + 3y = 10
A: (5,6) (5–3)2+62=40 This point is ON the circle.
B: (7,5) (7–3)2+52>40 This point is OUTSIDE the circle.
C: (0, -5) (0 – 3)2 + (-5)2 < 40 This point is IN the circle.
D: (-1, 4) (-1 – 3)2 + 42 < 40 This point is IN the circle.
................................................ C) y =
263. Which of the following is parallel to the line defined by the equation y = 3x – 4? D) y = 3x – 12
A. y + 3x = 5 B. x + 3y = 6 ................................................
C. y = x+7
265. Which of the following is coincidental to the line defined by the equation y = 2x + 13?
A. y + 2x = 13
C. y =
Solution:
Two lines are coincidental when their equations are equivalent to each other, or when their slopes and y-intercepts
are equal.
A) y + 2x = 13
B) 2x – y + 13 = 0
C) y =
D) 2y = 2x + 13
D. y = 3x + 8
In the form 4A(y – k) = (x – h)2, the length of the latus rectum is 4A.
................................................
D.
Explanation:
270. How far is the vertex from the directrix of the parabola defined by 16y = x 2?
A parabola’s equation includes two variables, one of which is squared. A.16 B.8 C.4 D.2
................................................ Explanation:
The distance of the vertex from the directrix is A (from 4A, which is the length of the latus rectum). The
267. Which of the following equations pertain to a parabola that opens to the right? distance of the vertex from the focus is also A. The vertex is between the focus and the directrix, which
means from focus to directrix is 2A. The distance from focus to any endpoint of the latus rectum is also 2A.
A. 4(y + 3) = (x – 2)2 B. -3(y – 4) = (x + 5) 2 ................................................
C. (y – 6)2 = 5(x + 1) D. (y + 8)2 = -2(x – 3)
Explanation:
271. Find the equation of the directrix of the parabola defined by (y – 2) 2 = -4(x + 3).
The parabola opens to the right when the squared variable is y and the coefficient of x is
positive. It opens to the left when the squared variable is y and the coefficient of x is negative.
................................................ A. x = -2
C. y = -4
268. Which of the following equations pertain to a parabola that opens downward? Explanation:
A. 4(y + 3) = (x – 2)2 B. -3(y – 4) = (x + 5)2
C. (y – 6)2 = 5(x + 1) D. (y + 8)2 = -2(x – 3) To find the directrix, first get the vertex
Explanation: and the opening of the parabola. The
vertex is at (-3, 2) and the parabola
opens to the left, which means the
The parabola opens upward when the squared variable is x and the coefficient of y is positive. It opens to directrix is vertical.
the downward when the squared variable is x and the coefficient of y is negative. ................................................ The directrix is A units from the vertex. Since
4A = -4, then A = -1. The directrix is 1 unit to the right of the vertex (-3, 2), meaning it passes
through (-2, 2). The vertical line passing through (-2, 2) is x = -2. See attached image.
................................................
269. How long is the latus rectum of the parabola defined by 12(y – 4) = (x + 3) 2?
A.12 B.6 C.4 D.3 272. Find the coordinates of the focus of the parabola defined by -12(y – 4) = (x + 5) 2.
A. (-5, 7)
C. (-8, 4)
................................................
A. First quadrant
C. Third quadrant
Explanation:
Quadrant
First
Second
Third
Fourth
................................................
y = 3x + 9 and y = 3x – 12.
A. 7
Solution:
D=
( )
=
D= √ √
................................................
A. (3, 10)
C. (4, 10)
Solution:
y = 2x + 5
2x + 5 = -4x + 23
6x = 18
x=3
y = 2x + 5 = 2(3) + 5 = 11
1000 MMR
279. Which of the following angles is coterminal with 143 o?
A. 217o
C. 323o
Author: Victor A. Tondo Jr., LPT Explanation:
Explanation: The value of a trigonometric function of an angle equals the value of the cofunction of the complement of
the angle. Cofunctions are sine and COsine, tangent and COtangent, and secant and COsecant.
Coterminal angles share the same initial side and terminal sides. Finding coterminal angles is done by
adding or subtracting multiples of 360 or 2π rad to each angle, depending on whether the given angle is in cos θ = sin (90 – θ) sec θ = csc (90 – θ)
degrees or radians. tan θ = cot (90 – θ)
csc θ = sec (90 – θ)
sin θ = cos (90 – θ)
cot θ = tan (90 – θ)
503o = 143o + (1)(360o). ................................................
................................................
283. The hypotenuse of a 30-60-90 triangle is 48 cm long. How long is its shortest side?
280. Which of the following is NOT a trigonometric identity?
A. sin2 θ + cos2 θ = 1 B. tan θ =
C. 1 + tan2 θ = sec2 θ
D. 1 – cot2 θ = csc2 θ
Explanation: A. 24 cm
The correct identity goes 1 + cot2 θ = csc2 θ.
................................................
281. Which of the following is false?
C. 24 √3
A. tan θ = Explanation:
For any 30-60-90 triangle, the ratio of the sides are as follows:
Short leg (opposite of the 30o angle) = n
C. cos θ =
Since the hypotenuse 2n is equal to 48, then n equals 24.
................................................
284. In a right triangle, the side opposite an
angle measuring 50o is 100 cm long. How long is the side adjacent to the 50o angle?
A. 93.45 cm
Explanation: C. 149.14 cm
Solution:
Just use the SohCahToa – ChoShaCao mnemonic. The reciprocal of sine is cosecant (csc),
the reciprocal of cosine is secant (sec), and the reciprocal of tangent is cotangent (cot). The given are an angle and the sides opposite and adjacent to it. Use the tangent function.
................................................ tan 50o =
tan 50o =
x= = 83.91 cm
282. Which of the following is NOT a cofunction identity?
A. cos θ = sin (90 – θ)
B. cot (90 – θ) = tan θ C. sec θ = csc (90 – θ) D. csc θ =
sin (90 – θ)
Solution:
A.
The preferred outcomes are (4,6), (5,5), and (6,4). There are 36 possible outcomes. Therefore the probability is 3/36
or 1/12.
................................................
288. Six-sevenths of a number is 6 less than nine-tenths of the same number. What is the number?
Author: Victor A. Tondo Jr., LPT
A. 130 B. 140 C. 200 D. 210
Solution:
= 6 289. A certain University has a dormitory. If 10 students stay in a room, 24 students
=
10(6x) = 7(9x – 60) will not have a room. If 12 students stay in a room, there will be 6 vacant beds.
60x = 63x – 420
420 = 63x – 60x How many rooms are there in the dormitory? How many students are staying
420 = 3x
140 = x in the dormitory?
................................................ A. 116 B. 115 C. 114 D. 113
Solution:
A.
Solution:
= -0.4 and
A. = -0.8
B. = -0.65
C. = -0.5
D. = -0.6
Only A is beyond -0.4 to -0.75.
................................................
291. Which value describes the position of C?
A. -0.75 B. -0.6 C. -1.25 D. -1.4
C. -148.84
Author: Victor A. Tondo Jr., LPT
Solution:
(x + y)(x – y) = (3.5 + (-8.7))(3.5 – (-8.7))
= (-5.2)(12.2)
= -63.44 297. Subtract: (–2x2 + 5x – 9) – (2x
................................................
293. Which two triangles are similar?
A. 2x2 + 3x + 16
C. -2x2 + 3x – 2
................................................
298. Evaluate the polynomial 4x2 – 6x – 3 if x = 2.
A.-1 B.1 C.3 D.5
Solution:
4x2 – 6x – 3 when x = 2
= 4(2)2 – 6(2) – 3
= 1
................................................
299. Determine the measure of Y and Z.
A. A and B
C. B and D
Explanation:
B and D are isosceles right triangles, or 45-45-90 triangles. Since they have congruent corresponding angles, then
they are similar.
A. 45o, 55o
................................................
A. 4x = 10
A. y = 21 – x
C. y = 3x + 7
................................................ A. 15 sin 72o
C. 72 sin 72o
296. Determine which polynomial expression matches the algebra tile model. Solution:
x is the adjacent side to the 72o angle and 15 is the hypotenuse. The trigonometric function for adjacent
and hypotenuse is cosine. However, none of the choices made use of cos 72 o. Therefore we must use the
other angle which measures 18o. Its opposite side is x and the hypotenuse is still 15. Therefore,
sin 18o =
A. 2x2 + x + 4 15 sin 18o = x
C. 3x2 – x + 4
303. The graph below represents the motion of a car. The graph shows us that the car is: C. 162n + 3
Explanation:
Apply the laws of exponents.
................................................
A. accelerating
B. standing still
C. travelling north-east 306. The greatest number of Fridays that can occur in a 75 day period is:
D. travelling at a constant speed A. 10 B. 11 C. 12 D. 13
................................................
304. The units digit of the number 543444 is: Explanation:
A.3 B.9 C.7 D.1
Let the first day be Friday. The last Friday will be
on the 71st day. That’s a total of 11 Fridays.
................................................
307. Which model is not a function?
A. B.
C. D.
Explanation:
A. -8132
Author: Victor A. Tondo Jr., LPT A. between hours 3 and 4
B. between hours 4 and 5
C. between hours 5 and 6
D. between hours 7 and 8
................................................
A. √28
C. 20 inches
Explanation:
Solution: Use the Pythagorean theorem.
................................................
54×5-6=5-2=
................................................
311. . Which comparison is true? 315. Which equation has infinitely many solutions?
A. 2x + 4 = 7x + 9
B. 3(2x + 5) = 6x + 15
A. 4 < 180.5 < 4.5 C. 4x + 13 = 5x + (20 – x) D. x + 3 = 5x – 21
C. 8.5 < 180.5 < 9.5
Explanation:
180.5 = √18 4.24
................................................
312. A weather station recorded the amount of rain that fell during an 8-hour time
frame using a rain gauge. The findings are recorded in the graph below.
Between which hours was the rate at which the rain fell greater than the rate at which the rain fell between hours 0
and 1?
This is a free reviewer. All rights reserved.
1000 MMR
Explanation:
A. 2x + 4
4–9 B. 3(2x + 5) = 6x + 15
means there are infinitely
many solutions.
C. 4x + 13 = 5x + (20 – x) Author: Victor A. Tondo Jr., LPT 317. Which expression has a value of -2?
4x + 13 = 4x – 20
4x – 4x = -20 – 13
D. x + 3 = 5x – 21
0 = -33 False equation. There is no solution. A. |2| + |-4|
3 + 21 = 5x – x
24 = 4x
6=x One unique solution C. |4| – |-2|
................................................
Explanation:
A.|2|+|-4|=2+4=6
B.|-2|–|4|=2–4=-2
C.|4|–|-2|=4–2=2
316. Praetor jogged on a path that was 2 miles long, took a break, and then jogged back along the D.|-4|+|2|=4+2=6
................................................
same path to where he started. He jogged at different speeds for different distances along the
path as shown in the graph.
318. Reion is tossing a six-sided number cube labeled 1, 2, 3, 4, 5, and 6. What is the probability of tossing 6
twice in a row?
A.
................................................
319. Which represents the value of x in
6 – 4x
A. x
Heart
Rate
A. mean
C. mode
Explanation:
The original mean is 69.833. When 40 is removed from the data, it becomes 75.8. The mean decreases by
5.967.
The original median is 74.5. When 40 is removed from the data, it becomes 78. The median increases by -3.5.
The original mode is 71. When 40 is removed from the data, it stays the same.
The original range is 40. When 40 is removed from the data, it becomes 9. The range decreases by 31.
321. The sum of a number, n, and 5 is subtracted from 8. Which expression represents this statement?
A. 8 – (n + 5)
C. (n + 5) – 8
................................................
322. How is 0.5600 written in scientific notation?
A. 5.6 × 10
C. 5.6 × 10-2
................................................
323. What is the value of x in 3(x – 4) = –21?
A. x = –11
C. x = 3
Solution:
3(x – 4)
x–4
x = -7 + 4 = -3
................................................
324. In the spinner, what is the probability of the arrow NOT landing on the space with the ∆?
A.
................................................
B. C. D.
325. Which values of x and y make the system of equations below true?
2x - y = -1
3x - y = -3
A. x = -4; y = -7
C. x = 2; y = 5
Solution:
2x - y = -1
A.-8 B.8 C.10 D.40
................................................
328. Which of the following is closest to the value of the expression below?
C. 100,000
326. The lengths of two sides of a triangle are 8 inches and 13 inches. Which of the Explanation:
Round off and solve.
following represents x, the possible length in inches of the remaining side of the 202 x 8 x 3.14 = 10,048
triangle? ................................................
A. 5 < x < 21
B. 5 x 21
C. x < 5 or x > 21
D. x 5 or x 21
Explanation: 329. Which of the following expressions has a value of 0?
A.(2–3)–(2–3)
The third side of any triangle must be between the difference and the sum of the two other sides.
................................................
C.(2–3)+(-3+2)
................................................
A. (5x – 7) (2x + 3)
C. (5x + 3) (2x – 7)
A. 7√7
................................................
332. Find the length of the latus rectum of the 336. The shell shape , as used in the Mayan numeral system, is the symbol for which number?
A.100 B.10 C.1 D.0
Explanation:
ellipse defined by The Mayan numerals consisted of only three symbols: zero, represented as a shell shape; one, a dot; and five, a bar.
................................................
337. Which of the following is irrational?
A. A. 0.125
Solution: C. √200
C. √200 cannot be rewritten as a fraction with whole numbers as numerator and denominator
D. √343 is 7.
................................................
333. Let A be a set such that A = {v, w, x, y, z}. How many subsets does set A have?
A.5 B.10 C.25 D.32
Explanation:
Let n = number of elements 338. Aira is six years older than Zayne. Six years ago, she was twice as old as he. How old is Aira now?
Number of subsets = 2n
................................................ A. 21 B. 18 C. 15 D. 12
334. Solve for x: Solution:
2 (5x – 11) + 7 = 3 (x – 7) – 15
Let A = Aira’s present age
A – 6 = Zayne’s present age
A. x = 3
2 (5x – 11) + 7 = 3 (x – 7) – 15
10x – 22 + 7 = 3x – 21 – 15
10x –15 = 3x – 36
7x = -21
x = -3
................................................
C. 3780 x3y4
339. The two parallel sides of a trapezoidal lot measure 100m and 70m. If these sides are 80m apart, what is
the area of the lot? 341. If the difference between the squares of two consecutive counting numbers is 49, what is the larger
number?
A.99 B.49 C.25 D.7
A. 13600 m2
Solution:
C. 3400 m2
Let the consecutive counting numbers be x and x + 1.
(x + 1)2 – x2 = 49
(x2 + 2x + 1) – x2 = 49
2x + 1 = 49
2x = 48
x =24
.: The larger number is 24 + 1 = 25.
Solution: ................................................
Area of Trapezoid =
................................................
340. If y = x and y = 2x + 2, find the value of x.
A. x = -2
C. x = 0
Solution:
3 (x + 4) = 3x + 12 to find the total perimeter in terms of x. Which property did he use to multiply?
C. (3, -4)
343. A ride in a Feak Taxi costs P25.00 for the first km and P10.00 for each additional km.
345. The distance from the sun to the earth is approximately 9.3 × 10 7 miles. What is this distance
Which of the following could be used to calculate the total cost, y, of a ride that was x expressed in standard notation?
km?
A. y = 25x + 10 B. y = 10x + 25 A. 9,300,000,000
C. y = 25(x
C. 93,000,000
................................................
A. -9
Solution: C. 1,024
that 2x
A. y = -1 10P5 =
C. y = 3
Or simply use your calculator to evaluate 10P5 by using the nPr button.
Solution: ................................................
352. Jay bought twenty-five P4.57 stamps. How much did he spend?
A. P 104.25
C. P 119.75
Solution:
25 x 4.57 = 114.25
If you’re using a scientific calculator and it shows
2x you , simply press then S D button.
................................................
353. Given f(x) = x3 + kx2 – 7, find k if f(2) = 41.
3y = 5 A.5 B.10 C.15 D.20
2(1) – 3y = 5
2 – 3y = 5
Solution:
2 – 5 = 3y Just substitute x with 2.
-3 = 3y f(2) = 23 + k(22) – 7
-1 = y 41 = 8 + 4k – 7
................................................ 41 = 4k + 1
349. The sum of two consecutive even integers is 40 = 4k
10 = k
126. What is the smaller integer? ................................................
A. 63 B. 62 B. 61 D. 60
Solution:
Let x = smaller even integer
.: x + 2 = next even integer
A. -8
x = 2x + 2
A. (0, 2)
C. (6, 2)
Explanation:
.: x + y = -2 + (-2) = -4
................................................
Simply substitute the x- and y-values of the point into the equation y = 2x + 2. Whichever point holds a true equation is
the correct answer.
................................................
355. Mulan and Lilo are competing to see who can sell the most candy bars for a fundraiser. Mulan
sold 4 candy bars on the first day and 2 each day after that. Lilo sold 7 on the first day and 1 each day
after that. On what day will they have the same number of candy bars sold? 359. The distance from the earth to the moon is approximately 240,000 miles.
A. 7th B. 6th C. 4th D. 3rd What is this distance expressed in scientific notation?
Solution:
A. 24 × 104
Let x = number of days to pass after the 1st day 4 + 2x = 7 + x
2x – x = 7 – 4
x=3 C. 2.4 × 105
They will have the same number of candy bars sold 3 days after the first day, or on the 4 th ................................................
day. ................................................
356. Which of the following is not a polynomial? 360. =
A.
Solution:
A. -3x2 + x–9
√
C.
Explanation:
................................................
A. (3p – 5) (p + 1)
C. (3p + 1) (p – 5)
................................................
x + (x + 34) = 180
2x + 34 = 180
2x = 146
x = 73
This is a free reviewer. All rights reserved.
C. (3, 2) C. 82 m
Solution: Solution:
3x + y = -9 Let w = width of the field
( 3x 2y = 12 .: 2w + 2 = length of the field
–y = 3 y = -3 w (2w + 2) = 3280
Using the first equation 3x + y = 9, 2w2 + 2w = 3280 divide equation by 2 w2 + w = 1640 complete the square
3x + (-3) = -9
3x = -6
w2 + w + 0.25 = 1640 + 0.25 w2 + w + 0.25 = 1640.25
................................................
x = -2
√ + + 0.25 = √1640.25 w +
363. Find the x-intercept of 3x + 2y = 24 0.5 = 40.5
w = 40
................................................
A. x = 8
C. y = 12 366. Rayon used the following mathematical statement to show he could change an
Solution: expression and still get the same answer on both sides:
To find the x-intercept, let y = 0. 10×(6×5)=(10×6)×5 Which mathematical property did Rayon use?
3x + 2y = 24 3x + 2(0) = 24 A. Identity Property of Multiplication
3x = 24 B. Commutative Property of Multiplication
................................................
x=8 C. Distributive Property of Multiplication over Addition
D. Associative Property of Multiplication
................................................
̅̅
364. A circle is drawn such that is a diameter and its midpoint is O. Given that C is a point on
the circle, what is the measure of ACB? 367. Factorize x3 – 27y3. A. (x – 3y) (x2 – 3x + 9y2) B.
(x – 3y) (x2 + 3x + 9y2) C. (x + 3y) (x2 + 3x – 9 y2) D. (x +
A. 180o 3y) (x2 – 3x + 9 y2)
................................................
368. What is the intersection of the lines
x + 3y = 5 and -2x + 4y = 0?
C. 60o
Explanation: A. (2, 4)
C. (2, 1)
There are only five possible choices for the hundreds digit since we cannot start with 0. There are five
2(x + 3y = 5) choices for the tens since out of the six, we have used one for the hundreds, and we can use 0 for the
tens digit.
We have used two for the hundreds and tens, so we only have four choices for the units digit.
-2x + 4y = 0 5 x 5 x 4 = 100
x + 3y = 5
................................................
369. Given f(x) = 7x3 – 3x2 + 2x – 9, f(2) =
A. 56 B. 48 C. 44 D. 39
Solution:
Simply substitute x with 2.
f(x) = 7x3 – 3x2 + 2x – 9
f(2) = 7(23) – 3(22) + 2(2) – 9
f(2) = 56 – 12 + 4 – 9
f(2) = 39
................................................
371. How many 3-digit numbers can be made using the digits 5, 6, 7, 8, 9, and 0 if repetition is not allowed?
372.
Author: Victor A. Tondo Jr., LPT
A researcher is curious about the IQ of students at the Utrecht University. The entire group of
Subtract Eqn 1 from Eqn 2:
students is an example of a:
A. parameter
B+
C. population
Explanation:
Entirety is population, part of it is sample. (–) B+
................................................
___________________________________________________
G=10
5G=80
G=16
373. Jordan filled a bottle with grains until it was 1/4 full and weighed 8 kg. He added more grains Solving for B,
into the bottle until it was 7/8 full. It now weighed 18 kg. What is the mass of the empty B+¼(16)=8
B+4=8; B=4
bottle? ................................................
C. x < -5
Eqn 1: B +
Eqn 2: B +
*Remember to change your inequality sign after multiplying or dividing the inequality by a negative number. Author: Victor A. Tondo Jr., LPT Solution:
Substitute x and y with -2 and 3 respectively.
................................................ ax + by = 17 -2a + 3b = 17
2ax by = 11 -4a – 3b = -11 Solve for a and b by elimination.
375. How many solutions are there for the following system of linear equations?
-3x + 5y = 6
6x 10y = 0
A. only one solution
B. two solutions
C. infinitely many solutions
D. no solution
Explanation: 4a – 6b = -34
Try to solve for x and y first.
( -4a – 3b =
-11 -9b = -45
-2 (-3x + 5y = 6) 6x
b=5 Using -2a + 3b = 17 and substituting b = 5,
-2a + 3(5) = 17
6x -2a + 15 = 17
-2a = 17 – 15
-2a = 2
a = -1
................................................
377. What is the probability choosing only one vowel when three letters are randomly selected from the
(–) word NUMBERS?
A.
When you get a false equation, it means there is no solution for the given system of linear equations.
Solution:
Also, when there’s no solution for a system of linear equations, the lines defined by the two NUMBERS has two vowels and five consonants. The number of ways we can get only one vowel out of three is taken
equations are parallel – they will never intersect. ................................................ as 2C1 x 5C2, or 20.
The number of ways we can get three letters from seven is 7C3 or 35.
Therefore, the answer is or .
................................................
376. Find a and b so that the system below has the unique solution (-2, 3). 378. If A > B, which is always true?
ax + by = 17 A.
2ax by = 11
B.A2>B2
C.A<B+2
D.A–B>0
A. a = 3, b = -1
C. a = 1, b = 3
379. Statistical techniques that summarize and organize the data are classified as what? 383. What is the center of the circle defined by x2 + y2 – 8x + 6y – 10 = 0?
A. Population statistics B. Sample statistics C.
Descriptive statistics D. Inferential statistics A. (-8, 6)
................................................
C. (-4, 3)
380. Five-point Likert scales (strongly disagree, disagree, neutral, agree, strongly Explanation:
agree) are frequently used to measure motivations and attitudes. A Likert scale is a:
A. Discrete variable. B. Ordinal variable. Since the equation of the circle is already in( the form x2 + y)2 + Dx + Ey + F = 0, then the center
C. Categorical variable. D. All of the above (h, k) is at (
................................................
, ) or ( , ) or (4, -3).
................................................
381. What is the radius of the circle defined by (x + 2)2 + (y – 3)2 = 16? 384. Find the equation of the line passing (1, 4) with slope equal to 5.
A.256 B.16 C.8 D.4
Explanation: A. y = 5x + 3
The equation of the circle is already in the
center-radius form.
Since r2 = 16, then r = 4.
................................................ C. y = 5x – 1
Solution:
Use the form y = mx + b and substitute the x-and y- values of the point. x = 1, y = 4, m = 5
y = mx + b
4 = 5(1) + b
4–5=b
-1 = b
y = mx + b
y = 5x + (-1) or y = 5x – 1
................................................
385. The seminar rooms in the library are identified by the letters A to H. A researcher
records the number of classes held in each room during the first semester. What
kind of graph would be appropriate to present the frequency distributions of these
data?
A. Histogram
C. Bar chart
48o x
388. In now many ways can the letters AAABBCDEEE be arranged in a straight line?
................................................
A. 50,400 391. The median is always:
A. The most frequently occurring score in set of data
B. The middle score when results are ranked in order of magnitude
C. 12,600 C. The same as the average
392. A teacher gave a statistics test to a class of Geography students and computed the measures of
central tendency for the test scores. Which of the following statements cannot be an accurate description of
389. Find the smaller angle formed by the x-axis and the line y = 5x. the scores?
A. The majority of students had scores above the mean.
A. 78.69o B. The majority of students had scores above the median.
C. The majority of students had scores above the mode.
D. All of the above options (A, B and C) are false statements.
C. 54.15o
Explanation:
Solution:
tan =5 A. If majority of the students had scores above the mean, then this is an example of a negatively-skewed data. This can
= tan-1 5
= 78.69o happen.
................................................
390. Convert 48o to radians. B. 50% of the class is always above the median, and the other 50% of the class is below the median.
C. The mode can pop up anywhere. That means the students may have a low modal score.
................................................
A.
C.
C. 28 π cm2
A. 4,320
394. Find the length of each side of an equilateral triangle whose perimeter is 90 cm.
C. 17,280
A. 45 cm
Solution:
C. 22.5 cm
Let 4 girls be one unit. So now, there are 6 units in all: 5 boys and the solo unit made by the four girls. They can be
Solution: arranged in 6! ways.
An equilateral triangle has three congruent sides.
Its perimeter is given as P = 3s. In each of these arrangements 4 girls can be arranged in 4! ways.
3s = 90 Total number of arrangements in which girls are always together
s = 30
................................................ = 6!×4!
= 720 × 24
= 17,280
................................................
395. Find the number of subsets having 4 elements of the set {1,2,3,4,5,6,7,8,9,10,11}.
399. A box contains 8 batteries, 5 of which are good and the other 3 are defective. Two batteries
A. 165 are selected at random and inserted into a toy. If the toy only functions with two good
batteries, what is the probability that the toy will function?
C. 660
Solution:
11C4 = 330 A.
................................................
396. There are ten true - false questions in an exam. How many responses are possible?
A. 1024 B. 256 C. 20 D. 10
Solution:
Solution:
There are two possible responses per item, and there are ten items. 2 10 = 1024.
................................................ =
................................................
397. In a 500m speed skating race, time results would be considered an example of which level of
measurement?
400. IQ tests are standardized so that the mean score is 100 for the entire group of people who
take the test. However, if you select a group of 50 who took the test, you probably would not
A. Nominal
get
100. What statistical concept explains the difference between the two means?
C. Interval
................................................ A. Statistical error
C. Residual error
................................................
C. Archimedes
................................................
Apollonius of Perga was a Greek geometer and astronomer known for his theories on the topic of conic
sections. His definitions of the terms ellipse, parabola, and hyperbola are the ones in use today.
................................................ 404. The book Philosophiæ Naturalis Principia Mathematica, more fondly known simply as Principia, is the
work of which Mathematician?
A. Euclid
402. A researcher studies the factors that determine the number of children future
couples decide to have. The variable ‘number of children’ is a:
A. Discrete variable B. Continuous variable C. C. Einstein
................................................
Categorical variable D. Ordinal variable
................................................
405. A researcher is interested in the travel time of Rayon’s University students to college. A group of
403. Surface area and volume, center of gravity, and hydrostatics are some of the studies of which 50 students is interviewed. Their mean travel time is 16.7 minutes. For this study, the mean of 16.7
Mathematician? minutes is an example of a
A. Apollonius A. parameter
C. population
Author: Victor A. Tondo Jr., LPT
406. Who is considered by many Mathematicians as “The Last Universalist”?
Explanation: A. Jules Henri Poincare B. Hendrik Lorentz C. Georg
Cantor
D. Gottfried Wilhelm Leibniz
The 50 students in the above case is only part of the population of students of Rayon’s Explanation:
University, thus, they are only a sample. Data taken from a sample is called statistic.
Both Poincare and Leibniz are polymaths, but Jules Henri Poincare is fondly referred to
by fellow Mathematicians as the Last Universalist. ................................................
407. In the theory he developed, there are infinite sets of different sizes (called
cardinalities). Which Mathematician formalized many ideas related to infinity and
infinite sets during the late 19th and early 20th centuries? A. Jules Henri Poincare
B. Hendrik Lorentz C. Georg Cantor
D. Gottfried Wilhelm Leibniz
................................................
408. Which of the following sets of scores has the greatest variability or range?
A. 2, 5, 8, 11
Explanation:
Range = highest score – lowest score
................................................
409. This Mathematician was the first to describe a pinwheel calculator in 1685 and
invented the wheel named in his honor, which was used in the arithmometer, the first
mass-produced mechanical calculator. He also refined the binary number system,
which is the foundation of all digital computers. Which Mathematician is this, who is
also crucial to the development of computers?
A. Gottfried Wilhelm Leibniz B. Charles Babbage
C. Ada Lovelace
D. Alexander Graham Bell
A. x = 40
411. Using Calculus, this Mathematician explained why tides occur, why the shapes of planetary orbits
are conic sections, and how to get the shape of a rotating body of fluid, among many other things. Which
Mathematician is this?
A. Kepler
C. Apollonius
Explanation:
412. Which of the following terms does NOT describe the number 9?
A. rational number
C. real number
Explanation:
415. When five is added to three more than a certain number, the result is 29. What is the number?
Its factors are 1, 3, and 9. Therefore, 9 is a composite number. A.24 B.21 C.8 D.4
................................................ Solution:
413. Which expression below is equal to 5? (n + 3) + 5 = 29
n + 8 = 29
n = 21
................................................
A. (1 + 2)2
C.11 10×5
416. The math club is electing new officers. There are 3 candidates for president, 4
................................................
candidates for vice-president, 4 candidates for secretary, and 2 candidates for
treasurer. How many different combinations of officers are possible?
414. A bus picks up a group of tourists at a hotel. The sightseeing bus travels 2 blocks north, 2 blocks A. 13
east, 1 block south, 2 blocks east, and 1 block south. Where is the bus in relation to the hotel?
C. 480
Solution:
A. 2 blocks north Apply the fundamental counting principle.
3 x 4 x 4 x 2 = 96
................................................
C. 3 blocks south
417. Twelve points lie on a circle. How many cyclic quadrilaterals can be drawn by using
these points? [Note: Cyclic quadrilaterals are quadrilaterals whose vertices are on a
circle.]
A. 48
C. 11,880
Solution:
12C4 = 495
This is a free reviewer. All rights reserved.
1000 MMR
418. What is the variance for the following set of
scores? 143
A. 0
Explanation: y – 3 = 2(x – 2)
The scores are all the same, so the variance is 0. y – 3 = 2x – 4
y = 2x – 1
................................................
................................................
419. When 18 is subtracted from six times a certain number, the result is 42. What is the number? 421. Of the following Z-score values, which one represents the location closest to the mean?
A. 10 B. 4 C. -4 D. -10
Solution:
6x – 18 = -42 A. Z = +0.5
6x = -42 + 18
6x = -24
x = -4
................................................ C. Z = -1.5
Explanation:
420. Find the equation of the line passing (2, 3) and (-7, -15) A lower absolute value of Z-score means the item is closer to the mean. The Z-score with the lowest absolute value is
D, Z = -0.3.
................................................
A. y = 2x + 1
C. y = x + 2
Solution:
Use the two-point form.
Author: Victor A. Tondo Jr., LPT
................................................
422. The sum of five consecutive integers is 215. What is the largest of these integers?
A. 43
Solution:
B. 44 C. 45 D. 46
424. What can be said about the following statements?
Let x = smallest number i. Any quadrilateral with four congruent sides is a square.
.: The numbers will be expressed as x, x + 1, x + 2, x + 3, and x + 4. ii. Any square has four congruent sides. A. Only the first statement is true.
x + (x + 1) + (x + 2) + (x + 3) + (x + 4) = 215 B. Only the second statement is true. C. Both statements are true.
5x + 10 = 215 D. Both statements are fall.
5x = 205
x = 41 Explanation:
.: The numbers are 41, 42, 43, 44, and 45.
................................................
i. Any quadrilateral with four congruent sides is a RHOMBUS.
ii. Any square has four congruent sides. TRUE.
................................................
423. You go to the cafeteria for lunch and have a choice of 4 entrees, 5 sides, 5 drinks, and 4
desserts. Assuming you have one of each category, how many different lunches could be made?
A. 18 B. 81 C. 40 D. 400
Solution:
Apply the fundamental counting principle.
4 x 5 x 5 x 4 = 400
C. Skewed
................................................
A. x = 431. A population has a mean of μ=35 and a standard deviation of σ=5. After 3 points are added to
every score of the population, what are the new values for the mean and standard deviation?
Solution: When a constant is added or subtracted to every score of a population, the mean increases or
decreases by the same amount. The standard deviation, however, stays the same.
................................................
The side measuring 10 units is opposite the given angle, while x is the hypotenuse. The trigonometric
ratio that involves the opposite side and the hypotenuse is the sine function. From the mnemonic
SohCahToa, we can recall
that sin = .
sin 35o =
x sin 35o = 10
x= 432. Given sin θ =
................................................
427. The shortest side of a 30-60-90 triangle is
20. 19 cm long. How long is the hypotenuse?
A. 40.38 cm
C. 34.97 cm A.
Explanation:
A. 40.38 cm
C. 34.97 cm
A.
Solution:
Solve for the missing leg by using the Pythagorean theorem. You will get the measure
of the adjacent side as √25 7 , or √625 49 which is equal to 24. Since cos θ =
opposite = √13
A. centroid
C. circumcenter A. 59o
................................................
B. 69o C. 79o D. 89o
................................................
434. Which triangle has the centroid, incenter, circumcenter, orthocenter, and nine-point-center at the same 437. If the scores on a test have a mean of 26 and a standard deviation of 4, what is the z-score for a score
location? of 18?
A. -1.41 B. 11 C. -2 D. 2
A. isosceles right triangle B. 30-60-90 triangle Solution:
C. equilateral triangle D. hyperbolic triangle Z-score =
Z-score = = = 2
................................................
438. If a researcher sets a level of significance at
1. 05 (i.e. 5%), what does this mean?
A. Five times out of 100, a significant result will be found that is due to chance alone and not to true relationship.
C. Five times out of 100, a significant result will be found that is not due to chance, but to true relationship.
B. Ninety-five times out of 100, a significant result will be found that is due to chance alone and not to true
relationship. D. None of the above
441. Which of the following is equidistant from the vertices of the triangle? A. incenter
A. circumcenter C. orthocenter
................................................
C. incenter
................................................
446. How much water must be evaporated from 2000 mL of 30% acid solution to make a 50% acid solution?
442. Which of the following is equidistant from the sides of the triangle?
A. circumcenter
A. 800 mL
C. orthocenter C. 900 mL
Solution:
Note: Water is 0% acid, and evaporation means removal of volume thus –x.
This is a free reviewer. All rights reserved.
1000 MMR Author: Victor A. Tondo Jr., LPT
447. Which of the following is the intersection of angle bisectors of a triangle? 452. In which geometry are there no parallel lines?
A. elliptic geometry
B. hyperbolic geometry C. spatial geometry D. solid
A. circumcenter geometry
................................................
C. centroid
................................................
453. What do we call the ratio of two numbers (larger number: smaller number) whose ratio to each
other is equal to the ratio of their sum to the larger number? [Note: This is applied in Fibonacci sequences]
448. In terms of a conditional statement, what is the statement formed by exchanging and negating the
antecedent and the consequent? A. pi
A. inverse C. 1.618
................................................
C. adverse
................................................
449. What is formed when the hypothesis and the conclusion of the conditional statement are
interchanged?
A. converse
A. x = 8
C. x = 10
C. adverse ................................................
................................................
456. What do we call an angle formed by two chords of the circle with a common endpoint (the vertex
450. What is formed when both the hypothesis and the conclusion of the conditional statement are negated? of the angle)?
A. inscribed angle B. tangential angle
A. converse
C. circumscribed angle D. interior angle
................................................
C. adverse
................................................
= x Then
switch y and x.
= y
Lastly, replace y with y-1.
= y-1.
................................................
458. Which lines are not in the same plane and do not intersect but are not parallel?
A. asymptotes
C. skew lines
................................................
A. 50 tan 37o
C. 50 sin 37o
459. Two adjacent angles whose distinct sides lie on the same line are called what? Solution:
tan 37o =
50 tan 37o = x
................................................
A. linear pair
463. Solve for x:
C. alternate
................................................
A. circumcenter
Solution:
94 + 86 + 94 + 11x – 2 = 360
272 + 11x = 360
461. What do we call three positive integers with the property that the sum of the squares of two of the integers 11x = 360 – 72 = 88
equals the square of the third? ................................................
x=8
A. Euclid’s triple
B. Pythagorean triple
C. Newton’s triple
464. A bus travels 600 km in 7 hrs and another
D. Cartesian triple 300 km in 5 hrs. What is its average speed?
A. 72.86 kph
C. 77.86 kph
Solution:
Average speed = total distance / total time
Average speed = (600 + 300) / (7 + 5)
Average speed = 900/12 = 75 kph
................................................
This problem is on permutation with repeated elements. PILIPINAS has nine letters, of
which two are P, three are I, and the other letters are singular.
9!
2! 3! = 30,240
................................................
C. 10√2
468. A coin is tossed 60 times. Head appeared 27 times. Find the experimental probability of getting heads.
Solution:
A.
Explanation:
Let trajectory = x sin 30o = The experimental probability of an event is the ratio of the number of times the event occurs to the total number of
0.5 = trials.
................................................
x = . = 20
................................................ 469. A parabola is defined by the equation
5x = -3y2 – 4y + 2. Which of the following is true
about the parabola?
A. It opens to the left.
B. It opens to the right.
C. It opens upward.
466. Rowena received a total of 25 bills. These bills are either P20 or P50 bills. If D. It opens downward.
Explanation:
Rowena received an amount of P800, how many P20 bills did she receive?
A. 10 B. 13 C. 15 D. 17
Solution: The squared variable is y and the coefficient of y 2 is negative. Therefore it opens to the left.
................................................
A. 14 B. 13.5
Solution:
up with Ana
60 (x + 2) = 70 (x)
120 = 10x
12 = x
................................................
acid solution?
A. 250 B. 500
Solution:
................................................
angle?
A. 216 cm
C. 216 √3 cm
Explanation:
................................................
1000 MMR
478. A rectangle is 60 cm long and 45 cm wide. How long is its diagonal?
A. 75 cm
C. 95 cm
Author: Victor A. Tondo Jr., LPT Explanation:
Solution:
The diagonal D is given as √ + . This problem is on circular permutations so use (n – 1)!.
................................................
= √45 + 60 = √2025 + 3600 483. There are 24 mangoes in a basket, of which
D = √5625 = 75
................................................ 7 are rotten. What is the probability that when randomly getting two mangoes at the same time, both
are rotten?
479. Find the length of the diagonal of a cube given each side measures 17 cm.
A.
A. √290
C. 17
Solution:
Solution: =
................................................
A. 162o A.
C. 144o
Explanation: Explanation:
MIA = 180 - where n = number of sides of
regular polygon
................................................
There are only seven females, of which one is an admin.
................................................
481. How many diagonals does a regular 14-sided polygon have? 485. What is the remainder when 3x6+ 4x5 – 5x4 + 6x3 + 7x2 – 8x + 3 is divided by (x – 1)?
A.8 B.9 C.10 D.11
A. 77
Solution:
Use the Remainder Theorem:
Solution:
486. Seven people have an average weight of 49 kg. A child was added to the group and the average became
45 kg. How heavy is the child?
A. 15 kg
D=
................................................
C. 17 kg
482. How many ways can 14 people be seated in a Ferris wheel given that each cart can only contain one
person?
A. 15! B. 14! C. 13! D. 12!
487. Six numbers have an average of 71. If 85 is added to the group, what is the new average? D=
................................................
A. 72
489. Triangle ABC has sides measuring 20 cm, 20 cm, and 29 cm. What kind of triangle is ABC?
Solution: A. acute
C. obtuse
For any triangle with sides X, Y, and Z, given that X Y Z, if X 2 + Y2 > Z2, then the triangle is obtuse.
................................................
New average =
................................................
490. Find the surface area of a sphere given that the sphere sits perfectly inside a cube whose sides
measure 20 cm each.
488. In an arithmetic sequence, the 7th term is 25 and the 10th term is 67. What is the common difference?
A. 400
C. 1200
A. 42
+ 2376 x10 – 720 x9
C. f ‘(x) = 550 x21 – 4725 x20 + 150 x19 – 288 x11
+ 2376 x10 – 720 x9
D. f ‘(x) = 450 x21 – 4725 x20 + 1500 x19 – 288 x11
+ 2376 x10 – 720 x9
Solution:
Use the product rule.
Let u = (25 x20 – 24x10)
and v = (x2 – 9x + 3)
.: du = 500 x19 – 240 x9
Author: Victor A. Tondo Jr., LPT Solution: dv = 2x – 9
f ‘(x) = u dv + v du
= (25 x20 – 24x10) (2x – 9) + (x2 – 9x + 3) (500 x19 – 240 x9)
The diameter of the sphere is of the same length as each side of the cube. That means the = 50 x21 – 48 x11 – 225 x20 + 216 x10 + 500 x21 – 4500 x20 + 1500 x19 – 240 x11 + 2160 x10 –
720 x9
diameter of the sphere is 20 cm, and subsequently, the radius is 10 cm. SA = 4 r 2 = 550 x21 – 4725 x20 + 1500 x19 – 288 x11 + 2376 x10 – 720 x9
SA = 4 (102)
................................................
= 400 cm2
Alternative Solution:
f(x) = (25 x20 – 24x10)(x2 – 9x + 3)
= 25x22 – 225x21 + 75x20 – 24x12 + 216 x11
– 72 x10
491. Given f(x) = (25 x20 – 24x10)(x2 – 9x + 3), find f ’(x). f ‘(x) = 550 x21 – 4725 x20 + 1500 x19 – 288 x11 + 2376 x10 – 720 x9
................................................
A. f ‘(x) = 550 x21 – 4725 x20 + 1500 x19 – 288 x11
+ 2376 x10 – 720 x9
B. f ‘(x) = 450 x21 – 4725 x20 + 150 x19 + 288 x11
A. 2019x
Explanation:
Remember that eln u = u.
In the question, u = 2019 x.
Therefore, eln 2019 x = 2019 x
496. Find the measure of the smaller angle formed by the hands of the clock at 11:20.
................................................
A. 130o
493. If the roots of a quadratic equation are and , which of the following could be the
C. 140o
quadratic equation?
A. 63x2 + 22x – 21 = 0
B. 63x2 – 22x – 21 = 0
C. 63x2 + 22x + 21 = 0
D. 63x2 – 22x + 21 = 0
Solution:
Solution:
A = 30h -
x=
9x
Since the angle taken from our formula is a reflex angle and we are looking for the smaller angle, then 360 – 220 =
9x 140.
................................................
(9x + 7) (7x – 3) = 0
63x2 + 49x – 27x – 21 = 0
63x2 + 22x – 21 = 0
................................................
497. How many even 3-digit even numbers can be formed using the digits 7, 6, 5, 4, 3, 2, 1, and 0 if repetition
is not allowed?
A. 150 B. 160 C. 170 D. 180
494. If three more than twice a number is seventeen less than seven times the number, what is the Solution:
number? Number of 3-digit numbers:
A.2 B.3 C.4 D.5 7 x 7 x 6 = 294
Solution: 6 x 6 x 4 = 144
Number of 3-digit odd numbers:
498. There are 50 students in a class. Twenty of them have a laptop. Thirty-two of them have a
smartphone. Seven of them have both a laptop and a smartphone. How many of them have neither a
495. A team is to be made from a group of seven teachers and six scientists. If the team is to be laptop nor a smartphone?
composed two teachers and two scientists, how many different ways can they form a team? A.4 B.5 C.6 D.7
A. 325 B. 315 C. 300 D. 285 Solution:
50–(20+32–7)=50–45=5
na-gamete eh.
C. 11 hrs
Bakit daw nagalit ang mga Pinoy nung namatay si Rizal?
Kasi wala siyang pang-buyback.
Mr. Lazada bought 200 cars. OMG ang yaman ni Mr. Lazada!
................................................
500. What conic figure does the equation x2 + y2 + 10x – 16y = -100 form?
A. Real circle
Nabutas ang isang water tank. Normies: OMG! Sayang ang tubig! Math
major: *computes the rate at which the tank is being emptied*
C. Imaginary circle
Solution:
Use CTS (completing trinomial squares) to convert the equation to its center-radius form. x 2 + y2 + 10x – 16y = -100
x2 + 10x + y2 – 16y = -100
(x2 + 10x + 25) + (y2 – 16y + 64) = -100 + 25 + 64
(x + 5)2 + (y – 8)2 = -11
Solution:
503. The average of 5 different counting numbers is 143. What is the highest possible value that one
of the numbers can have? Let the three numbers 2x, 3x, and 4x so that the ratio will still be 2:3:4.
2x + 3x + 4x = 5,670,000
9x = 5,670,000
A. 706 x = 630,000
Difference: 4x – 2x = 2x
2x = 2(630,000) = 1,260,000
................................................
C. 704
Solution:
506. Rayon and Wena can do a job together in four hours. Working alone, Rayon
The 5 different counting numbers will assume the values of 1, 2, 3, 4, and N. Since the does the job in six hours. How long will it take Wena to do the job alone?
average is 143, the sum is 5(143) or 715. 1+2+3+4+N = 715
10+N=715
N=705
A. 12 hours
C. 10 hours
Solution:
Formula for “working together”: AB/(A+B)
AB/(A+B) = 4; but A = 6
6B/(6+B) = 4
6B=24+4B
2B=24
B=12
C. 2,039,190
Author: Victor A. Tondo Jr., LPT Solution:
Solution:
508. In a certain school, the ratio of boys to girls is 4 is to 9. If there are 260 boys and girls in the school, how
many boys are there?
A. 100
x + 20 + 5 = 2(x + 5) – 5
x + 25 = 2x + 5
25 – 5 = 2x – x
C. 85 20 = x
................................................
Solution:
Let 4x = boys, 9x = girls
4x + 9x = 13x = 260
.: x = 20
Number of boys = 4x = 4(20) = 80 510. What is the minimum value of
................................................
Solution:
A. x = 9 Minimum Value = c – b2/4a
11 – 36/12 or 11 – 3 = 8
................................................
C. x = -9
Solution:
A.
Solution:
................................................
Sum of roots =
................................................
510. Mr. Park is 20 years older than his son now. Five years from now, the Mr. 512. If xy = 17 and x2 + y2 = 135, find x + y.
Park’s age will be 5 less than twice his son’s age. Find the son’s present age.
A. 13
A. 18
C. 12.5249
C. 38 Solution:
x2 + 2xy +y2 = x2 + y2 + 2xy x2 + 2xy +y2 = 133 + 2(17) x2 + 2xy
+y2 = 169
x + y = 13
................................................
513. How many mL of 50% acid solution must be added to 200mL of 10% acid solution to make a 40% acid
solution?
A. 400 mL
C. 600 mL
Solution:
514. Nine consecutive even numbers have a sum of 180. What is the smallest of these even numbers? Let x = lowest even number
x + (x+2) + (x+4) + (x+6) + (x+8) + (x+10)
+ (x+12) + (x+14) + (x+16) = 180
9x + 72 = 180
9x = 108
x = 12
Author: Victor A. Tondo Jr., LPT
................................................
515. If x = 13, which of the following is equal to
200?
517. If A and B are the roots of x2 + 19x + 20, what is AB?
A. 15x + 2
C. x3 – 4x – 2 A. 20
Explanation:
Just substitute x with 13.
A. 15(13) + 2 = 197
B. (13)2 + 2(13) + 5 = 200
C. (13)3 – 4(13) – 2 = 2143
D. (13)2 + (13) + 2 = 184
................................................ C. 3√2
Explanation:
516. For which value of k does 9x2 – kx + 25 have only one root? Since A and B are the roots, then AB pertains to the product of the roots which is given as c/a.
A. 3.75 B. 7.5 C. 15 D. 30 ................................................
Explanation: 518.0.25+0.5+1+2+4+…+1024=____
b2 – 4ac = 0 when there’s only one root.
k2 – 4(9)(25) = 0
k2 = 900
k = 30 A. 2046.75
C. 2048.75
Solution:
You may use the Geometric Series formula which
is ∑
Use the shortcut for the sum of a geometric sequence wherein the ratio is 2 or ½, which is SUM = 2(largest) –
smallest.
In this problem, that’s 2(1024) – 0.25 = 2047.75.
................................................
519. How many terms are there in the sequence
2, 9, 16, 23, …, 345?
A. 40
C. 50
Solution:
An = A1 + (n – 1)d
345 = 2 + (n – 1)(7)
345 – 2 = 7(n – 1)
343 = 7n – 7
350 = 7n
50 = n
................................................
520. If 2x + 3 = 25, then what is (2x + 3)2 – 25?
A. 650 B. 625 C. 600 D. 575
1000 MMR
Solution:
(2x + 3)2 – 25 = 252 – 25 = 625 – 25 = 600
................................................
521. What is 50% of 200% of 2019?
A. 1009.5
Author: Victor A. Tondo Jr., LPT Solution:
C1 V1 + C2 V2 = CRVR
10 (V) + 40 (500) = 30 (V + 500)
C. 4038 10V + 20,000 = 30V + 15,000
20,000 – 15,000 = 30V – 10V
Solution: 5,000 = 20V
250=V
(0.5)(2)(2019) = 2019 ................................................
................................................
522. If 3x = 5 and 2y = 11, what is 12(x – y)? 525. 3log2 3 + 2 log2 5 – log2 7 = ______.
A. -46 B. -45 C. 45 D. 46
Solution:
12(x – y) = 12x – 12y
= 4(3x) – 6(2y)
= 4(5) – 6(11) A. log2
= 20–66
= -46
................................................
523. If two numbers have a product of 267 and the sum of their squares is 250, what is their sum?
C. log2
A. -30√3
Explanation:
C. 7√3
A. 1000 ml
C. (1, 7)
Solution:
y = 3x + 4
- y = 5x – 8
0 = -2x + 12
2x = 12
x=6
y = 3x + 4
= 3(6) + 4
= 22
................................................
527. Find the remainder when
x5 – 3x3 + 5x2 – 7x + 9 is divided by (x – 1).
A.11 B.9 C.7 D.5
Solution:
Use the Remainder Theorem.
14 – 3(13) + 5(12) – 7(1) + 9 = r
1–3+5–7+9=r
5=r
A. 15x2 – 8x – 63
................................................
C. 15x2 + 8x – 63
Solution:
Author: Victor A. Tondo Jr., LPT Solution:
Let x = number of P5 coins .: x+7 = number of P10 coins 5(x) +
10(x+7) = 475
................................................ 5x + 10x + 70 = 475
15x + 70 = 475
15x = 405
x = 27
.: He has twenty-seven P5 coins.
................................................
530. x varies directly as y and inversely as z. If x = 40 when y = 8 and z = 2, what is x when y = 24 and
z = 4?
A. 120 532. When a number is increased by 5, its square also increases by 255. What is this number?
A. 22 B. 23 C. 24 D. 25
Solution:
(x+)2 – x2 = 255
x2 + 10x + 25 – x2 = 255
10x + 25 = 255
Solution: 10x = 230
x = 23
................................................
Solution:
(x + 4)2 = (x – 6)2
531. Pedro has 7 more P10 coins than P5 coins. If he has a total of P475, how many P5 coins does he have? x2 + 8x + 16 = x2 – 12x + 36 8x + 12x = 36 - 16
A. 23 B. 25 C. 27 D. 29 20x = 20
x=1
................................................
x = x + 44
20 ( x = x + 44)
11x = 880
x = 80
................................................
B = price of a hamburger
10C + 6B = 440
................................................
A. 31 B. 37
Solution:
x + 2 = larger number
√x + 2x + 1 = √1764
542. Find
546. Factorize: x3 + 3x2 – 4x – 12 A. (x + 2) (x + 3) (x – 2)
B. (x + 4) (x + 3) (x – 1) C. (x + 2) (x – 3) (x + 2) D. (x + 4)
(x – 3) (x + 1)
Solution:
Factorize by grouping.
x3 + 3x2 – 4x – 12
= (x3 + 3x2) – (4x + 12)
A. = x2 (x + 3) – 4 (x + 3)
= (x2 – 4) (x + 3)
= (x + 2) (x – 2) (x + 3)
................................................
A. 60
C. 2√5
Solution:
Solution:
Let x = first number;
x(x+2) = 1023
x2 + 2x = 1023
x2 + 2x + 1 = 1023 + 1
√ + 2x + 1 = √125
................................................
√1024 x+1 = 32
Therefore x = 31 and x+2 = 33
The numbers are 31 and 33, so their sum is 64.
................................................
549. The product of two consecutive even counting numbers is 3968. Find the smaller number.
A. 42 B. 46 C. 54 D. 62
544. There are 250 pigs and chickens in a farm, all of which are healthy. If there are 720 legs in total, how
many pigs are there?
A. 110
Solution:
C = number of chickens
P+C=250
4P+2C=720
2(P + C = 250)
4P+2C=720
................................................
545. Solve for x given 13x + 17 = 15x – 21.
A. x = 17
C. x = 19
Solution: A. 115 cm
This is similar to our “Working Together”
problem, except instead of adding their times, we
will subtract (since the draining pipe is doing the
opposite of helping).
AB/(A-B) = 10(5)/(10-5) = 50/5 = 10 hrs C. 128 cm
................................................
d=
................................................
Author: Victor A. Tondo Jr., LPT Solution: 555. Find the general term An of the sequence
Seesaw problems call for inverse or indirect 7, 16, 25, 34, 43, 52, 61, …
proportion.
48(160) = 60N
7680 = 60N A. An = 9n – 2
128=N
................................................
C. An = 9n + 2
553. Insert one term between 81 and 169 to make a geometric sequence.
A. 113 B. 117 C. 121 D. 125
Solution:
Solution:
Shortcut for inserting one term is √AB. This is also the formula for the geometric mean.
√81(169) = 9(13) = 117
................................................
An = dn – (d – A1)
554. Find the common difference of an arithmetic sequence whose 21 st term is 1987 and 29th term is = 9n
2019.
= 9n
................................................
A. 2.5
556. Find the remainder when x4 – 4x3 + 3x2 + 5x – 6 is divided by (x – 2).
A.3 B.2 C.1 D.0
Solution:
A. 48; 192
C.
C. 46, 196
Solution:
Solution:
Since the common ratio is 2, then next terms should be 24(2) and 96(2), or 48 and 192.
................................................
562. In a parking lot, there are 65 tricycles and motorcycles. If there are 150 wheels in all, how many
tricycles are there?
n=
A. 18
Solution:
n=
T+M=65
Sum = 3T+2M=150
................................................
563. Which fraction is equivalent to 0.425?
A. 21/50
Sum =
................................................
C. 27/40
Explanation:
558. Factorize: (2x + y) (2x + y + 3) + 2. A. (2x + y + 1) (2x + y + 2)
B. (2x + y + 1) (2x + y + 6) C. (2x + y + 2) (2x + y + 3) D. (2x + y + 0.425 is read as 425 thousandths. In fraction form, that’s or in its simplest form, .
3) (2x + y + 4)
Solution: Alternative Solution:
Let 2x + y = A
(2x + y) (2x + y + 3) + 2 = A (A + 3) + 2
=A2+3A+2
=(A+2)(A+1)
559. The area of a rectangle is (x2 – x – 30). If its length is x + 5, what is its width?
A. x + 4
C. x + 2
Solution:
Just input 0.425 and press your scientific calculator’s S D button.
x2 – x – 30 = (x + 5) W (x + 5) (x – 6) = (x + 5) W
x–6=W
This is a free reviewer. All rights reserved.
1000 MMR
564. What are the zeroes of 6x2 – x – 35?
565. Seven more than four times a number is 79. What is five less than three times the number?
A. 59 B. 49 C. 39 D. 29
Solution:
4x + 7 = 79
A. 4x = 72
x = 18 3x – 5 = 3(18) – 5 = 49
................................................
566. Mr. Park is five times as old as his son. Eight years from now, he will only be thrice as old as his son. How
old is Mr. Park now?
C.
A. 30
Solution: Solution:
3x + 7 = 0
x=
................................................
Author: Victor A. Tondo Jr., LPT Solution:
x=8
................................................
567. Mr. Yu is four times as old as his daughter. Ten years from now, he will only be thrice as old as her. How 4x + 10 = 3(x + 10)
old is his daughter now? 4x + 10 = 3x + 30
A. 10 B. 15 C. 20 D. 25 4x – 3x = 30 – 10
x = 20
................................................
568. The speed of the current of a river is 9 kph. Rayon rows his boat upstream for 8 hours, and
then travels back to his original position by rowing downstream for 4 hours. What is Rayon’s speed on
calm water?
A. 9 kph
C. 27 kph
Solution:
8(x – 9) = 4(x + 9)
8x – 72 = 4x + 36
8x – 4x = 36 + 72
4x = 108
x = 27
................................................
569. Thrice Rayon’s age 16 years ago is equal to his age 16 years from now. How old is Rayon now?
A. 16 B. 24 C. 32 D. 40
Solution:
3(x – 16) = x + 16
3x – 48 = x + 16
3x – x = 16 + 48
2x = 64
x = 32
................................................
570. Divide 8x5 – 4x3 + 2x2 – 3x + 4 by x + 2. A. 8x4 – 16x3 + 28x2 – 54x + 105 r. -206
B. 8x4 + 16x3 + 28x2 – 54x + 105 r. -206 C. 8x4 – 16x3 + 28x2 – 54x – 105 r. 206 D. 8x4 + 16x3
+ 28x2 – 54x – 105 r. 206
This is a free reviewer. All rights reserved.
1000 MMR
Solution:
Solution:
(7C4)(2x)3(3)4 = 35 (8x3) (81)
= 22680x3
x+
................................................
C. 21600x3
Author: Victor A. Tondo Jr., LPT 24 = 6k
573. In the equation 2x + 5y = 50, what is x when y is 8?
A.2 B.3 C.4 D.5
Solution: 4=k
2x + 5(8) = 50 ................................................
2x + 40 = 50
2x = 50 – 40
2x = 10
575. What is the 4th term of (3x – 2)5?
x=5
................................................
A. 360x2
574. X varies directly as Y and inversely as Z. If X is 24 when Y is 30 and Z is 5, find X when Y is 36 and Z is 4. C. -240x2
Solution:
5C3 (3x)2 (-2)3 = 5 (9x2) (-8)
= -360x2
................................................
X = kY/Z
24 = 30k/5
C. k > 45
A=5
A. 10x15y20z10
A. π r2 + 2 π r
C. 32x15y20z10
C. √2
Explanation:
Variables cannot bear an irrational exponent.
................................................
583. Which of the following is irrational?
A. 0.193193193193…
C.
Explanation:
A. 0.193193… is in fraction form
B. is already a fraction
C. √1023 cannot be expressed as a fraction
D. √2401 is equal to 49
................................................
584. If x + y = 17 and x2 + y2 = 135, find xy.
A. 78 B. 77 C. 76 D. 75
Solution:
585. If H and K are the roots of 5x2 – 8x + 9, find H+K–HK.
A. -2/5 B. -1/5 C. 0 D. 1/5
Solution:
H + K = sum of roots = 8/5
HK = product of roots = 9/5
H+K–HK=8/5–9/5=-1/5
7(2x + 3y = -8)
................................................
3(5x – 7y = 67)
Solution:
H+K=-4/3
HK = 5/3
(H + K)2 = 16/9
H2 + K2 + 2HK = 16/9
H2 + K2 + 2(5/3) = 16/9 ................................................
H2 + K2 = 16/9 – 10/3
H2 + K2 = -14/9
................................................
589. A farmer has 15 goats, 23 pigs, and a few chickens in his farm. If he counted a total
of 200 legs in his farm (excluding his, of course), how many chickens does he
587. Factorize 9x2 + 24xy + 16y2 – 81z2. A. (3x + 4y + 9z) (3x + 4y – 9z) have?
B. (3x – 4y + 9z) (3x + 4y – 9z) C. (3x + 4y + 9z) (3x – 4y – 9z) D. (3x – 4y + A. 96 B. 72 C. 48 D. 24
9z) (3x – 4y – 9z)
Solution: Solution:
9x2 + 24xy + 16y2 – 81z2
=
=
(3x + 4y)2 – (9z)2
(3x + 4y + 9z) (3x + 4y – 9z) 15(4) + 23(4) + n(2) = 200 60 + 92 + 2n = 200
................................................ 2n = 200 – 152
2n = 48
n = 24
................................................
√
590. Rationalize
A.
................................................
A. 15x + 9
C.
C.
Solution:
This is a free reviewer. All rights reserved.
1000 MMR
Solution:
3x + 2 = 4y
= y
5y – 3 = z
5( )–3=z
= z
= z Author: Victor A. Tondo Jr., LPT Solution:
................................................ A = P (1 + tr)
A = 1,600,000 [1 + 4(0.025)]
A = 1,600,000 (1 + 0.1)
A = 1,600,000 (1.1)
A = 1,760,000
................................................
A. 343
595. Solve for x: 45x+3 = 82x+6
Solution:
72x + 3 = B
(72)x (73) = B A. 2
49x (343) = B
(343) 49x = B
B ÷ 49x = (343) 49x ÷ 49x = 343
Solution:
................................................
times
596. Given
expressions is equal to x?
A.
times
Solution:
42x3
42x3
................................................
594. Rayon invested P1,600,000.00 in a bank that offers 2.5% interest per annum. How much will his
account hold after 4 years? 2cxy = 3abz
A. P160,000
C. P1,460,000 x=
................................................
A. x2 + 2x + 10
C. x2 – 2x + 10
................................................
A. 9
Express each side as a power of a common number before solving for x. A. 378
272x–8 = 9x+10
(33)2x–8 = (32)x+10 Apply laws of exponents.
26x–24 = 22x+20
6x – 24 = 2x + 20 C. 412
6x – 2x = 20 + 24
................................................
4x = 44
x = 11 Solution:
599. If 25x – 49y = 2019, what is 49y – 25x? 28C2 = 378
................................................
A. 2018
603. The vertex angle of an isosceles triangle is 70°. What is the measure of one of the base angles?
A. 45° B. 50° C. 55° D. 60°
Solution:
C. (180 – 70)/2 = 110/2 = 55
................................................
Explanation:
49y – 25y = (-1)(25x – 49y)
= (-1)(2019)
= -2019
................................................
604. A shoebox measures 20 inches by 15 inches by 9 inches. What is its volume in cubic inches?
600. If x + y = 90 and x2 + y2 = 2020, find xy.
A. 900
A. 3025
C. 1650
C. 3035
Solution:
Solution: V=LxWxH
(x + y)2 = x2 + 2xy + y2 = 20 x 15 x 9 = 2700
................................................
902 = x2 + y2 + 2xy
8100 = 2020 + 2xy
8100 – 2020 = 2xy
6080 = 2xy
3040 = xy
................................................
605. Which of these has the longest perimeter? A. A regular pentagon with sides 21 cm long B. A rectangle
29 cm long and 24 cm wide
C. An equilateral triangle with sides 36 cm long D. A square whose area is 1024 cm2
601. Which of the following is ALWAYS true? A. Vertical pairs of angles are supplementary. B. Vertical Solution:
pairs of angles are congruent. A.P=5(21)=105
C. Linear pairs of angles are congruent. B.P=2(29+24)=106
D. Linear pairs of angles are complementary. C.P=3(36)=108
Explanation: D. S = 32, so P = 4 (32) = 128
................................................
A. 7000 cm2
611. Two parallel lines are cut by a transversal, forming R and V. If the two angles are corresponding angles, what is
the measure of R
A. 35o
and m B = 5x + 40, what is the value of x?
C. 160o
A. 17.5
Solution: Explanation:
Since the two angles form a linear pair, then they are supplementary. Corresponding angles formed by two parallel lines cut by a transversal are congruent.
3x + 5x + 40 = 180 ................................................
8x + 40 = 180
8x = 140
x = 17.5
................................................
612. If the sum of the supplement and the complement of an angle is 100 degrees, what is the angle?
609. A. 65o B. 70o C. 75o D. 85o
Solution:
(90 – x) + (180 – x) = 100
parallelogram. If m 270 – 2x = 100
270 – 100 = 2x
170 = 2x
85 = x
................................................
A. 40o
Explanation:
613. It is the perpendicular bisector of a regular polygon’s side, passing through the center.
Opposite angles of a parallelogram are congruent.
A. side
C. asymptote
Explanation:
610. A and C are consecutive angles in a parallelogram. If m A = 60 o, what is m C?
A. 40o B. 50o C. 150o D. 120o
The apothem is only for regular polygons. It is the perpendicular bisector of one of its sides, passing through
the center.
A diagonal is a line segment formed by connecting two non-consecutive vertices of a polygon.
Solution:
614. In the figure,
34
17
................................................
619. The perimeter of a rectangle is 84. If its length is 29 cm, find its area.
A. 377 cm2
C. 2436 cm2
A. 50o
Solution:
Explanation:
P=2(L+W)
AFE and 84
A. 45
Solution:
D=
................................................
Explanation:
̂ ̂
BAC is a central angle measuring 40o. That means = 40o. Since BDC is an inscribed angle intercepting , then its measure is half of
617. Find the area of a square whose perimeter is 480 cm. ̂ , or 20o.
A. 14400 cm2
C. 57600 cm2
Solution:
480
120
1000 MMR
621. Find the measure of E using the figure of parallelogram WENA below.
C. 73
625. Which of the following has its incenter, circumcenter, centroid, and orthocenter in just one point?
Solution:
623. The diagonal of a rectangular prism is 17 cm long. If it is 9 cm thick and 12 cm long, how wide is it? 626. In triangles, this line segment is drawn from midpoint of one side to its opposite vertex.
A. Median
A. 15 cm C. Bisector
Explanation:
................................................
627. The radius of a cone measures 12 cm and its height is 15 cm. Find its volume.
A. 240 π cm3
C. 480 π cm3
C. Incenter
Explanation:
629. The radius of a cylinder measures 9 cm and its height is 15 cm. Find its volume.
52 + h2 = 132
25 + h2 = 169
h2 = 144
h = 12
A. 1215 π cm3
C. 540 π cm3
Area =
Solution:
Area =
V = πr2h Area = 25(12) = 300
................................................
= π (81) (15)
= 1215 π
................................................
631. Find the area of an isosceles trapezoid given bases measuring 20 cm and 30 cm, with each of the
congruent slants measuring 13 cm. Solution:
A. 250 cm2
A.52+72<92
C. 300 cm2
B.62+82=102
C.62+92<122
D.72+92>112
................................................
633. A prism has a right triangle as its base. The two legs of the right triangular base are 6 cm
and C. 56 cm2
8 cm long. If the prism is 2 cm thick, find its lateral surface area.
Solution:
The hypotenuse of the base is 10 cm.
A. 24 cm2 Lateral surface area = 2(6 + 8 + 10) = 48 cm2
C. 300 cm2
C. 1575 cm3
Solution: Solution:
V= LxWxH A=½D1D2
A = ½ (600) = 300
................................................
V = (21)(10)(15)
V = 1050
................................................
A. 312 π cm2
C. 68 cm
C. 936 π cm2
Solution:
SA = 2πr2 + 2πrh
SA = 2π (144) + 2π(12)(40)
SA = 288π + 960π Solution:
SA = 1248π
................................................
636. In the figure, AC is a diagonal of rhombus
ABCD. If m B = 118o, what is m CAD?
A. 124o B. 118o C. 62o D. 31o
Solution:
By Pythagorean Theorem, the hypotenuse of each right triangle is 17 cm. That means each
Since ABCD is a rhombus, then AC bisects BAD, and BAD and B are supplementary. side of the rhombus is 17 cm, and its perimeter is 4 x 17 or 68 cm.
................................................
m
B + m BAD = 180 118 + m BAD = 180 m
BAD = 62
m CAD = ½ (62) = 31
................................................
639. Find the approximate distance around a semicircular park with radius 100m.
A. ~1028 m B. ~771 m
C. ~514 m D. ~257 m
Solution:
Diameter = 2r = 200
Semicircle = r = 100π ~ 314
Perimeter = ~514
................................................
640. Find the equation of the line perpendicular to 7x + 4y = 12, passing through (-2, 9).
A. 4x – 7y = -71
C. 4x – 7y = 71
Solution:
The perpendicular line for Ax + By = C is given
as Bx – Ay = B(xp) – A(yp).
4x – 7y = 4(-2) – 7(-9)
4x – 7y = -71
This is a free reviewer. All rights reserved.
A. 7x + 9y = 1
C. 7x – 9y = 1
Solution:
The perpendicular line for Ax + By = C is given
as Bx – Ay = B(xp) – A(yp).
-7x – 9y = -7(4) – 9(-3)
-7x – 9y = -7(4) – 9(-3)
-7x – 9y = -1
But since the new A is negative, then we will change all signs of the new equation, giving us 7x + 9y = 1.
................................................
642. A rectangle is drawn with dimensions 28 cm by 42 cm. A larger rectangle is drawn by adding
5 cm margins from each side of the original rectangle. What is the area of the larger rectangle?
643. Find the length of the diagonals of a rectangular prism 6 cm thick, 15 cm long, and 10 cm wide.
645. What is the slope of the line defined by the equation 3x + 5y = 11?
A.
A. 17√3
C. 19 cm
Solution:
Solution:
3x + 5y = 11
5y = -3x + 11
D=√
D=√15 +10 +6
D=√225+100+36
D = √361
D=19 y=
................................................
A. B. -2 C. D. 15
Solution: 4x + 8 = 5x – 7
8 + 7 = 5x – 4x
15 = x
2(2x + 4) = 5x – 7
This is a free reviewer. All rights reserved.
A. 119o
Solution:
Since
A. 9720 π cm2
same side of the transversal, then they are
supplementary.
Solution:
648. Find the sum of all interior angles of a regular 15-sided polygon.
A. 5400o
A=
C. 2340o
A= (900 π)
Solution: A = 270 π
................................................
180 (n – 2) = SIA
180 (15 – 2) = SIA
2340 = SIA
................................................
̂ ̂
651. In the figure, = 120o and m E = 28o. Find .
................................................
A. 289,000 π
A. 864 cm C. 51,000 π
C. 144 cm
C. y = ½ x + 9
Solution:
653. Find the center of a circle given one of its longest chords has endpoints at M(-11,8) and N(23,-24). Use the two-point form.
A. (12, -16) y – y1 =
C. (-17,16)
Solution: y–5=
The longest chord of a circle is the diameter and its midpoint is the center.
Midpoint Formula: ( , )
y–5=
y – 5 = 2 (x – 7)
y – 5 = 2x – 14
y = 2x – 9
Midpoint: ( ................................................
656. Find the equation of the line passing through (2, -7) with a slope of -3.
Midpoint: (6,
................................................ A. y = -3x – 1
C. y = -3x – 2
654. Find the radius of a circle if its diameter has endpoints at M(3,5) and N(-12,13).
A. 8 B. 8.5 C. 9 D. 9.5
Solution:
The diameter’s midpoint is the center.
Center: ( , )
Center: ( 4.5,9) Solution:
Radius is the distance between the center and any of its endpoints.
R=√( 4.5 3) +(9 5)
y = mx + b
-7 = -3(2) + b
A. 784 cm2
-1 =b
R = √( 7.5) +4
R= √56.25 + 16
R= √72.25
R=8.5
Better Solution:
D=√( 12 3) + (13 5)
D=√225+64
A.
C.
................................................
Solution:
Let’s rebuild the pool as a figure that is left by removing a triangular prism (gray figure) from a rectangular prism
(whole figure).
C. (3, -6)
Author: Victor A. Tondo Jr., LPT 662. In the figure, ACB is formed by the tangents AC
and BC. If m ACB = 34o, what is the measure of the minor intercepted arc?
Solution:
A (-13, -16)
x=4
C. (-13, 16)
y = 5+
Solution:
y = 5+
7=
14 y = 5+
13
y = 5 + (-10) y = -5
................................................
664. Point O is two-thirds the way from A(-4,5) to B(8,-10). Find the coordinates of point O.
This is a free reviewer. All rights reserved.
1000 MMR
665. H and K form a vertical pair. If m H =
4x + 20 and m K = 6x – 10, find x.
A. 13 B. 15 C. 17 D. 19
Solution:
4x + 20 = 6x – 10
20 + 10 = 6x – 4x
30 = 2x
15 = x
......... ............
......... ............
A. 13 B. 15
Solution:
4x + 20 + 6x – 10 = 180
10x + 10 = 180
10x = 170
x = 17
................................................
668. Point C is two-fifths the way from M(-3,4) to
A. (0, 0)
C. (1, -2)
Solution:
x = -3+ (7 – (-3))
x = -3+ (10)
x = -3 + 4
x=1
................................................
A. B.
When two parallel lines are cut by a transversal, corresponding and alternate
C. 2304π – 960 cm2 interior/exterior angles are congruent. Vertical pairs of angles are also congruent.
Solution: W2 + 242 = 252
However, interior/exterior angles on the same side of the
transversal are supplementary.
................................................
The rectangle’s diagonal is the diameter of the circle. By Pythagorean Theorem, the diameter is W2 + 576 = 625 676. A rectangle is 24 cm long. If its
52 cm, meaning the radius is 26 cm. Therefore, the area of the entire circle is 676 π cm 2. diagonal is
The area of the rectangle is 20 x 48 or 960 cm2. A. 49
25 cm, what is its perimeter in cm?
B. 62 C. 64 D. 98
W2 = 625 – 576 W2=49
That means the area of the shaded figure is equal to 676π – 960 cm 2.
................................................
W=7
674. What is the area of the red trapezium in the given figure?
677. Square FACE is drawn such that one of its sides AC is the diagonal of a rectangle, ABCD. If AB = 20 and BC = 43, find the
area of FACE.
A. 23 sq. units B
Solution:
The entire figure is 8 x 6, or 48 sq. units.
The white triangle in the upper left area is 3 x 4 units, giving it an area of 6 sq. units.
The white triangle in the upper right area is 5 x 3 units, giving it an area of 7.5 sq. units.
The white triangle in the lower left area is 5 x 2 units, giving it an area of 5 sq. units.
This is a free reviewer. All rights reserved.
1000 MMR
(AC)2 = 2249
................................................
Solution:
(AC)2 = 202 + 432
(AC)2 = 400 + 1849
678.
A. 130o
Solution:
If the vertex angle of an isosceles triangle is 50 o, what is the measure of each base angle?
B. 80o C. 65o D. 25o Author:
681.
Victor A. Tondo Jr., LPT
Find the length of the intercepted arc of a central angle measuring 60 given that the radius of the
o
50 + x + x = 180 circle is 12 cm.
50 + 2x = 180
2x = 130
x = 65
................................................ A. 4 π cm
C. 8 π cm
679. Which of the following is a square? A. VCTR whose sides measure 20 cm each B. LUVS
whose angles are 90o each
C. RWNA whose diagonals are 50 cm each D. AYIE whose congruent diagonals perpendicularly
bisect each other
Explanation: Solution:
A. VCTR is a rhombus
B. LUVS is a rectangle
C. RWNA is a rectangle
D. Only squares have congruent diagonals that are perpendicular bisectors of each other.
................................................
680. The following are measures of sides of four triangles. Which of them are taken from an obtuse
................................................
triangle? 682. Which of the following is false?
A. Radii of the same circle are always congruent.
B. Any two right angles are congruent.
C. An inscribed right angle always intercepts a semicircle.
D. There are infinitely many lines that may be drawn parallel to a given line passing through a given point.
A. 20, 20, 29
Explanation:
C. 20, 22, 28
There can only be one line that may be drawn parallel to a given line passing through a given point.
Explanation: ................................................
A. 24 cm2
B. 202 + 212 = 292
C. 48 √13
Solution:
S3 = 343
S=7
684. Find the volume of a rectangular plank of wood that is half-inch thick, six inches wide, and
6 feet long.
A. 18 cubic inches
A. 18o B. 36o C. 54o D. 72o 687. Find the surface area of a ball whose radius is 24 cm.
Solution: A. 576 π cm2
Most people would solve for m DBA by doing this long solution: C. 2304 π cm2
Measure of arc BC = 2 x 36 = 72
Since chord AC passes through center O, then AC is a diameter, and arc CBA is a semicircle. That means arc AB is 180
– 72 = 108. Solution:
Since chord BD passes through center O, then BD is a diameter too, making arc BAD a semicircle. That means arc AD SA = 4 π r2
SA = 4 π (242)
= 180 – 108 = 72. SA = 2304 π
Finally, since DBA intercepts arc AD, then m DBA = ½ x 72 = 36. ................................................
688. Give the converse of the conditional statement “If two angles are vertical angles, then they are
That’s a long solution. The best way to deal with this is to remember that OA and OB are both congruent.”
radii of the same circle, making them congruent. That means △OAB is an isosceles triangle, with A. “If they are are congruent, then two angles are vertical angles.”
a base angle at OAB (same angle as CAB). Since the base angles of an isosceles triangle are B. “If two angles not vertical angles, then they are not congruent.”
congruent, then OBA (same angle as DBA) is the same measure as OAB, or 36 o. C. “If they are not congruent, then two angles are not vertical angles.”
................................................ D. “If two angles are linear angles, then they are supplementary.”
Explanation:
686. What is the measure of each interior angle of a regular nonagon? The converse of a conditional is formed by interchanging the if-statement (hypothesis) and the then-statement
(conclusion).
................................................
A. 135o
C. 144o
A. “If they are are congruent, then two angles are vertical angles.” 690. In which geometry do two distinct lines intersect in two points?
B. “If two angles not vertical angles, then they are not congruent.” A. Euclidean Geometry B. Hyperbolic
C. “If they are not congruent, then two angles are not vertical angles.” Geometry C. Elliptic Geometry D. Plane
D. “If two angles are linear angles, then they are supplementary.” Geometry
Explanation: Explanation:
In Elliptic Geometry, two distinct lines intersect in two points. In both Euclidean and
Author: Victor A. Tondo Jr., LPT
Hyperbolic Geometry, two distinct lines will intersect at only one point. 692. In which geometry are the summit angles of a Saccheri quadrilateral obtuse angles?
................................................ A. Euclidean Geometry B. Hyperbolic Geometry
C. Elliptic Geometry D. Plane Geometry
Explanation:
691. In which geometry can the sum of the measures of the angles of a triangle be less than Euclidean: right angles
Hyperbolic: acute angles
180 degrees? Elliptic: obtuse angles
A. Euclidean Geometry B. Hyperbolic Geometry ................................................
694. Each side of a regular hexagon is 256 cm long. Find the distance around it.
A. 1536 cm
C. 1024 cm
Solution:
P=nS
P = 6 (256)
P = 1536
................................................
695. What can be said about these statements? i. Any rectangle has two congruent
diagonals. ii. Any rhombus has two perpendicular diagonals.
A. Only the first statement is true. B. Only the second statement is true. C. Both
statements are true.
D. Both statements are false.
................................................
P=4S
................................................
699. Find the altitude to the hypotenuse of a right triangle whose legs are 60 cm and 80 cm.
Explanation: Solution:
AB = AE and AD = AC since A is the midpoint of the segments and . Their included angles, Solve for the hypotenuse first using Pythagorean Theorem.
DAE and CAB, are also congruent because vertical angles are congruent. 602 + 802 = H2
3600 + 6400 = H2
10000 = H2
When two sides and an included angle of a triangle are congruent to two sides and an included Then use the following formula:
100=H
angle of another triangle, then the two triangles are congruent by SAS Congruence Postulate.
................................................
C. 169 π cm
Solution:
C=2πr Alt to Hyp =
C = 2 π (13) = 26 π
................................................
................................................
698. Find the area of a square whose perimeter is 280 cm. 700. This is formed by two rays with a common end point.
A. 1120 cm2
A. polygon
C. 19600cm2
C. line
................................................
701. Convert
A. 210o
C. 630o
Solution:
rad x
................................................
A. 32o
Solution:
Since 212o is in QIII, then 212o – 180o = 32o
This is a free reviewer. All rights reserved.
1000 MMR A. sin 40o
703. A negative sine and a positive cosine are properties of angles in which quadrant?
A. QI B. QII C. QIII D. QIV
Explanation: C. csc 40o
Use the CAST mnemonic. See item #164.
................................................ Explanation:
704. Which of the following equal to sec 50o?
Apply the cofunctions. The COfunction for secant is COsecant. The COmplement of 50 o is 40o.
Author: Victor A. Tondo Jr., LPT
................................................
705. Which of the ff. is coterminal with 2019o?
A. 219o
50√3 = x√3
C. 199o
Solution:
50 = x
................................................
2019/360 = 5.608333… Grab 5 only 2019 – 5(360) = 219 707. The longest side in a 30-60-90 triangle is
................................................
706. The angle of elevation to the top of a
480 cm. How long is the shortest side?
building is 30o. If the observer is 50√3 meters away from the building, how tall is the building? A. 120 cm
C. 240 cm
A. 25 m Solution:
2x = 480
x = 240
................................................
C. 50 m
708. If the hypotenuse of a 45-45-90 triangle is
50 cm long, how long is a leg?
Solution:
A. 25 cm
C. 25√3
tan 30 =
Solution:
x√2
x = 25√2
................................................
50√3 (√3) = 3x
709. Which of the ff. is coterminal with 143o?
150 = 3x
50 = x
Alternative Solution: A. 2403o
Use the 30-60-90 map. (You have to know this by heart.) C. 2103o
Solution:
A. 2403 – 143 = 2260 (not divisible by 360)
B. 2303 – 143 = 2160 (divisible by 360)
C. 2103 – 143 = 2060 (not divisible by 360)
D. 2003 – 143 = 1960 (not divisible by 360)
Remember:
Two angle measures are coterminal if and only if their difference a multiple of 360.
................................................
A. 180o
C. 163.625o
Author: Victor A. Tondo Jr., LPT Explanation:
Solution: Refer to this mnemonic hexagon. Any trigonometric function is equal to (katabi) divided by
A = |30H – 5.5M| (susunod sa katabi).
A = |30(5) – 5.5(55)|
That means tan = sin ÷ cos or tan = sec ÷ csc, sin = cos ÷ cot or sin = tan ÷ sec, etc.
A = |150 – 302.5|
A = 152.5
................................................
711. What is the measure of the smaller angle formed by the hands of the clock at 3:50?
A. 185o
C. 170o
Solution:
................................................
A = |30H – 5.5M|
A = |30(3) – 5.5(50)|
A = |90 – 275|
A=185
However, 185o is a reflex angle. The smaller angle is 360 – 185 or 175 o.
714. If sin x = 0.936, which of the following could be cos x?
A. 0.064
C. 0.016
................................................ Solution:
712. Which of the following is false?
A. tan θ = B. csc θ =
sin2 x + cos2 x = 1
cos2 x = 1 – 0.876096
cos2 x = 0.123904
cos x = ±0.352
Alternative Solution:
Input cos (sin-1 (0.936)) in your calculator.
................................................
715. A man stands 20 meters away from a building. If the angle of elevation to the top of the building is 60 o,
how tall is the building?
A. 20√2
C. 40 m
D. sin θ =
Plot your entries in the 30-60-90 triangle and
solve. See item #706.
................................................
716. If csc x =
A.
1000 MMR
Explanation:
sin x is the reciprocal of csc x.
................................................
A.
A.
Solution:
Solution:
Since sec x = , then cos x = .
sin2 x + cos2 x = 1
sin2 x + ( )2 = 1
sin2 x +
160o x
................................................
=1
sin2 x = 1 –
sin2 x =
sin x = 721. What is the reference angle of 156o?
................................................
A. 14o
Refer to the mnemonic hexagon in #712. Kahit aling trig function ay product ng dalawang trig functions na
katabi niya.
................................................
722. In which quadrants is the sine function negative?
719. Which of the ff. is coterminal with 63o?
A. QI and QII
A. 7503o
A. 19o
Explanation:
The coterminal angle for 2019o is 219o (see item #705). Since 219o is in QIII, then its reference angle is 219 – 180 or
39o.
................................................
724. A sniper is on top of a 20-meter cliff. He spots a target at an angle of depression of 30 o. How far must
his bullet travel to hit the target?
A. 20 meters
C. 20 √3
Solution:
.: 2x = 40 m
x = 20 m
This is a free reviewer. All rights reserved.
1000 MMR
725. A meter stick leans to a wall and reaches a point 50 cm away from the wall. x sin 45
What is the measure of the angle formed by the meter stick and the wall? x=
A. 30o B. 45o C. 60o D. 75o x = 500√2
Solution:
................................................
=
727. Which of the following can
sin x = 0.5 be used to solve for x in the figure?
x = arcsin 0.5 A. sin
x = 30o B. cos
................................................ C. arcsin
D. arccos
726. A tight rope connects the top of a 500 cm pole to the ground. If the angle formed Explanation:
by the ground and the tight rope is 45o, how long is the tight rope? is the ratio of the adjacent side to the
hypotenuse. That’s the cosine value for x. Since
the angle x is missing, then we will use arccos .
A. 500 cm
C. 500√3
Solution:
sin 45 =
Author: Victor A. Tondo Jr., LPT
................................................
C. 2
A. 2019
Solution:
2 sin 30 – 4 cos 60 + 3 tan 45
= 2 (0.5) – 4 (0.5) + 3 (1) C. 0
= 1–2+3
=
................................................
2
Explanation:
729. Which of the following is true?
A. sin x = sin (-x) B. cos x = cos (90 – x)
C. –sin x = sin (-x) D. tan x = (sin x)(cos x) The limit of a constant function is the constant itself.
Explanation: ................................................
A/C: –sin x = sin (-x)
B: cos x = sin (90 – x)
D. tan x = (sin x) ÷ (cos x)
................................................
1000 MMR Author: Victor A. Tondo Jr., LPT 737. Give the domain of f(x) =
A. 3
Solution:
Substitute x with 3.
33 – 5(32) = 27 – 45 = -18 C. x
................................................
Explanation:
The given function is a rational function. It will be undefined when the denominator is 0. Therefore, 3x – 6 0, or
734. Evaluate: lim x 2.
................................................
A. LDNE
738. Give the domain of f(x) =
Solution: A. x | x
lim
C. x > 0
Explanation:
= lim The given function is a radical function, more specifically a square root function. If the radicand 2018x –
2019 is negative, f(x) becomes imaginary. That means 2018x – 2019 must be greater than or equal to 0.
= lim (
2018x – 2019
=4
................................................
Retain only the highest term of the numerator and the denominator.
C. x > -81
Explanation:
lim The given function has no denominator and no radicand. Therefore f(x) exists for all real values of x.
................................................
740. Give the range of f(x) = 2020.
A. y | y
lim
C. y = 2020
Since x approaches + , and the function is 2x2, then the limit is + . Explanation:
................................................
736. Evaluate: lim The given function is a constant function. That means y = 2020 for all values of x.
................................................
A. + B. - C. LDNE D.
Explanation:
Retain only the highest term of the numerator and the denominator.
lim lim
lim
................................................
This is a free reviewer. All rights reserved.
1000 MMR
A. y | y
C. y 2
Solution:
y=
3xy – 6y = 2x + 3
3xy – 2x = 6y + 3
x(3y – 2) = 6y + 3
x=
3y – 2 0
3y 2
Shortcut:
Proof:
6x + 9
9+12
21 0
................................................
742. Give the range of f(x) = 20x – 20.
A. y | y
C. y = 20
Explanation:
................................................
A. y | y
C. y 2019
Explanation:
The function has an odd degree (125). All odd-
degree functions have a range of y | y .
This is a free reviewer. All rights reserved.
1000 MMR
746. Give the range of f(x) = 2019x – 2019.
Author: Victor A. Tondo Jr., LPT
751. Find the first derivative: f(x) = (2x + 3)5.
A. f ’(x)= 5 (2x + 3)4 B. f ’(x)= 10 (2x + 3)4 C. f
A. y | y ’(x)= 15 (2x + 3)4 D. f ’(x)= 20 (2x + 3)4
Solution:
C. y = 2019
Explanation: Let u = 2x + 3 so we can rewrite f(x) = u5. Since du = 2, and f ’(x)= 5u4 du, then by substitution,
All linear functions have a range of y | y . f ’(x)= 5u4 du
................................................ f ’(x)= 5(2x + 3)4 (2)
752. Find f ‘(x) given f(x) = (3x2 – 2x + 1 )7. A. f ’(x)= (42x – 14) (3x2 – 2x + 1)6
A. y | y B. f ’(x)= (42x – 28) (3x2 – 2x + 1)6 C. f ’(x)= (42x + 14) (3x2 – 2x + 1)6 D. f ’(x)= (42x
+ 28) (3x2 – 2x + 1)6
Solution:
Let u = 3x2 – 2x + 1 so we can rewrite f(x) = u7.
Since du = 6x – 2, and f ’(x)= 7u6 du, then by
substitution,
f ’(x)= 7u6 du
f ’(x)= 7(3x2 – 2x + 1)6 (6x – 2) f ’(x)= (42x – 14) (3x2 – 2x + 1)6
................................................
C. y
753. Find the maximum area that can be enclosed by a rectangle given its perimeter should only
be 202 meters.
Explanation:
(See item #741.)
................................................
A. 2550 m2
748. Give the domain of f(x) =
C. 2550.5 m2
Solution:
A. x | x
Max area for such a question is taken by making the rectangle a square.
P=4S
4S = 202
S = 50.5
A=S2
S2 = 50.52
C. x 2 50.52 = 2550.25
Explanation:
The given function is a rational function. It will be undefined when the denominator is 0. Therefore, 3x + 9 0, or
x -3.
................................................
749. What is 0! equal to?
A. 0
C. undefined
................................................
C. y 2019
Explanation:
function f(x) = x2 – 8x + 7.
A. -10 B. -9
Solution:
f ‘(x) = 2x – 8
f(4) = 42 – 8(4) + 7
f(4) = 16 – 32 + 7
f(4) = -9
Shortcut:
C– =7– =7–16=-9
................................................
A. 37 B. 48
Solution:
Find f ’(2).
f ’(x) = 15x2 – 4x + 7
f ’(2) = 59
................................................
A. -15 B. 0
Solution:
Find f ’(1).
f ’(x) = 12x2 + 10x – 7
f ’(1) = 15
A. y = 2x Explanation:
C. {3, 8}
A. no elements ................................................
C. all elements
A. since Explanation:
find X
C.A–B
A. {1, 4}
C. {2, 3}
Explanation:
X Y refers to the common elements of X and Y.
A. A’
C. (A
Explanation:
The area that is not shaded is A B. The shaded region is its complement, (A B)’.
This is a free reviewer. All rights reserved.
1000 MMR
775. In a class of 50, there are 20 students who excel in Math, 24 who excel in
Science, and 4 who excel in both Math and Science. How many of them excel in
neither?
A.2 B.4 C.7 D.10
Solution A:
50–(20–4)–(24–4)–4=10 Author: Victor A. Tondo Jr., LPT Solution:
Solution B: 7 x 6 x 5 x 4 = 840
................................................
50–(20+24–4)=10
................................................
780. Using the digits 0, 1, 2, 3, 4, 5, and 6 without repetition, how many 3-digit numbers can be made?
776. There are 50 students in a class. 39 of them have a cellphone, 17 of them have a A. 360 B. 180 C. 90 D. 45
laptop, and 5 of them have neither. How many have both a cellphone and a laptop? Solution:
A.6 B.7 C.9 D.11
6 x 6 x 5 = 180
Solution: ................................................
(39+17)–(50–5)=56–45=11
................................................
781. Using the digits 0, 1, 2, 3, 4, 5, 6, and 7 without repetition, how many 4-digit numbers can be made?
777. Rayon has 10 shirts, 5 pairs of pants, and 4 pairs of shoes. How many ways can he pick what to wear for A. 1680 B. 1470 C. 840 D. 735
today?
A. 19 B. 20 C. 190 D. 200
Solution:
10 x 5 x 4 = 200
................................................
Solution:
7 x 7 x 6 x 5 = 1470
................................................
778. Using the digits 1, 2, 3, 4, 5, and 6 without repetition, how many 3-digit numbers can be made?
A. 480 B. 240 C. 120 D. 60 782. Using the digits 0, 1, 2, 3, 4, 5, 6, and 7 without repetition, how many 3-digit numbers can be made?
A. 672 B. 588 C. 336 D. 294
Solution:
Solution: 7 x 7 x 6 = 294
................................................
6 x 5 x 4 = 120
................................................
783. Using the digits 0, 1, 2, 3, 4, 5, and 6 without repetition, how many odd 3-digit numbers can be
779. Using the digits 1, 2, 3, 4, 5, 6, and 7 without repetition, how many 4-digit numbers can be made? made?
A. 1680 B. 840 C. 420 D. 210 A. 180 B. 150 C. 90 D. 75
Solution:
5 x 5 x 3 = 75
................................................
784. Using the digits 0, 1, 2, 3, 4, 5, 6, and 7 without repetition, how many odd 4-digit numbers
can be made?
A. 1470 B. 1440 C. 735 D. 720
786. Using the digits 0, 1, 2, 3, 4, 5, 6, and 7 without repetition, how many even 3-digit numbers Author: Victor A. Tondo Jr., LPT
can be made?
A. 162 B. 156 C. 150 D. 144
Solution:
789. Evaluate: log6 8 + 3 log6 3.
When asked about even n-digit numbers tapos may 0 sa usable digits, do this:
A. 24
3-digit #s:
Solution:
log6 8 + 3 log6 3
= log6 8 + log6 33
odd 3-digit #s: = log6 8 + log6 27
= log6 (8x27)
= log6 216
= 3
................................................
even 3-digit #s:
................................................
791. There are 10 different beads and a locking mechanism to be used in making a bracelet. How many
different bracelet patterns can be made?
Solution:
A. 3,628,800
C. 1,814,400
Solution:
odd 4-digit #s:
n = 11 here
(
since the locking mechanism
)!
is considered
!
as another unit
The formula is . Thus, there are or
1,814,400 different patterns.
................................................
even
................................................
792. There are six different Math books, four different Science books, and two
788. Using the digits 0, 1, 2, 3, 4, 5, and 6 without repetition, how many even 3-digit numbers can be made? different English books to be arranged on a shelf. How many ways can this be done?
A. 12C3 B. 12! C. 48 D. 3!
A. 120 B. 105 C. 90 D. 75 Explanation:
Solution:
There are 12 different books to be arranged in a linear fashion. There are 12! ways to do this.
3-digit #s:
Solution:
A. 479,001,600
5C2 x 4C1 x 6C2 = 10 x 4 x 15 = 600
................................................
797. If today is a Sunday, what day is 125 days
C. 34560
There are 3 x 2 x 1 or 3! ways to arrange the subjects Math, Science, and English. There
are 6! ways to arrange the Math books, 4! ways for Science, and 2! ways for English. A. Friday
3! x 6! x 4! x 2! = 6 x 720 x 24 x 2 = 207360
................................................
C. Sunday
Explanation:
794. There are six people to be seated on a bench for a picture. A certain couple, Vic
and Rowena, are to be seated next to each other. How many ways can this be done? Exactly 18 weeks or 126 days from now would be Sunday again, therefore, 125 days from now would be
A. 120 B. 240 C. 360 D. 720 Saturday.
Explanation: ................................................
Let Vic and Rowena take one unit, and then add the other four people to make five units. Since
Vic and Rowena can be arranged in 2! ways and the five units can be arranged in 5! ways, then 798. If today is a Monday, what day is 200 days from now?
the total number of ways this can be done is the product of 2! and 5!, or 240.
................................................ A. Friday
C. Sunday
795. There are six people to be seated on a bench for a picture. A certain trio, Vic,
Rayon, and Aira, do not want to be separated. How many ways can this be done? Explanation:
A. 60 B. 72 C. 144 D. 180
Explanation: Exactly 28 weeks or 196 days from now would be Monday again, therefore, 200 days from now would be Friday.
................................................
Let the trio take one unit, and then add the other three people to make four units. The trio can be
arranged in 3! ways and the four units can be arranged in 4! ways. 3! x 4! = 144.
................................................
799. If three-fourths of a number is 33 more than one-fifth of itself, then what is the
number?
A. 80 B. 60 C. 40 D. 20
Solution:
= + 33 Multiply equation by 20.
15x = 4x + 660
11x = 660
x = 60
802. The prime factorization of a number is given as 23 x 52 x 133. How many factors does it have?
A. 6:00 AM A. 96 B. 48 C. 36 D. 18
Solution:
C. 6:00 PM
Add 1 to the exponent of each prime factor, then multiply to get the number of factors.
(3 + 1)(2 + 1)(3 + 1) = 4 x 3 x 4 = 48
................................................
Explanation:
803. The prime factorization of a number is given as 22 x 32 x 7 x 11. How many factors does it have?
A. 96 B. 48 C. 36 D. 18
Solution:
2019 ÷ 24 = 84 r. 3 (2 + 1)(2 + 1)(1 + 1)(1 + 1) = 36
................................................
3 hours after 3 PM is 6 PM
................................................
804. There are ten red, nine blue, and six black pens in a bag. If a pen is randomly drawn from the bag,
what is the probability that it is red?
801. In a certain fastfood chain, soft drinks are served in Small, Medium, and Large cups. What level of
data is this?
A. Nominal
A.
C. Interval
Explanation:
A.
806. What does a probability of 0 pertain to? A. an impossible event
√6
Explanation:
(√a √b)2 = a + b – 2√
Therefore,
A. √2
√a + b 2√ab = √a √b
In the problem, a + b = 11 and ab = 30. The only pair that has a sum of 11 and a product of 30 is 6 and 5.
................................................
809. ∫(3 + 4)5 dx
C. √3
Explanation:
(√a + √b)2 = a + b + 2√ A. (3x + 4)6+ c
Therefore,
√a + b + 2√ab = √a + √b
In the problem, a + b = 10 and ab = 21. The only pair that has a sum of 10 and a product of 21 is 3 and 7.
................................................
C. 5(3x + 4)4+ c
∫(3
∫(3
................................................
810. ∫(6
A. 2x3 + 3x2 + 6x + c
C. 2x3 + 3x2 – 6x + c
Solution:
∫(6 +6
= +– 6x + c
= 2x3 + 3x2 – 6x + c
................................................
811. ∫
A.
C.
Solution:
∫
= +c
= +c
= +c
................................................
812. ∫
A.
C.
Solution:
∫
Author:
813.
Victor A. Tondo Jr., LPT
Mr. dela Cruz is going to present the growth of a certain plant over a span of 10 weeks. Which graph
she randomly picks a number from 1 to 5. All students whose number is that which she
picked will be sent to the clinic. What sampling method did Ms. Rowena use?
would be best suited for this?
A. cluster
A. bar graph
C. stratified
................................................
C. pie graph
816. What is the 7th decile of the following data?
Explanation:
Line graphs are best suited for presenting data on development over time.
................................................
814. Rayon is going to present the sales of six different teams in the company from January to June. Which
graph would be best suited for this?
A. bar graph
Since the divisor is x – 3, then the remainder is given by f(3). Author: Victor A. Tondo Jr., LPT 821. ( ) = _________
f(3) = 33 – 3(32) + 5(3) – 13
f(3) = 27 – 27 + 15 – 13
f(3) = 2
................................................
A.( )
B.(
818. (x2 + 3x – 2)4 = _________ )
A. (x2 + 3x – 2)3
B.
C.
4(x2 + 3x – 2)3
(8x + 12) (x2 + 3x – 2)3
C.( )
D. (2x + 3) (x2 + 3x – 2)3
D.( )
Solution:
Solution:
Let u = x2 + 3x – 2
u = 2x
v = 3x – 4
u4 = 4u3 du
= 4 (x2 + 3x – 2)3 (2x + 3)
= (8x + 12) (x2 + 3x – 2)3
................................................
819.
A. =
( )
................................................
822. There are 17 red bags, 19 green bags, and
14 blue bags in a store. What percent of the bags is blue?
A. 38% B. 34%
C. C. 28% D. 27%
Solution:
14/(17 + 19 + 14) = 14/50
√ 14/50 = 28%
................................................
Let u = x3 – 2x + 3
u1/2 = ½ u-1/2 du
= √
................................................
820. (2x + 3)(4x – 5) = _________
A. 8x2 + 2x + 15 B. 8x2 + 2x – 15 C.
8x2 – 2x – 15 D. 8x2 – 2x + 15
This is a free reviewer. All rights reserved.
1000 MMR
................................................
Explanation:
Explanation:
We are looking for a number x such that x leaves a remainder of 7 when divided by 9.
................................................
find m
That means:
(sin x) (cot x) = cos x;
A. 17o
(cos x) (csc x) = cot x;
(tan x) (csc x) = sec x; and
(cot x) (sec x) = csc x.
................................................
Explanation:
824. Which of the following could be the value of x if x 4 (mod 10)?
A. 46 B. 49 C. 52 D. 54
We are looking for a number x such that x leaves a remainder of 4 when divided by 10. Simply put, the number’s units means
digit is 4.
ABF.
Author: Victor A. Tondo Jr., LPT
827. If f(x) = x2 – 2x + 3 and g(x) = x + 1, find fog(x).
A. x2 – 1
C. x2 – 3x + 5
Solution:
f(g(x)) = f(x + 1)
= (x + 1)2 – 2 (x + 1) + 3
= (x2 + 2x + 1) – (2x + 2) + 3
= x2 + 2
................................................
828. How many 3-digit numbers can be formed using the digits 0, 1, 2, 3, 4, 5, and 6 if repetition is not
allowed?
A. 160 B. 180 C. 200 D. 210
Solution:
Use FCP (Fundamental Counting Principle):
__ x __ x __
For the first digit, we cannot use 0. That means we only have 6 choices for the first digit.
For the second digit, we can now use 0. Since we have already used one digit for the first, that means we
have 6 choices for the second digit. For the last digit, since we have already used two digits, we only have 5
choices.
6 x 6 x 5 = 180
................................................
̅̅
A.
VC
Explanation:
m A = 180 – (45 + 65) = 70
The longest side is opposite the largest angle, A.
................................................
830. Which quadrilateral has two congruent diagonals that are perpendicular to each other?
A. kite
C. rectangle
833. A bus drove for 7 hours at 73 kph and 3 hours at 88 kph. What was its average speed?
C. 77.5 kph
Get the total distance and the total time first. 7 hrs x 73 kph = 511 km 3 hrs x 88 kph = 264 km
Total distance = 775 km, total time = 10 hrs
Sum of grades of 7 students:
834. Fifteen guests shake hands with each other. If each guest is to shake hands with all the other guests, how
many handshakes will be made?
A. 105 B. 150 C. 210 D. 225
Solution: 837. A radius of a circle is 25 cm long. How long is its longest chord?
15C2 = 105
A. 20√2
C. 50 cm
Explanation:
The longest chord is the diameter, and the radius is half the diameter.
................................................
838. Find the largest area of a rectangular piece of land that can be enclosed with 800 meters of fencing
material.
A. 40,000 m2
C. 62,500 m2
Solution:
Instead of jumping to differential calculus (minima and maxima) to solve this, simply
make the rectangle a square. That’s the shortcut for this kind of question.
This is a free reviewer. All rights reserved.
1000 MMR
839. Which statistical test is used for testing for relationship between two variables?
find lim
A. ANOVA
A. 5
C. Pearson R
Explanation:
Observed vs Expected: Chi Square C. 16
Relationship: Pearson R (R for relationship)
Group differences: ANOVA (variance = differences) Solution:
Limit from the left: 2(5) + 4 = 14
Comparing sets of normal distributions: T-test ................................................ Limit from the right: 52 – 9 = 16
Since the limits are not equal, then the limit does not exist.
................................................
841. Find the equation of the line passing through (3, 7) and (-3, -5).
840. Given A. y = 2x + 1
C. y = 2x – 1
Author: Victor A. Tondo Jr., LPT
842. In which non-Euclidean model for geometry can we have any given line ℓ and
Solution: a point A which is not on ℓ, wherein all lines through A will intersect ℓ?
Two-point form of linear equations:
y – y1 = (x x )
A. hyperbolic
C. Saccheri
y–7= Explanation:
y – 7 = 2(x – 3)
y – 7 = 2x – 6
y = 2x + 1
In Euclidean geometry, only one line will pass through A. In elliptic geometry, all lines will pass through A and
................................................ intersect ℓ.
................................................
843. If A is at (-9, 11) and B is at (6,-9), find C if C is three-fifths the way from A to
B.
A. (1, -1)
C. (1, 1)
Solution:
A. (
C. (
Solution:
h=
h=
h=
h=
h=
h=
C. 598
Solution: 849. The sum of two numbers is 73 and their difference is 41. What is the larger number?
129 = 30% x Original number of chickens 129 ÷ 0.3 = Original number of chickens = A. 65
430 ................................................
846. Find the average rate of change of
y = x3 – 6x2 + 5x – 20 from x = 1 to x = 6. C. 57
A. 5
C. 7
Solution:
A. 1
C. 3
Explanation:
Simply divide the last two digits, 15, by 4 and get the remainder.
................................................
A. y = x + 1
C. y = 1 – x
y=x+1
850. An item is sold for P7,280 after being marked down by 30%. What was its original price?
y = 2x – 3
A. P11,400
y=1–x
y = 3 – 2x
C. P10,400 ................................................
Solution:
(100% - 30%) OP = 7280
70% OP = 7280
0.7 OP = 7280 852. Which of the following is outside the circle defined by the equation (x + 2) 2 + y2 = 50?
OP = 7280 ÷ 0.7
OP = 10,400
................................................
A. (-9, 1)
C. (-5, 7)
851. Which of the following lines passes through the point (-4, 5)?
This is a free reviewer. All rights reserved.
1000 MMR
Solution:
A point is outside the circle when its distance from the center is greater than the radius.
A: (-9, 1) (-9 + 2)2 + 12 = 50 This point is ON the circle.
Author: Victor A. Tondo Jr., LPT Explanation:
B: (-7, 5) (-7 + 2)2 + 52 = 50 This point is ON the circle.
C: (-5, 7) (-5 + 2)2 + 72 > 50 This point is OUTSIDE the circle. The third side of any triangle must be between the difference and the sum of the two other sides.
................................................
D: (2,4) (2+2)2+42<50 This point is IN the circle.
................................................
856. Rayon has 46 coins in P1 and P5 denominations for a total worth of P186. How many P5 coins
does he have?
A. 25 B. 30 C. 35 D. 40
853. Which of the following equations pertain to a parabola? Solution:
A. y2 – 5y = x2 + 4 B. x2 + 5x – 4y = 13 Let x = number of P5 coins
.: 46 – x = number of P1 coins
5(x) + 1(46 – x) = 186
5x + 46 – x = 186
5x – x = 186 – 46
4x = 140
x = 35
C. ................................................
857. If x = 2 and y = 3, what is x2 – y2 + 3xy?
A. 31 B. 13 C. -5 D. -13
Solution:
x2 – y2 + 3xy = 22 – 32 + 3(2)(3)
x2 – y2 + 3xy = 4 – 9 + 18 = 13
................................................
D. 858. In an arithmetic sequence, the first term is
Explanation: 200 and the common difference is -3. What is the 64th term?
A. 20 B. 17 C. 14 D. 11
Solution:
A parabola’s equation includes two variables, one of which is squared. An = A1 + (n – 1)d
A64 = A1 + (64 – 1)(-3)
................................................ A64 = 200 + 63(-3)
A64 = 200 – 189
A64 = 11
................................................
859. In an arithmetic sequence, the 23rd term is
854. Which of the following equations pertain to a parabola that opens to the left? 157 and the common difference is -4. What is the 34th term?
A. -2(y + 7) = (x – 3)2 B. 5(y – 9) = (x – 5)2 A. 113 B. 117 C. 121 D. 125
The parabola opens to the left when the squared variable is y and the coefficient of x is negative.
................................................
855. The lengths of two sides of a triangle are 7 inches and 15 inches. Which of the
following represents x, the possible length in inches of the remaining side of the triangle?
A. 8 < x < 22
C. x < 8 or x > 22
An = Am + (n – m)d
A34 = 113
................................................
A. √ + 4 +3
C. √ 4 +3
Solution:
y – 5 = x2 – 6x
y + 4 = x2 – 6x + 9
y + 4 = (x – 3)2
√ +4=√(
√ +4=x–3
√ +4 +3=x
√ +4 + 3 = y-1
................................................
861. ln e4 = _____
A. 24.1295
C. 6.032375
Explanation:
ln ea = a
................................................
A. 5 B. 6.25
Explanation:
eln a = a
................................................
863. What is ln e?
A. 0 B. 1
................................................
A. Sumerians
C. Chinese
1000 MMR
872. Which Mathematician developed the use of AND, OR, and NOT operators in Algebra?
A. George Boole B. Janos Bolyai
C. Charles Babbage D. August Mobius
................................................
C. Indians
879. Convert rad to degrees.
A. 70o B. 105o
................................................ C. 210o D. 420o
Solution:
rad x = 105o
875. Notched tally bones proved their use of Mathematics around 35000 BCE. 880. Find the volume of a cylinder whose radius is 50 cm and height is 24 cm.
A. Africans A. 20,000 π cm3
Explanation: 881. Find the altitude to the hypotenuse of a right triangle whose legs measure 16 cm and 30 cm.
C. 59 kph
C. 34√2
884. Find the surface area of a sphere whose radius is 24 cm. 887. In solid geometry, what do you call a solid bound by polygons?
A. tessellation
A. 192 π cm2
C. polyhedron
C. 1152 π cm2
................................................
Solution:
Surface Area = 4 π r2
Surface Area = 4 (242) π = 2304 π
888. Two triangles have a pair of congruent angles. Which of the following is true about these triangles?
C. P560,000
Solution:
Since the interest is compounded annually, Acct = Principal x (1 + rate)time Acct = 500,000 x (1.043)
Acct = 500,000 x 1.124864 = 562,432
................................................
This is a free reviewer. All rights reserved.
1000 MMR Author: Victor A. Tondo Jr., LPT
890. Find the range of the following scores:
894. The amount of money you have falls under what level of data?
A. nominal
C. interval
................................................
Explanation:
Range = Highest Score – Lowest Score
................................................
891. What is the 9th decile of the following data? 895. To study tooth decay a researcher takes a sample at random but with the stipulation that all
age groups are represented proportionally. What sampling method did the researcher use?
A. systematic
C. stratified
Explanation:
The age groups are the different strata or subgroups of the population presented proportionally.
................................................
A. 34
Explanation:
(-2019)2 = 4,076,361
-20192 = -4,076,361
................................................ 898. How long is the latus rectum of the parabola defined by (y + 3) 2 = 20x –
196?
A.3 B.12 C.20 D.19
893. The product of two numbers is 1425. If each of the two numbers is doubled, the product of these larger
Explanation:
numbers is _____.
The length of the latus rectum is equal to the coefficient of the non-squared variable.
................................................
A. 2,850
C. 8,550
Solution:
Let xy = 1425
(2x)(2y) = 4xy
(2x)(2y) = 4(1425)
(2x)(2y) = 5700
A. cos M = sec N x
x + 4 = 194 + 4 = 198
................................................
C. tan M = csc N
Explanation:
Sine and Cosine are trigonometric cofunctions. 905. x varies directly as y and inversely as z. If x = 20 when y = 10 and z = 3, what is x when y = 15 and
................................................
z = 9?
A. 10
902. The top ten students of a graduating class got the following scores in their final examination in
Calculus:
Solution:
Solution:
Sum of scores = 881
881 ÷ 10 = 88.1
................................................ x = 6y/z
x = 6(15)/9
x = 90/9
x = 10
................................................
903. The top ten students of a graduating class got the following scores in their final examination in 906. Factorize 15x2 – 13x – 20.
Calculus:
A. (5x – 4)(3x – 5)
C. (5x + 4)(3x – 5)
................................................
C. x = 3
Explanation:
909. Find the instantaneous rate of change for f(x) = 4x3 + 7x2 – 27x – 49 at x = 1.
A.-2 B.-1 C.1 D.2
A. 6
Solution:
Find f ’(1).
f ’(x) = 12x2 + 14x – 27
C. 11 f ’(1) = 12(12) + 14(1) – 27
f ’(1) = -1
Solution: ................................................
910. log3 24 – 3 log3 2 = _____
Complete the squares on the left side of the equation to return it to its center-radius form.
x2 + y2 + 12x – 14y = 36 A. 1
x2 + 12x + y2 – 14y = 36
(x2 + 12x + 36)+ (y2 – 14y +49)= 36 + 36 + 49 (x2 + 12x + 36)+ (y2 – 14y +49)= 121
................................................
C. -1
Solution:
log3 24 – 3 log3 2 = log3
= log3 3
= 1
................................................
Author: Victor A. Tondo Jr., LPT Solution:
911. After using one-sixth of her budget on bills, two-fifths on groceries, and P1700
on books and magazines, Mrs. Lazada still had P7400 left. How much was her budget?
A. P20,000 B. P21,000
C. P22,500 D. P30,000
21000 = x
................................................
912. An assistant’s response times were recorded on the table below. What is her average response time in
minutes?
A. 3.625
Solution:
First, insert a column for fx, which is the product of the frequency and response time in each
row.
∑ = 143
n = 40
143 ÷ 40 = 3.575
................................................
913. The average grade of 22 students in Section Abaca is 95, while the average grade of 28
students in Section Acacia is 89. What is the average grade of all 50 students in both sections?
A. 92.36 B. 91.64 C. 90.72 D. 89.8
This is a free reviewer. All rights reserved.
1000 MMR
5x = -25
Solution:
................................................
x = -5; y = 2(-5) + 23 y = 13
916. A book was sold for P630 after a 10% discount was given. How much was the book originally?
Average =
................................................
914. Find the slope of 2x – 6y = 13.
A. P800
A. 3 C. P700
Solution:
SP = OP (1 – DC Rate)
630 = OP (0.9)
630/0.9 = OP
700 = OP
................................................
Solution:
917. If the sum of the supplement and the complement of an angle is 146, what is the angle?
A. 61 B. 62 C. 63 D. 64
Convert it to its slope-intercept form.
2x – 6y = 13
y= x+
.: m = or
................................................
C. (5, -13)
Solution:
y = -3x – 2
(-)
y = 2x + 23
0 = -5x – 25
3x + 5x + 4x = 2460
12x = 2460
x = 205
Amitaf’s share is 3(205) or P615.
................................................
920. Fifty-four kilometers per hour is equal to how many meters per second?
Author: Victor A. Tondo Jr., LPT Solution: A. 15 B. 30 C. 54 D. 60
(180 – x) + (90 – x) = 146
270 – 2x = 146
Solution:
270 – 146 = 2x = =
124 = 2x
................................................
62 = x
................................................
918. A bag contains some marbles. When the marbles are grouped by 2, 3, 4, 5, or 6, there is 921. Simplify .
always one marble left. Which of the following could be the number of marbles in the bag?
A. 31 B. 41 C. 51 D. 61
Explanation:
N – 1 should be divisible by 2, 3, 4, 5, and 6.
................................................
A. 1
919. Amitaf, Bernard, and Chloe share a total of P2,460 in the ratio 3:5:4 respectively. How much is
Amitaf’s share?
A. P600
C. P630
Solution:
This is a free reviewer. All rights reserved.
1000 MMR
Solution:
()
= = =1
................................................
922. If x = 10, which of the following is equal to
219?
Explanation:
Just substitute x with 10.
A. 23(10) – 1 = 229
B. 2(10)2 + 5(10) + 5 = 255
C. 2(10)2 + 2(10) – 1 = 219
D. (10)2 + (10) + 2 = 112
................................................
926. Simplify:
924. What is the measure of each interior angle of a regular 30-sided polygon?
Solution:
A. 158o
x = x
x =
C. 165o x =
Solution: ................................................
MIA = 180 – 927. Which of the following is a diagonal matrix?
MIA = 180 –
MIA = 180 – 12
MIA = 168
................................................
A.[ 2
925. Simplify:
A. csc x – tan x
C.
C. csc x – cot x
Explanation:
A diagonal matrix is a square matrix where all its non-diagonal elements are 0.
................................................
928. Which of the following is a scalar matrix?
Explanation:
A scalar matrix is a diagonal matrix where all its leading diagonal elements are of equal value.
................................................
1
of A = *
5
A.
A.AT=*1 4
6
C. [1
Explanation: C.AT=
Explanation:
=
= 2
931. When dealing with matrices A, B, and C, which of the following is always true? x
x + 2 = 2(x – 1)
+ 2 = 2x – 2
A.AB=BA 2 + 2 = 2x – x
B. If AB = 0, then A = 0 or B = 0 C. If AB = AC, then B = C 4=x
................................................
D. None of the above.
................................................ 933. What are the zeros of 12x2 – x – 35?
932. If the numbers x-1, x+2 and 2x+4 are consecutive terms of a geometric sequence, what is x?
A.
A. 1
C. 4
C.
Solution:
(4x – 7)(3x + 5) = 0
4x – 7 = 0
................................................
C. 1980o
Solution:
SIA = 180(n – 2)
SIA = 1800
................................................
935. 45x – 20 = _____.
A. 45x – 20
C. 20(45x
Solution:
au = au du
Let u = 5x – 20
45x – 20
D.
Explanation:
A positive leading coefficient means the right tail must point upward. An even degree
A. means that the left tail points to the same direction (up) as the right tail.
................................................
B.
C.
Author: Victor A. Tondo Jr., LPT
937. Which of the following graphs have a negative leading coefficient and an odd degree?
A.
D.
B. Explanation:
A negative leading coefficient means the right tail must point downward. An odd degree
means that the left tail points to the opposite direction as the right tail.
................................................
C.
939. What can be said about the equation f(x) of the following graph?
941. If (
Since the right tail is pointing downward, the leading coefficient is negative. Solution:
The left tail points upward (opposite of right tail); therefore, the degree is odd.
Since the graph touches the y-axis above the origin, then the constant is positive.
................................................
lim
lim
940. If
Since lim
therefore, limit does not exist.
................................................
A. Limit does not exist. 942. Give the domain of f(x) = √8 + 24.
C. 13 A. x | x
C. x > 24
Solution: Explanation:
The given function is a radical function, more specifically a square root function. If the radicand 8x +
24 is negative, f(x) becomes imaginary. That means 8x + 24 must be greater than or equal to 0.
lim 8x + 24 0-24
8x
x -3
................................................
lim
943. What is the 3rd term in the expansion of (A + B)5?
Since lim A. 5A4B
C. 10A2B3
lim
Solution:
1000 MMR
944. Find the remainder when
Solution:
Use the Remainder Theorem.
2x4 – 5x3 + 3x2 – 5x + 7 is divided by (x – 1).
A.1 B.2 C.4 D.7
2(14) – 5(13) + 3(12) – 5(1) + 7 = r 2 – 5 + 3 – 5 + 7 = r 2=r
................................................
945. Give the range of f(x) = Author: Victor A. Tondo Jr., LPT
946. In the arithmetic sequence -2, 1, 4, 7, …, which term is 100?
A. 34th B. 35th C. 36th D. 37th
Solution:
A. y | y An = A1 + (n – 1)d
100 = -2 + (n – 1)(3)
100 = -2 + 3n – 3
100 = 3n – 5
105 = 3n
35 = n
................................................
C. y
947. Find the length of the intercepted arc of a central angle measuring 45 o given the radius is
80 cm.
A. 20 π cm
Solution:
First, express y in terms of x.
y= C. 40 π cm
(5x – 8)y = 3x + 4
5xy – 8y = 3x + 4
5xy – 3x = 8y + 4
x (5y – 3) = 8y + 4
x=
The denominator 5y – 3 cannot be equal to 0. Solution:
5y – 3
5y
y Arc length =
Shortcut:
Since the numerator and denominator are of the same degree, just take the leading coefficients of the numerator (3)
and denominator (5).
Proof: Arc length =
Cross-multiply. Arc length = 20 π
................................................
ARoC =
................................................
24
957. Find the equation of the line with slope 3, passing through (2, 5).
A. y = 3x – 2
A. 24
C. y = 3x + 1
Solution:
Solution:
Mean =
................................................ y = mx + b
y = 3x + b
952. Rayon’s Social Security System (SSS) ID number is 02-2525255-2. What level of data is the SSS ID?
5 = 3(2) + b
A. nominal -1 = b
.: y = 3x – 1
C. interval
................................................
Explanation:
ID numbers fall under the nominal level of data.
................................................
958. Find the distance of the point (7, 8) from the line 2x – 3y + 4 = 0
953. A tree is 4 meters tall, while the flagpole is 8 meters tall. What level of data is used?
A. nominal
C. interval
Explanation:
We can infer that the flagpole is literally twice as tall as the tree. Height or tallness falls under the ratio level of data.
................................................
954. The freezing temperature of water is 0o Celsius. What level of data is temperature in Celsius?
A.
A. nominal
C. interval
Explanation: C. 4√73
0o Celsius does not mean the absence of temperature or heat. Temperature in Celsius Solution:
Use the formula for distance of point (x1, y1)
(and also in Fahrenheit) falls under the interval level of data. from line Ax + By + C = 0:
D=
D=
D=
y = 2x – 1 and y = 2x + 4.
A. 5 B. 2√5
Solution:
D=
( )
D= √ =√ = √5
................................................
A. 5 B. 10
Solution:
D = number of ducks
C+D=80
4C+2D=190
2(C+D=80)
4C+2D=190
................................................
A. 2570o
C. 7830o
Solution:
= 6.722…
= 14.833…
= 21.333…
= 27
Explanation:
Just use your calculator.
This is a free reviewer. All rights reserved.
A=
................................................
A. LDNE
= lim
971. The distance D of a projectile from the ground t seconds after launch is given as
= lim (
D = 14t – t2. How many seconds after launch does it attain its peak?
A.4 B.6 C.7 D.8
=
=
32+3(3)+9
27
Solution:
................................................ D = 14t – t2
D’ = 14 – 2t first derivative
Max height or peak is attained when D’ = 0.
14 – 2t = 0
14 = 2t
967. Which of the following circles is concentric with x2 + y2 – 7x + 9y = 25? 7=t
A.A=
C. A = 16P2
A. 16 m
C. 32 m
Solution:
D = 16t – 2t2
D’ = 16 – 4t first derivative
Max height or peak is attained when D’ = 0.
16 – 4t = 0
16 = 4t
4=t
Max height = 16(4) – 2(4)2 Explanation:
= 64–32
= 32
................................................
A diagonal matrix is a square matrix where all its non-diagonal elements are 0. (See item #927)
973. Find the slope of the line tangent to the graph of f(x) = x3 – 3x2 + 5x – 1 at x = 1.
A.-1 B.0 C.1 D.2
Solution:
f ‘(x) = 3x2 – 6x + 5
f ‘(1) = 3 – 6 + 5
f ‘(1) = 2
................................................
974. Which of the following is a diagonal matrix?
Author: Victor A. Tondo Jr., LPT
975. Which of the following is a scalar matrix?
B.
Explanation:
An identity matrix is a scalar matrix where the
elements on its leading diagonal are 1 and the
rest are of value 0.
(See item #929)
................................................
977. Find the quotient when the polynomial 7x5 – 3x3 + 2x2 – 9x + 3 is divided by x – 1. A. 7x4
+ 7x3 + 4x2 + 6x – 3
B. 7x4 + 7x3 + 4x2 – 6x – 3 C. 7x4 + 7x3 – 4x2 – 6x – 3 D. 7x4
– 7x3 – 4x2 – 6x – 3
Explanation: Solution:
A scalar matrix is a diagonal matrix where all its leading diagonal elements are of equal value. (See item
#928)
................................................ Use synthetic division.
976. Which of the following is an identity matrix?
Solution:
979. There are 210 candies in a jar. The ratio of red candies to blue is 2:3, and the
ratio of blue to yellow is 4:5. How many yellow candies are there?
Solution:
982. Given f(x) = x2 + 10x + 25 and g(x) = 3x + 1, find f(g(x)).
First, make a common ratio. A. 3x2 + 30x + 26
C. 9x2 + 30x + 26
Red Solution:
2
f(g(x)) = (g(x))2 + 10(g(x)) + 25 f(g(x)) = (3x + 1)2 + 10(3x + 1) + 25
f(g(x)) = (9x2 + 6x + 1) + (30x + 10) + 25 f(g(x)) = 9x 2 + 36x + 36
................................................
983. A whole number is 3 more than another number. The sum of their squares is 4145. Find the larger
8 number.
A. 47 B. 46 C. 45 D. 44
8x + 12x + 15x = 210
35x = 210
Solution:
x=6 Let x = larger number
Number of yellow candies = 15x = 15(6) = 90 x – 3 = smaller number
................................................ x2 + (x – 3)2 = 4145
x2 + (x2 – 6x + 9) = 4145
2x2 – 6x + 9 = 4145
2x2 – 6x = 4136
x2 – 3x = 2068
x2 – 3x + 2.25 = 2068 + 2.25
980. A certain bacteria doubles its population after 3 minutes. If the bacteria in a x – 1.5 = 45.5
(x – 1.5)2 = 2070.25
petri dish is 512,000 at 8:45 AM, at what time was its population count 125? x = 47
A. 6:30 AM
C. 8:09 AM
Solution:
125 x 2n = 512,000
2n = 4096
2n = 212
n = 12
12(3) = 36
36 minutes before 8:45 AM is 8:09 AM.
This is a free reviewer. All rights reserved.
1000 MMR
984. The numbers x, y, z, and w have an average of 30. If x, y and z have an average 35, what is w? ................................................
A.5 B.10 C.15 D.20
Solution:
x + y + z + w = 4(30)
x + y + z + w = 120
x + y + z = 3(35)
x + y + z = 105
120 – 105 = 15
................................................
985. A is a constant. Find A such that the equation 2x + 1 = 2A + 3(x + A) has a solution at x = 2.
A. -0.4 B. -0.2 C. 0 D. 0.2
Solution:
2(2)+1=2A+3(2+A)
5=2A+6+3A
-1=5A
-0.2 = A
................................................
986. It takes 4 men 9 days to build 2 houses. How many days will it take 6 men to build 5 houses?
A. 12
Solution:
4m (9d) = 2h
36md = 2h
18md = h
Author: Victor A. Tondo Jr., LPT
988. In a certain university, a student’s grade is computed as the sum of 25% of his prelims grade,
30% of his midterms grade, and 45% of his finals grade. He knows that his prelims grade is 72 and his Solution:
midterms grade is 70. What is his grade for the finals if he got a grade of 75 in Calculus?
A. 75 B. 77 C. 79 D. 80
Solution:
0.25(72) + 0.3(70) + 0.45(X) = 75
18 + 21 + 0.45X = 75
39 + 0.45X = 75
0.45X = 36 MUR =
X=80
................................................
989. Mr. Lazada bought 75 pieces of Elunium for 8,000 per piece and sold them for a total of 720,000. What is
his mark-up rate?
MUR =
A. 20%
MUR = 20%
................................................
C. 32.5%
990. Triangular numbers are numbers that can be shown by triangular arrangements of dots. The
triangular numbers are 1, 3, 6, 10, 15, 21, …
What is the 20th triangular number?
A. 200 B. 205 C. 210 D. 220
Explanation:
A. 18 B. 24
Solution:
860 = 32x
(23)60 = (25)x
2180 = 25x
180 = 5x
36 = x
A. nominal scale
993. Point V is two-fifths the way from A(9, -7) to B(-6, 13). Find the coordinates of point V.
C. (3, 1)
997. The GCF of two numbers is 8 and their LCM is 80. What is their product?
A. 80 B. 160 D. 320 D. 640
Explanation:
Solution:
The product of two numbers is equal to the product of their GCF and LCM.
................................................
A. 784
x=9+ Solution:
c1.5 – 4 = 7
x =9+(-6) c1.5 = 11
x =3 (c1.5)2 = 112
c3 = 121
y = (-7)+ (13 – (-7)) ................................................
y = (-7)+ (20)
y = (-7)+ 8 y = 1
................................................ 999. A certain Math challenge gives the competitors a score of 4 for each correct
answer, and a deduction of 1 point for each wrong answer. If a contestant answered all
100 items and got a score of 200, how many items did the contestant answer correctly?
994. Find the angle of inclination of the line passing through V(5, -5) and T(2, 4) A. 45 B. 50 C. 60 D. 35
A. 88.145o
C. 97.593o
Solution:
( )
Slope = = -3
Angle of inclination = tan -1 (-3)
Angle of inclination = -71.565o or 108.435o
C. 217
Solution:
101011012 ____10
1 x 27 = 128
1 x 26 =
1 x 25 =
0 x 24 = 0
1 x 23 = 8
1 x 22 = 4
0 x 21 = 0
1x 20 = 1
End of 1000MMR.
Congratulations!!